You are on page 1of 61

CHAPTER 1 ■ Safety and Quality in the Hematology Laboratory 21

Exposure to Bloodborne Pathogens regulation require that Quality Assessment in the Hematology
laboratories develop, implement, and comply with a plan Laboratory
that ensures the protective safety of laboratory staff to poten-
tial infectious bloodborne pathogens, HBV and HIV. The law The assessment of quality results for the various analyses is
further specifies the rules for managing and handling medi- critical and is an important component of the operation of a
cal waste in a safe and effective manner. high-quality laboratory. Quality assessment is used in the clini-
Blood is the most frequently implicated infected body cal hematology laboratory to ensure excellence in performance.
fluid in HIV and HBV exposure in the workplace. An occu- A systematic approach to quality assures that correct labora-
pational exposure is defined as a percutaneous injury, for tory results are obtained in the shortest possible time and at
example, needlestick or cut with a sharp object, or contact a reasonable cost. A quality assessment system is divided into
by mucous membranes or nonintact skin, or the contact is two major components: nonanalytical factors and the analysis
prolonged or involves an extensive area with blood, tissues, of quantitative data (QC). Nonanalytical factors that support
blood-stained body fluids, body fluids to which standard quality testing include qualified personnel, laboratory policies,
precautions apply, or concentrated virus. The most wide- laboratory procedure manual, test requisitioning, patient iden-
spread control measure required by OSHA and CLSI is tification, and specimen procurement and labeling; specimen
the use of puncture-resistant sharps containers. An occu- collection, transport, and processing and storage; and preven-
pational exposure should be considered to be an urgent tive maintenance of equipment, appropriate methodology,
medical concern to ensure timely postexposure manage- and accuracy in reporting results and documentation. Delta
ment. After skin or mucosal exposure to blood, the ACIP checks are particularly important to rule out mislabeling, cleri-
recommends immunoprophylaxis, depending on several cal error, or possible an analytical (examination) error.
factors.
Quality Control in the Hematology Laboratory
Safe Work Practices and Protective
Techniques for Infection Control QC monitors the accuracy and precision of test performance
over time. The purpose of QC is to detect errors that result
Each laboratory must have an up-to-date safety manual. This from test system failure, adverse environmental conditions,
manual contains a comprehensive listing of approved poli- and variance.
cies, acceptable practices, and precautions including standard It is important for hematology technologists and techni-
precautions. Standard precautions represent an approach to cians to understand basic statistical concepts used in QC.
infection control used to prevent occupational exposures to Knowledge of specific elements of statistics is important in
bloodborne pathogens. hematology in order to apply statistical analysis of results
Gloves should be used as an adjunct to, not a substitute and in instrumental applications of statistics to erythrocyte,
for, handwashing. All work surfaces are cleaned and sanitized leukocyte, and platelet reports.
at the beginning and end of the shift with a 1:10 dilution of Statistical analysis of results has been used in the clini-
household bleach or an EPA-registered disinfectant. A vari- cal laboratory since the original introduction of the Levey-
ety of other safety practices should be adhered to, to reduce Jennings chart. With the advent of computer technology and
the risk of inadvertent contamination with blood or certain computerized instrumentation in hematology, many addi-
body fluids. Protective gloves should always be worn for han- tional systems have been introduced to monitor test results
dling any type of biological specimen. numerically.

REVIEW QUESTIONS

1. The function (or functions) of a hematology laboratory C. comply with local health and state regulatory
is (are) to requirements
A. confirm the physician’s impression of a possible he- D. comply with OSHA regulations
matological disorder 3. Which of the following is not an appropriate safety
B. establish or rule out a diagnosis practice?
C. screen for asymptomatic disorders A. Disposing of needles in biohazard, puncture-proof
D. all of the above containers
2. The major intended purpose of the laboratory safety B. Frequent handwashing
manual is to C. Sterilizing lancets for reuse
A. protect the patient and laboratory personnel D. Keeping food out of the same areas as specimens
B. protect laboratory and other hospital personnel

(continued)

Turgeon_Chap01.indd 21 10/13/2010 6:28:27 PM


22 PART 1 ■ The Principles of Hematology

REVIEW QUESTIONS (continued)

4. If a blood specimen is spilled on a laboratory bench or Questions 12 and 13: Diluted bleach for disinfecting work
floor area, the first step in cleanup should be surfaces, equipment, and spills should be prepared daily by
A. wear gloves and a lab coat preparing a _____ (12) dilution of household bleach. This
B. absorb blood with disposable towels dilution requires _____ (13) mL of bleach diluted to 100 mL
C. clean with freshly prepared 1% chlorine solution with H2O.
D. wash with water 12.
5. Which of the following procedures is the most basic and A. 1:5
effective in preventing nosocomial infections? B. 1:10
A. Washing hands between patient contacts C. 1:20
B. Wearing laboratory coats D. 1:100
C. Isolating infectious patients 13.
D. Isolating infectious specimens A.1
6. The likelihood of infection after exposure to HBV-in- B.10
fected or HIV-infected blood or body fluids depends on C.25
all of the following factors except the D.50
A. source (anatomical site) of the blood or fluid 14. The laboratory procedure manual does not need to
B. concentration of the virus include
C. duration of the contact A. test method, principle of the test, and clinical appli-
D. presence of nonintact skin cations
7. HBV and HIV may be directly transmitted in the occu- B. specimen collection and storage procedures
pational setting by all of the following except C. the name of the supplier of common laboratory
A. parenteral inoculation with contaminated blood chemicals
B. exposure of intact skin to contaminated blood or D. QC techniques, procedures, normal values, and
certain body fluids technical sources of error
C. exposure of intact mucous membranes to contami- 15. Which of the following statements is not a nonanalytical
nated blood or certain body fluids factor in a Quality Assessment system?
D. sharing bathroom facilities with an HIV-positive A. Qualified personnel and established laboratory
person policies
8. Standard precautions have been instituted in clinical B. Monitoring the standard deviation and reporting
laboratories to prevent _____ exposures of healthcare results of normal and abnormal controls
workers to bloodborne pathogens such as HIV and C. Maintenance of a procedure manual and the use of
HBV. appropriate methodology
A. parenteral D. Preventive maintenance of equipment and correct
B. nonintact mucous membrane specimen collection
C. nonintact skin 16. In which of the following laboratory situations is a ver-
D. all of the above bal report permissible?
9. Exposure to _____ constitutes the major source of HIV A. When the patient is going directly to the physician’s
and HBV infection in healthcare personnel. office and would like to have the report available
A. sputum B. When the report cannot be found at the nurse’s station
B. blood C. When emergency test results are needed by a
C. urine physician
D. semen D. None of the above
10. The transmission of HBV is _____ probable than trans-
mission of HIV. Questions 17 through 19: Match the following terms with
A. less the best description.
B. more 17. _____ Accuracy A. The value is known in a speci-
11. Gloves for medical use may be 18. _____ Calibration men similar to a patient’s
A. sterile or nonsterile 19. _____ Control whole blood or serum.
B. latex or vinyl B. Closeness to the true value
C. used only once C. The process of monitoring
D. all of the above accuracy
D. Comparison to a known physi-
cal constant
(continued)

Turgeon_Chap01.indd 22 10/13/2010 6:28:27 PM


CHAPTER 1 ■ Safety and Quality in the Hematology Laboratory 23

REVIEW QUESTIONS (continued)

Questions 20 through 22: Match the following terms with 29. The z score measures
the best description. A. how many standard deviations a particular number
20. _____ Precision A. How close test results are when is from the right or left of the mean
21. _____ Standards repeated. B. the sum of the squared differences from the mean
22. _____ Quality B. A purified substance of a known C. the square root of the variance from the mean
composition. D. the expression of the position of each test result to
C. The process of monitoring the average
accuracy and reproducibility 30. Acceptable limits of a control value must fall
of known control results. A. within ±1 standard deviation of the mean
D. The value is unknown. B. between 1 and 2 standard deviations of the mean
C. within ±2 standard deviations of the mean
23. Which of the following is not a function of a quantitative
D. within ±3 standard deviations of the mean
QC program? 31. A trend change in QC data is
A. Monitors the correct functioning of equipment, re-
A. a progressive change all in one direction away from
agents, and individual technique the mean for at least 3 days
B. Confirms the correct identity of patient specimens
B. an abrupt shift in the control values
C. Compares the accuracy of controls to reference val-
C. scattered variations from the mean
ues D. a progressive change in various directions away from
D. Detects shifts in control values
the mean for at least 1 week
32. A continuously increasing downward variation in a con-
Questions 24 through 27: Match the following terms with
trol sample in one direction from the mean can indi-
the appropriate description.
cate
24. _____ Mean A. The difference between the A. deterioration of reagents used in the test
25. _____ Range upper and lower measure- B. deterioration of the control specimen
26. _____ Variance ments in a series of results C. deterioration of a component in an instrument
27. _____ Standard B. The expression of the position of D. all of the above
deviation each test result to the average 33. Which of the following statements is true of a gaussian
C. The arithmetic average curve?
D. The degree to which test data A. It represents the standard deviation.
vary about the average B. It represents the coefficient of variation.
28. The coefficient of variation is the C. It represents variance of a population.
A. sum of the squared differences from the mean D. It represents a normal bell-shaped distribution.
B. square root of the variance from the mean 34. Two standard deviations (2 SD) from the mean in a nor-
C. standard deviation expressed as a percentage of the mal distribution curve would include
mean A. 99% of all values
D. degree to which test data vary about the average B. 95% of all values
C. 75% of all values
D. 68% of all values

BIBLIOGRAPHY Dunikoski LK. Take pride in SAFEty: A comprehensive lab-safety pro-


gram, Med Lab Observer, 35(10):28–31, 2003.
Clinical and Laboratory Standards Institute (CLSI). Clinical labora- Ferdinand M. OSHA’s bloodborne pathogens standard: enforcement,
tory waste management: approved guideline, 2nd ed, Wayne, PA, compliance and comment, J Healthc Mater Manag, 11(8):12–14,
GP5-A2, 2002. 1993.
Clinical Laboratory and Standards Institute (CLSI): Clinical laboratory Gile TJ. Laboratory training: safety at any age, Med Lab Observer,
safety: approved guideline, 2nd ed, Wayne, PA, GP17-A2, 2004. 37(8):28, 2005.
CLSI Clinical and Laboratory Standards Institute (CLSI) Protection of Harty-Golder B. Prepare for occupational bloodborne pathogen expo-
laboratory workers from infectious disease transmitted by blood, sure. Med Lab Observer, 41(4):40, 2009.
body fluids, and tissue: tentative guideline, 3rd ed, Wayne, PA, Kaplan LA, Pesce AJ. Clinical chemistry: theory, analysis, and correla-
M29-A3, 2005. tion, 4th ed, St Louis, MO, Mosby, 2004.
DeCraemer D. Postmortem viability of human immunodeficiency Larson EL. APIC guideline for hand washing and hand antisepsis in
virus—implications for the tracking of anatomy, N Engl J Med, health-care settings, Am J Infect Control, 23:251–269, 1995.
33(19):1315, 1994.

Turgeon_Chap01.indd 23 10/13/2010 6:28:27 PM


CHAPTER 2 ■ Principles of Blood Collection 47

REVIEW QUESTIONS

1. When the coagulation of fresh whole blood is prevented 14. A blood sample is needed from a patient with IV fluids
through the use of an anticoagulant, the straw-colored running in both arms. Which of the following is an
fluid that can be separated from the cellular elements is acceptable procedure?
A. serum A. Any obtainable vein is satisfactory.
B. plasma B. Obtain sample from above the IV site.
C. whole blood C. Obtain sample from below the IV site with special
D. platelets restrictions.
2. Which characteristic is inaccurate with respect to the D. Disconnect the IV line.
anticoagulant K3 EDTA? E. Do not draw a blood specimen.
A. Removes ionized calcium (Ca2+) from fresh whole 15. The bevel of the needle should be held _____ in the per-
blood by the process of chelation formance of a venipuncture.
B. Is used for most routine coagulation studies A. sideways
C. Is the most commonly used anticoagulant in hema- B. upward
tology C. downward
D. Is conventionally placed in lavender-stoppered evac- D. in any direction
uated tubes 16. A hematoma can form if
3. Heparin inhibits the clotting of fresh whole blood by A. improper pressure is applied to a site after the veni-
neutralizing the effect of puncture
A. platelets B. the patient suddenly moves and the needle comes
B. ionized calcium (Ca2+) out of the vein
C. fibrinogen C. the needle punctures both walls of the vein
D. thrombin D. all of the above
17. Phlebotomy problems can include
Questions 4 through 7: Match the conventional color- A. the use of improper anticoagulants
coded stopper with the appropriate anticoagulant. B. misidentification of patients
4. _____ EDTA A. Red C. improper angle of the needle or having the needle up
5. _____ Heparin B. Lavender against the side of the vessel wall
6. _____ Sodium citrate C. Blue D. all of the above
7. _____ No anticoagulant D. Green 18. Which of the following skin puncture areas is (are)
acceptable for the collection of capillary blood from an
Questions 8 through 12: The following five procedural infant?
steps are significant activities in the performance of a veni- A. Previous puncture site
puncture. Place these steps in the correct sequence. B. Posterior curve of the heel
8. _____ A. Select an appropriate site and prepare C. The arch
9. _____ the site. D. Medial or lateral plantar surface
10. _____ 19. The proper collection of capillary blood includes
B. Identify the patient, check test req-
11. _____ A. wiping away the first drop of blood
12. _____ uisitions, assemble equipment, wash
hands, and put on latex gloves. B. occasionally wiping the site with a plain gauze pad to
C. Remove tourniquet, remove needle, apply avoid the buildup of platelets
pressure to site, and label all tubes. C. avoiding the introduction of air bubbles into the col-
D. Reapply the tourniquet and perform umn of blood in a capillary collection tube
the venipuncture. D. all of the above
E. Introduce yourself and briefly explain 20. A peripheral blood smear can be prepared from
the procedure to the patient. A. EDTA-anticoagulated blood within 1 hour of collec-
tion
13. The appropriate veins for performing a routine B. free-flowing capillary blood
venipuncture are the C. citrated whole blood
A. cephalic, basilic, and median cubital D. both A and B
B. subclavian, iliac, and femoral
C. brachiocephalic, jugular, and popliteal
D. saphenous, suprarenal, and tibial

(continued)

Turgeon_Chap02.indd 47 10/13/2010 6:30:06 PM


48 PART 1 ■ The Principles of Hematology

REVIEW QUESTIONS (continued)

21. Identify the characteristic(s) of a good peripheral blood 25. Appropriate bone marrow aspiration sites in an adult
smear. are the
A. It progresses from thick at the point of origin to A. anterior and posterior iliac crest
thin. B. sternum and posterior iliac crest
B. It has a blunt feathered termination. C. tibia and sternum
C. The outer margins do not touch the edges of the D. both A and B
slide.
D. All of the above. Questions 26 through 28: Match the following type of
22. Poor blood smears can be caused by staining effect with the color it imparts to blood cells.
A. a delay in preparing the smear once the drop of 26. _____ Basic stain A. Orange-red color
blood has been placed on the slide 27. _____ Acidic stain B. Pink-lilac color
B. a drop of blood that is too large or too small 28. _____ Neutrophilic C. Blue-purple color
C. holding the pusher slide at the wrong angle and poor
drying conditions Questions 29 through 32: Identify the following as
D. all of the above Romanowsky-type or non–Romanowsky-type stains.
23. If a blood smear is too long, the problem can be resolved 29. _____ Wright A. Romanowsky-type
by 30. _____ May-Grünwald B. Non–Romanowsky-type
A. decreasing the angle of the pusher slide 31. _____ Giemsa
B. increasing the angle of the pusher slide 32. _____ Methylene blue
C. using a larger drop of blood 33. If a blood smear stains too red on microscopic exami-
D. pushing the slide slower in smearing out the nation of a Wright-stained preparation, possible causes
blood include that
24. The examination of bone marrow is useful in A. the staining time was too long
A. diagnosing a bleeding disorder B. the stain was too basic
B. diagnosing some disorders associated with erythro- C. the buffer was too acidic and the exposure time was
cytes and leukocytes too short
C. diagnosing acute leukemias D. the buffer was too basic and the exposure time was
D. both B and C too long

REFERENCE Dale JC. Phlebotomy Complications, Presented at Mayo Laboratory’s


Phlebotomy Conference, August 1996, Boston, MA.
1. Lippi G, Fostini R, Guidi GC. Quality improvement in laboratory Ernst D, Calam R. NCCLS simplifies the order of draw: A brief history,
medicine: Extra-analytical issues, Clin Lab Med, 28(2):285–294, Med Lab Obs, 36(5):26, 2004.
2008. Faber V. Phlebotomy and the aging patient, Adv Med Lab Prof,
29(1):24–25, 1998.
Foubister V. Quick on the draw—coagulation tube response, CAP
BIBLIOGRAPHY TODAY, 16(10):38–42, 2002.
Gerberding JL. Occupational exposure to HIV in health care settings,
Adcock DM, et al. Effect of 3.2% vs. 3.8% sodium citrate concentration N Engl J Med, 348(9):826–832, 2003.
on routine coagulation testing, Am J Clin Pathol, 107:105–110, 1997. Haraden L. Pediatric phlebotomy: Great expectations, Adv Med Lab
American Hospital Association: Patient Care Partnership, www.aha. Prof, 28(11):12–13, 1997.
org (retrieved August 2005). Hurley TR. Considerations for the Pediatric and Geriatric Patient,
Avinoso D. Clot activator tubes, Med Lab Obs, 34(9):35, 2002. Presented at Mayo Laboratory’s Phlebotomy Conference, August
BD Vacutainer Systems, LabNotes, 13(2):1–16, 2003. 1996, Boston, MA.
BD Vacutainer Systems, LabNotes, 13(3): 2003. Iverson LK. Changing Roles of Phlebotomist/Customer Satisfaction,
Bush V. Why doesn’t my heparinized plasma specimen remain antico- Presented at Mayo Laboratory’s Phlebotomy Conference, August
agulated? LabNotes, 13: 2003 (www.bd.com, retrieved July 2, 2003). 1996, Boston, MA.
Bush V, Cohen R. The evolution of evacuated blood collection tubes, Linke EG, Henry JB. Clinical pathology/laboratory medicine purposes
LabNotes, 19(1): 2009 retrieved August 20, 2009. and practice. In: Henry JB (ed.). Clinical Diagnosis and Management
Clark K. Phlebotomy: Beyond the basics, Adv Med Lab Prof, by Laboratory Methods, 18th ed, Philadelphia, PA: Saunders, 1991.
28(9):12–15, 1997. Reneke J, et al. Prolonged prothrombin time and activated partial
CLSI. Protection of Laboratory Workers from Infectious Disease Trans- thromboplastin time due to underfilled specimen tubes with 109
mitted by Blood, Body Fluids and Tissue, M29-A3. Wayne, PA: mmol/L (3.2%) citrate anticoagulant, Am J Clin Pathol, 109:754–
CLSI, 2005. 757, 1998.

Turgeon_Chap02.indd 48 10/13/2010 6:30:07 PM


70 PART 1 ■ The Principles of Hematology

REVIEW QUESTIONS

1. The smallest organized unit of living tissue is the 15. A cellular inclusion that represents a common storage
A. nucleus form of iron is
B. cell A. glycogen
C. organelle B. vacuoles
D. cytoplasm C. Auer body
2. The cell membrane’s major components are D. ferritin
A. carbohydrates and proteins 16. The nucleus of the cell contains
B. proteins and lipids A. chromatin, nucleoli, and nucleoplasm
C. lipids and glycoproteins B. chromatin, nucleoli, and ribosomes
D. polysaccharides and lipids C. DNA, RNA, and ribosomes
3. Which of the following is a characteristic of osmosis? D. DNA, RNA, and mitochondria
A. Requires energy (ATP) 17. The overall function of DNA is
B. Movement of water molecules A. protein and enzyme production
C. An unusual cellular activity B. control of cellular function and transmission of
D. Requires a carrier molecule genetic information
4. Which of the following is a characteristic of active trans- C. control of heterochromatin and euchromatin
port? synthesis
A. Requires energy (ATP) D. production of cellular energy and transmission of
B. Movement of molecules up the concentration genetic information
gradient 18. Heterochromatin is
C. Requires a carrier molecule A. genetically inactive
D. All of the above B. found in patches or clumps
5. Phagocytosis is C. genetically inactive and pale staining
A. a type of endocytosis D. Both A and B
B. the engulfment of fluid molecules 19. Chromosomal translocation is
C. the engulfment of particulate matter A. a frequent activity of homologous chromosomes in
D. Both A and C meiosis
B. a rearrangement of genetic material
Questions 6 through 9: Match the following organelles C. the process in which a segment of one chromosome
with their appropriate function. breaks away from its normal location
6. _____ Centrioles A. Protein production D. All of the above
7. _____ Rough ER B. Concentration of 20. A chromosomal deletion is
8. _____ Smooth ER secretory granules A. loss of a pair of chromosomes
9. _____ Golgi C. Lipid synthesis B. loss of a segment of chromosome
apparatus D. DNA synthesis C. attachment of a piece of a chromosome
E. Points of attachment of the D. an exchange of genetic material
spindle fibers
Questions 21 through 24: Match the following activities
Questions 10 through 13: Match the following organelles with the appropriate period of time. Use an answer only
with their appropriate function. once.
10. _____ Lysosomes A. Energy production and
21. _____ G1 A. DNA replication
11. _____ Microtubules heme synthesis
22. _____ S B. Protracted state of mitotic inactivity
12. _____ Mitochondria B. Protein synthesis
23. _____ G2 C. Immediately precedes actual mitotic
13. _____ Ribosomes C. Cytoskeleton
24. _____ G0 division
D. Intracellular digestion
D. Actual mitotic division
E. Carbohydrate synthesis
14. Glycogen is a E. An active period of protein synthe-
A. protein sis and cellular metabolism
B. lipid
C. carbohydrate
D. hormone

(continued)

Turgeon_Chap03.indd 70 10/15/2010 9:26:15 AM


CHAPTER 3 ■ Molecular Genetics and Cellular Morphology 71

REVIEW QUESTIONS (continued)

Questions 25 through 29: Match the following mitotic B. typically has three temperature steps
activities with the appropriate cellular activity. Use an C. repeats the number of cycles about 30
answer only once. D. all of the above
37. Variations of PCR include
25. _____ Prophase A. Chromosomes line up at the
A. nested primers
26. _____ Metaphase cell’s equator
B. real-time PCR
27. _____ Anaphase B. Two identical daughter cells
C. microarray analysis
28. _____ Telophase form
D. both A and B
29. _____ Cytokinesis C. Division of the cellular cyto-
38. The method considered to be the “gold standard” of
plasm
molecular methods is
D. Chromatids separate and
A. DNA sequencing
move to opposite ends of the
B. Southern blot
mitotic spindle
C. Northern blot
E. Chromosomes tightly coil
D. Dot blot
and condense
39. The Southern blot procedure has diagnostic applica-
30. In meiosis, the cells produced contain
tions for diseases or disorders associated with
A. a 2n number of chromosomes
A. significant changes in DNA (e.g., deletion)
B. 22 pairs of chromosomes
B. determination of clonality in lymphomas of T- or
C. 23 pairs of chromosomes
B-cell origin
D. 23 chromosomes
C. detection of restriction fragment length polymor-
31. Hematologists are interested in inherited disorders.
phisms
Which of the following are inherited disorders?
D. all of the above
A. Sickle cell trait
40. The Northern blot procedure can be used
B. Sickle cell anemia
A. to mass-produce erythropoietin
C. Hemophilia
B. for analysis of the proximal product of gene
D. All of the above
expression
32. Molecular techniques are being used to detect abnor-
C. for antenatal genetic counseling
malities of
D. all of the above
A. erythrocytes
41. All of the following are true of FISH except _____
B. leukocytes
A. The acronym stands for fluorescent in situ
C. some coagulation factors
hybridization.
D. All of the above
B. It is a tissue-based molecular diagnostic assay.
33. The first inherited hematologic disorder to be diagnosed
C. It is a prenatal diagnosis of a genetic disorder.
using molecular biologic assay was
D. It is useful in the diagnosis of various anemias.
A. hemophilia A
42. Microarrays are
B. factor V Leiden
A. DNA probes bonded on glass chips
C. sickle cell anemia
B. tissue-based probes
D. CML
C. used to identify single-base mutations
34. PCR testing is useful in
D. used to determine clonality in lymphomas
A. forensic testing
43. Molecular techniques provide a diagnostic tool to
B. genetic testing
A. detect MRD in hematological malignancies
C. disease diagnosis
B. monitor patients following bone marrow
D. All of the above
transplantation
35. The traditional PCR technique
C. detect an early relapse in a patient treated for a
A. extends the length of the genomic DNA
hematological malignancy
B. alters the original DNA nucleotide sequence
D. all of the above
C. amplifies low levels of specific DNA sequences
D. amplifies the target region of RNA
36. PCR protocol
A. doubles the specific amount of DNA with each
cycle

Turgeon_Chap03.indd 71 10/15/2010 9:26:15 AM


CHAPTER 4 ■ Hematopoiesis 87

REVIEW QUESTIONS

1. The normal sequence of blood cell development is 5. As a blood cell matures, the overall cell diameter in most
A. yolk sac—red bone marrow—liver and spleen cases
B. yolk sac—thymus—liver and spleen—red bone A. increases
marrow B. decreases
C. yolk sac—liver and spleen—red bone marrow C. remains the same
D. liver and spleen—yolk sac—red bone marrow 6. As a blood cell matures, the ratio of nucleus to cyto-
2. The maturational sequence of the thrombocyte (plate- plasm (N:C) in most cases
let) is A. increases
A. megakaryoblast—promegakaryocyte—megakaryo- B. decreases
cyte—metamegakaryocyte—thrombocyte C. remains the same
B. promegakaryocyte—megakaryocyte—metamega- 7. The chromatin pattern, in most cells, as the cell
karyocyte—thrombocyte matures
C. megakaryoblast—promegakaryocyte—megakaryo- A. becomes more clumped
cyte—thrombocyte B. becomes less clumped
D. megakaryoblast—promegakaryocyte—metamega- C. remains the same
karyocyte—thrombocyte 8. The presence of nucleoli is associated with
3. The maturational sequence(s) of the erythrocyte is A. immature cells
(are) B. all young cells, except myeloblasts
A. rubriblast—prorubricyte—rubricyte—metarubri- C. only erythroblasts
cyte—reticulocyte—mature erythrocyte D. disintegrating cells
B. prorubricyte—rubricyte—metarubricyte—reticulo- 9. In the blast stage of development of leukocytes, the cyto-
cyte—mature erythrocyte plasm of the cell is
C. pronormoblast—basophilic normoblast—poly- A. dark blue and lacks vacuoles
chromatophilic normoblast—orthochromic nor- B. light blue and lacks granules
moblast—reticulocyte—mature erythrocyte C. light blue and has specific granules
D. both A and C D. gray with many dark-blue granules
4. The cell maturation sequence of the segmented neutro-
phil is Questions 10 through 14: Match the cellular characteris-
A. promyelocyte—myeloblast—myelocyte—metamy- tics with the name of the appropriate mature leukocyte.
elocyte—band or stab—segmented neutrophil Use an answer only once.
(PMN) 10. _____ Segmented A. Large orange granules
B. myeloblast—promyelocyte—myelocyte—metamy- neutrophil B. An elongated and curved
elocyte—band or stab—segmented neutrophil 11. _____ Monocyte nucleus
(PMN) 12. _____ Lymphocyte C. Light, sky-blue cytoplasm
C. monoblast—promyelocyte—myelocyte—metamy- 13. _____ Band form D. Kidney bean–shaped nucleus
elocyte—band or stab—segmented neutrophil neutrophil E. Averages approximately 56% of
(PMN) 14. _____ Eosinophil normal adult leukocytes in the
D. promyelocyte—myelocyte—metamyelocyte—band peripheral blood
or stab—segmented neutrophil (PMN)

BIBLIOGRAPHY Durand C, Dzierzak E. Embryonic beginnings of adult hematopoietic


stem cells. Haematologica, 90(1):100–108, 2005.
Becerra SP, Amaral J. Erythropoietin—an endogenous retinal survival Dzierzak E. Ontogenic emergence of definite hematopoietic stem cells,
factor, N Engl J Med, 347(24):1968–1970, 2002. Curr Opin Hematol, 10(3):229–234, 2003.
Blank U, Karlsson G, Karlsson S. Signaling pathways governing stem- Fleming MD, Kutok JL, Skarin AT. Examination of the bone marrow.
cell fate, Blood, 111(2):492–500, 2008. In: Handin RI, Lux SE, Stossel TP (eds.). Blood, 2nd ed, Philadel-
Bunn HF. New agents that stimulate erythropoiesis, Blood, 109(3), phia, PA: Lippincott Williams & Wilkins, 2003:59–79.
868–873, 2007. Golde DW. Hematopoietic growth factors, Int J Cell Cloning, 8(Suppl 1):
Chasis JA, Mohandas N. Erythroblastic islands: niches for erythropoi- 4–10, 1990.
esis. Blood, 112(3):470–476, 2008. Groopman JE, Molina J, Scadden DT. Hematopoietic growth factors,
De la Fuente J, et al. Alpha2beta1 and Alpha4beta1 integrins mediate N Engl J Med, 321(21):1449–1459, 1989.
the homing of mesenchymal stem/progenitor cells during fetal life, Hauke RJ. Hematopoietic growth factors, Lab Med, 31(11):613–615,
Hematol J, 4(Suppl 2):13, 2003. 2000.

Turgeon_Chap04.indd 87 10/13/2010 6:43:36 PM


CHAPTER 5 ■ Erythrocyte Maturation, Physiology, and Lifecycle 121

REVIEW QUESTIONS

1. The progression of erythropoiesis from prenatal life to 9. In a Wright-stained peripheral blood film, the reticulo-
adulthood is cyte will have a blue appearance. This is referred to as
A. yolk sac—red bone marrow—liver and spleen A. megaloblastic maturation
B. yolk sac—liver and spleen—red bone marrow B. bluemia
C. red bone marrow—liver and spleen—yolk sac C. polychromatophilia
D. liver and spleen—yolk sac—red bone marrow D. erythroblastosis
2. Which of the following is (are) characteristic(s) of eryth- 10. In the reticulocyte stage of erythrocytic development,
ropoietin? A. nuclear chromatin becomes more condensed
A. Glycoprotein B. RNA is catabolized and ribosomes disintegrate
B. Secreted by the liver C. full hemoglobinization of the cell occurs
C. Secreted by the kidneys D. both B and C
D. All of the above 11. On a Wright-stained peripheral blood smear, stress or
3. Which of the following is a characteristic of erythropoietin? shift reticulocytes are
A. Produced primarily in the liver of the unborn A. smaller than normal reticulocytes
B. Gene for erythropoietin is found on chromosome 11 B. about the same size as normal reticulocytes
C. Most erythropoietin is secreted by the liver in C. larger than normal reticulocytes
adults D. noticeable because of a decreased blue tint
D. Cannot cross the placental barrier 12. The normal range for reticulocytes in adults is
4. Stimulation of erythropoietin is caused by A. 0% to 0.5%
A. tissue hypoxia B. 0.5% to 1.0%
B. hypervolemia C. 0.5% to 2.0%
C. inflammation D. 1.5% to 2.5%
D. infection 13. If a male patient has a reticulocyte count of 5.0% and
5. The maturational sequences of an erythrocyte are a packed cell volume of 0.45 L/L, what is his corrected
A. rubriblast—prorubricyte—metarubricyte— reticulocyte count?
rubricyte-reticulocyte A. 2.5%
B. rubriblast—prorubricyte—rubricyte—metarubri- B. 4.5%
cyte-reticulocyte C. 5.0%
C. pronormoblast—basophilic normoblast—polychro- D. 10%
matic normoblast—orthochromatic normoblast— 14. If a male patient has a reticulocyte count of 6.0% and a
reticulocyte packed cell volume of 45%, what is his RPI?
D. both B and C A. 1.5
6. What is the immature erythrocyte found in the bone B. 3.0
marrow with the following characteristics: 12 to 17 mm C. 4.5
in diameter, N:C of 4:1, nucleoli not usually apparent, D. 6.0
and basophilic cytoplasm? 15. Normal adult hemoglobin has
A. Rubriblast (pronormoblast) A. two alpha and two delta chains
B. Reticulocyte B. three alpha and one beta chains
C. Metarubricyte (orthochromatic normoblast) C. two alpha and two beta chains
D. Prorubricyte (basophilic normoblast) D. two beta and two epsilon chains
7. The nucleated erythrocyte with a reddish pink cyto- 16. The number of heme groups in a hemoglobin molecule
plasm and condensed chromatin pattern is a is
A. rubricyte (polychromatic normoblast) A. 1
B. basophilic normoblast (prorubricyte) B. 2
C. metarubricyte (orthochromatic normoblast) C. 3
D. either B or C D. 4
8. With a normal diet, an erythrocyte remains in the retic- 17. Increased amounts of 2,3-DPG _____ the oxygen affin-
ulocyte stage in the circulating blood for ity of the hemoglobin molecule.
A. 1 day A. increases
B. 2.5 days B. decreases
C. 3 days C. does not alter
D. 120 days

(continued)

Turgeon_Chap05.indd 121 10/14/2010 12:46:38 PM


122 PART 2 ■ Erythrocytes

REVIEW QUESTIONS (continued)

18. After a molecule of hemoglobin gains the first two Questions 27 and 28: If globin synthesis is insufficient in
oxygen molecules, the molecule a person, iron accumulates in the cell’s (27) _____ as (28)
A. expels 2,3-DPG _____ aggregates.
B. has decreased oxygen affinity 27. _____
C. becomes saturated with oxygen A. nucleus
D. adds a molecule of oxygen to an alpha chain B. cytoplasm
19. If normal adult (A1) and fetal hemoglobin F are com- C. Golgi apparatus
pared, fetal hemoglobin has _____ affinity for oxygen. D. mitochondria
A. less 28. _____
B. the same A. transferrin
C. a greater B. ferritin
20. Oxyhemoglobin is a _____ than deoxyhemoglobin. C. albumin
A. weaker acid D. iron
B. stronger acid 29. Increased erythropoietin production in secondary poly-
21. Heme is synthesized predominantly in the cythemia can be caused by
A. liver A. chronic lung disease
B. red bone marrow B. smoking
C. mature erythrocytes C. renal neoplasms
D. both A and B D. all of the above
30. Relative polycythemia exists when
Questions 22 and 23: The initial condensation reaction in A. increased erythropoietin is produced
the synthesis of porphyrin preceding heme formation takes B. the total blood volume is expanded
place in the (22) _____ and requires (23) _____. C. the plasma volume is increased
22. _____ D. the plasma volume is decreased
A. liver 31. Which of the following is (are) characteristic(s) of meg-
B. spleen aloblastic maturation?
C. red bone marrow A. Cells of some leukocytic cell lines are smaller than
D. mitochondria normal
23. _____ B. Nuclear maturation lags behind cytoplasmic maturation
A. iron C. Cytoplasmic maturation lags behind nuclear matu-
B. vitamin B6 ration
C. vitamin B12 D. Erythrocytes are smaller than normal
D. vitamin D
24. The final steps in heme synthesis, including the forma- Questions 32 and 33: When porphyrin synthesis is impaired,
tion of protoporphyrin, take place in the (32) _____ become encrusted with (33) _____.
A. a cell’s nucleus 32. _____
B. a cell’s cytoplasm A. lysosomes
C. the spleen B. nucleoli
D. the mitochondria C. mitochondria
25. An acquired disorder of heme synthesis is D. vacuoles
A. congenital erythropoietic porphyria 33. _____
B. lead poisoning A. protoporphyrin
C. hemolytic anemia B. hemoglobin
D. hemoglobinopathy C. iron
26. The protein responsible for the transport of iron in D. delta-aminolevulinic acid
hemoglobin synthesis is 34. Which of the following hemoglobin types is the major
A. globin type present in a normal adult?
B. transferrin A. A
C. oxyhemoglobin B. S
D. ferritin C. A2
D. Bart

(continued)

Turgeon_Chap05.indd 122 10/14/2010 12:46:38 PM


CHAPTER 5 ■ Erythrocyte Maturation, Physiology, and Lifecycle 123

REVIEW QUESTIONS (continued)

35. The alkaline denaturation test detects the presence of 46. The Embden-Meyerhof pathway net gain of ATP
hemoglobin provides high energy phosphates to
A. A1C A. maintain membrane lipids
B. F B. power the cation pump needed for the sodium-
C. C potassium concentration pump and calcium flux
D. S C. preserve the shape and flexibility of the cellular
membrane
Questions 36 through 39: Match the following hemoglobin D. all of the above
types. 47. The end product of the Embden-Meyerhof pathway of
36. _____ A glucose metabolism in the erythrocyte is
37. _____ A2 A. pyruvate
38. _____ F B. lactate
39. _____ Embryonic C. glucose-6-phosphate
A. Two alpha and two delta chains D. the trioses
B. Zeta chains and either epsilon or gamma chains 48. The net gain in ATPs in the Embden-Meyerhof glycolytic
C. Two alpha and two beta chains pathway is
D. Two alpha and two gamma chains A. 1
40. Fetal hemoglobin (hemoglobin F) persists until B. 2
A. a few days after birth C. 4
B. a few weeks after birth D. 6
C. several months after birth 49. The most common erythrocytic enzyme deficiency
D. adulthood involving the Embden-Meyerhof glycolytic pathway is
41. Cellulose acetate at pH 8.6 separates the hemoglobin a deficiency of
fractions A. ATPase
A. S B. pyruvate kinase
B. H C. glucose-6-phosphate dehydrogenase
C. A D. lactic dehydrogenase
D. both A and C 50. If a defect in the oxidative pathway (hexose monophos-
42. If an alkaline (pH 8.6) electrophoresis is performed, phate shunt) occurs, what will result?
hemoglobin E has the same mobility as hemoglobin A. Insufficient amounts of reduced glutathione
A. S B. Denaturation of globin
B. F C. Precipitation of Heinz bodies
C. A D. All of the above
D. C 51. The function of the methemoglobin reductase pathway
43. The limited metabolic ability of erythrocytes is owing is to
to A. prevent oxidation of heme iron
A. the absence of RNA B. produce methemoglobinemia
B. the absence of ribosomes C. provide cellular energy
C. no mitochondria for oxidative metabolism D. control the rate of glycolysis
D. the absence of DNA 52. The Luebering-Rapoport pathway
44. Which of the following statements is (are) true of the A. permits the accumulation of 2,3-DPG
erythrocytic cytoplasmic contents? B. promotes glycolysis
A. High in potassium ion C. produces cellular energy
B. High in sodium ion D. produces acidosis
C. Contain glucose and enzymes necessary for 53. In conditions of acidosis,
glycolysis A. erythrocytic glycolysis is reduced
D. Both A and C B. available oxygen is increased
45. The Embden-Meyerhof glycolytic pathway uses _____ C. DPG levels fall to a level sufficient to normalize oxy-
% of the erythrocyte’s total glucose. gen tension
A. 10 D. all of the above
B. 20
C. 50
D. 90

(continued)

Turgeon_Chap05.indd 123 10/14/2010 12:46:38 PM


CHAPTER 6 ■ Erythrocyte Morphology and Inclusions 143

CASE STUDY (continued)

3. Hemoglobin 13.3 g/dL ■ Discussion


4. WBC 6.55 × 109/L 1. The patient has a decreased hemoglobin, hematocrit, and
5. Platelets 450 × 109/L RBC count. The follow-up PCR and immunofluorescent
assay were positive for B. microti.
■ Follow-Up
2. The small red bite on the patient’s leg could have been
Because of the patient’s symptoms and area of summer res-
from an infected tick. Nantucket Island is one of the
idence, his physician ordered a PCR and immunofluores-
areas in which tickborne diseases (e.g., Lyme disease,
cent assay titers to screen out babesiosis and ehrlichiosis.
ehrlichiosis, and babesiosis) are endemic.
■ Questions 3. Yes. Babesiosis poses the greatest risk of fatality to indi-
1. What do the laboratory results suggest? viduals older than age 50, asplenic individuals, and
2. What evidence is there in the patient’s history to suggest immunocompromised individuals as a result of immu-
babesiosis? nosuppressive drugs, malignancy, or human immuno-
3. Can babesiosis be fatal? deficiency virus infection.
DIAGNOSIS: Babesiosis

REVIEW QUESTIONS

1. The average diameter of a normal erythrocyte is _____ 12. _____ Schistocytes


mm. 13. _____ Spherocytes
A. 5.2 A. Short, scalloped, or spike-like projections that are
B. 6.4 regularly distributed around the cell
C. 7.2 B. Fragments of erythrocytes
D. 8.4 C. The scooped-out part of an erythrocyte that remains
after a blister cell ruptures
Questions 2 through 5: Match the following terms with D. Compact round shape
the appropriate description.
2. _____ Macrocytic Questions 14 through 17: Match the condition with the
3. _____ Microcytic predominant erythrocyte type seen on a peripheral blood
4. _____ Anisocytosis smear (use an answer only once).
5. _____ Poikilocytosis 14. _____ Associated with a defect in nuclear
A. Variation in erythrocyte size maturation
B. Larger than normal 15. _____ Associated with a decrease in hemoglobin
C. Smaller than normal synthesis
D. Variation in erythrocyte shape 16. _____ Represents an imbalance between erythrocytic
and plasma lipids
Questions 6 through 9: Match the common terms for 17. _____ Results from the gelation of polymerized
erythrocytes with the equivalent nomenclature. deoxygenated Hb S
6. _____ Normal erythrocyte A. Microcytes
7. _____ Oval macrocyte B. Sickle cells
8. _____ Target cell C. Macrocytes
9. _____ Sickle cell D. Acanthocytes
A. Megalocyte 18. Polychromatophilia is
B. Drepanocyte A. a blue-colored erythrocyte when stained with Wright
C. Codocyte stain
D. Discocyte B. caused by diffusely distributed RNA in the
Questions 10 through 13: Match the terms for erythro- cytoplasm
cytes with the appropriate morphological description. C. equivalent to a reticulocyte when stained with a
10. _____ Echinocytes supravital stain
11. _____ Helmet cells D. all of the above

(continued)

Turgeon_Chap06.indd 143 10/13/2010 6:46:40 PM


144 PART 2 ■ Erythrocytes

REVIEW QUESTIONS (continued)

Questions 19 through 22: Match the following erythro- Questions 32 through 35: Match the following erythro-
cytic inclusions with the appropriate description. cyte inclusions with the appropriate clinical condition or
19. _____ Basophilic stippling disorder.
20. _____ Howell-Jolly bodies 32. _____ Basophilic stippling
21. _____ Pappenheimer bodies 33. _____ Howell-Jolly bodies
22. _____ Heinz bodies 34. _____ Heinz bodies
A. DNA 35. _____ Pappenheimer bodies
B. Precipitated denatured hemoglobin A. Pernicious anemia
C. Granules composed of ribosomes and RNA B. G6PD deficiency
D. Aggregates of iron, mitochondria, and ribosomes C. Iron loading anemia
23. Which of the following is the term for erythrocytes D. Lead poisoning
resembling a stack of coins on thin sections of a periph-
eral blood smear? Questions 36 through 39: Match the appropriate species of
A. Anisocytosis malaria with one of the following characteristics.
B. Poikilocytosis 36. _____ Plasmodium vivax
C. Agglutination 37. _____ Plasmodium falciparum
D. Rouleaux formation 38. _____ Plasmodium malariae
39. _____ Plasmodium ovale
Questions 24 through 27: Match the following erythrocyte A. The schizont contains 6 to 12 merozoites; gener-
morphology with the appropriate clinical condition or ally abundant in hematin granules; may contain Zi-
disorder. emann stippling.
24. _____ Macrocytes B. The most predominant species worldwide; 12 to 24
25. _____ Microcytes merozoites; may contain Schüffner dots or granules.
26. _____ Acanthocytes C. Infected erythrocytes may be enlarged and oval
27. _____ Echinocytes shaped; may contain Schüffner dots; 6 to 14 mero-
A. Iron deficiency anemia zoites in the schizont.
B. Abetalipoproteinemia D. Young trophozoites and gametocytes are generally
C. Pernicious anemia the only stage seen in peripheral blood; gametocytes
D. No related disease state appear as crescent- or sausage-shaped structures in
Questions 28 through 31: Match the predominant eryth- erythrocytes; Maurer dots may be present.
rocyte morphology with the appropriate clinical condition 40. Nantucket Island is an endemic area for
or disorder. A. P. vivax
28. _____ Leptocytes B. P. falciparum
29. _____ Microspherocytes C. Babesiosis
30. _____ Codocytes D. all of the above
31. _____ Dacryocytes 41. Babesiosis infection shares many of the same symptoms as
A. Hepatic disorders A. Plasmodium falciparum malaria
B. Hemolytic disease of the fetus and newborn B. Lyme disease
C. Hemoglobinopathies C. Ehrlichia
D. Pernicious anemia D. None of the above

BIBLIOGRAPHY Kretchman DM, Rogers BS. Erythrocyte shape transformation


associated with calcium accumulation, Am J Med Technol,
Cserti CM, Dzik WH. The ABO blood group system and Plasmodium 47(7):561–565, 1981.
falciparum malaria, Blood, 110(7):2250–2258, 2007. Lehman D. Malaria and other bloodborne parasites, Adv Med Lab Prof,
Cunha BA, Barnett B. Babesiosis, eMedicine (www.emedicine.com), 28(10):19–21, 1997.
retrieved October 9, 2003. Mariani M, et al. Clinical and hematologic features of 300 patients affected
Glenister FK, et al. Functional alteration of red blood cells by a by hereditary spherocytosis grouped according to the type of the
megadalton protein of Plasmodium falciparum, Blood, 113(4):919– membrane protein defect, Haematologica 93(9), 1310–1316, 2008.
928, 2009. Rosenthal PJ. Hydrolysis of erythrocyte proteins by protease of malaria
Haun DE, Leach A. Improving RBC morphology assessments, Adv parasites, Curr Opin Hematol, 9(2):140–145, 2002.
Med Lab Prof, 13(24):21–23, 2001. Winter G, Wahlgren M. Severe anemia in malaria: defense gone
wrong?, Blood 106(10):3337–3338, 2005.

Turgeon_Chap06.indd 144 10/13/2010 6:46:40 PM


148 PART 2 ■ Erythrocytes

Malarial smears
7.1 Platelet count
TABLE Grading of Erythrocyte Morphology
Reticulocyte count
Sickle cell testing
Numerical Scale Description Glucose-6-phosphate dehydrogenase (G6PD) assay
Hemoglobin electrophoresis
0 Normal appearance or slight
variation in erythrocytes. Additional procedures may be of assistance in identifying spe-
cific types of anemias but are usually performed in other sec-
1+ Only a small population of
tions of the clinical laboratory. These procedures include
erythrocytes displays a particular
abnormality; the terms slightly Antibody screening and identification tests
increased or few would be Direct antiglobulin (AHG) test
comparable. Measurements of bilirubin levels
2+ More than occasional numbers of Folic acid assay
abnormal erythrocytes can be seen Measurement of haptoglobin level
in a microscopic field; an equivalent Lactic dehydrogenase (LDH) determination
descriptive term is moderately Serum iron and total iron-binding capacity (TIBC)
increased. Vitamin B12 assay
Occult blood testing
3+ Severe increase in abnormal
Urobilinogen screening
erythrocytes in each microscopic
field; an equivalent descriptive term
is many. CHAPTER HIGHLIGHTS
4+ The most severe state of
erythrocytic abnormality, with the Anemia is considered to be present if the hemoglobin con-
abnormality prevalent throughout centration of the RBCs or the packed cell volume of RBCs
each microscopic field; comparable (hematocrit) is below the lower limit of the 95% reference
terms are marked or marked interval for the individual’s age, gender, and geographi-
increase. cal location. The causes of anemia fall into three major
pathophysiological categories: blood loss, impaired red cell
production, and accelerated red cell destruction. Anemia
may be a sign of an underlying disorder.
from one laboratory to another but will generally conform The clinical signs and symptoms of anemia can result
to the scale as presented in Table 7.1. from diminished delivery of oxygen to the tissues. Signs and
symptoms of anemia are related to the lowered hemoglobin
Supplementary Assessment of Anemias concentration. The usual complaints of an anemic patient
are easy fatigability and dyspnea on exertion. Many differ-
Other procedures that support the diagnostic process of ent types of anemias exist, with many causes and manifes-
identifying an anemia may be needed. Some of these assays tations. Classifications group anemias based on erythrocyte
are performed in the hematology laboratory and others may morphology, physiology, or probable etiology. The method
be performed in another section of the clinical laboratory. based on red cell morphology categorizes anemias by the size
A bone marrow examination (see Chapter 26) may be per- of the erythrocytes. Anemia also may be classified by red cell
formed and may reveal an abnormal ratio of leukocytes to morphology as macrocytic, normocytic, or microcytic.
erythrocytes, the myeloid-erythroid (M:E) ratio. Other mat- The laboratory investigation of anemias involves the
urational irregularities or unusual cellular elements may be quantitative and semiquantitative measurements of erythro-
observed. The following procedures (see Chapter 26 for addi- cytes and supplementary testing of blood and body fluids.
tional details) may provide supplementary information: The results of these analyses provide the foundation for both
Fetal hemoglobin (Hb F) concentration the diagnosis and the treatment of anemia.

REVIEW QUESTIONS

1. The causes of anemia include 2. The clinical signs and symptoms of anemia can result from
A. blood loss A. diminished delivery of oxygen to the tissues
B. impaired red cell production B. lowered hemoglobin concentration
C. accelerated red cell destruction C. increased blood volume
D. all of the above. D. both A and B

(continued)

Turgeon_Chap07.indd 148 10/15/2010 9:31:18 AM


CHAPTER 7 ■ Classification and Laboratory Assessment of Anemias 149

REVIEW QUESTIONS (continued)

3. Which of the following is a significant laboratory finding A.1+


in anemia? B.2+
A. Decreased hemoglobin C.3+
B. Increased packed cell volume D.4+
C. Increased erythrocyte count 5. Anemias can be categorized into
D. Normal erythrocyte indices A. hemolytic types
4. If you are grading changes in erythrocytic size or shape B. blood loss types
using a scale of 0 to 4+ and many erythrocytes deviate from C. impaired production types
normal per microscopic field, the typical score would be D. all of the above

BIBLIOGRAPHY Soloway HB, Peter JB. Interpretation of tests, Diagn Med, 8(4):10–11,
1985.
Kahn F, et al. Guide to Diagnostic Testing, Philadelphia, PA: Lippincott Wallach J. Handbook of Interpretation of Diagnostic Tests, 8th ed, Phila-
Williams & Wilkins, 2002. delphia, PA: Lippincott Williams & Wilkins, 2007.
Pansky B. Dynamic Anatomy and Physiology, New York, NY: Mac-
millan, 1975.

Turgeon_Chap07.indd 149 10/15/2010 9:31:18 AM


CHAPTER 8 ■ Acute and Chronic Blood Loss Anemias 151

TABLE 8.1 Clinical Features of Acute Hemorrhage in Healthy Young Adults

Volume of Blood Loss (mL) Blood Volume (%) Symptoms


500–1,000 10–20 Few or none
1,000–1,500 20–30 At rest-(recumbent) asymptomatic
Upright position-light-headedness, and hypotension,
tachycardia
1,500–2,000 30–40 Symptomatic (recumbent)-thirst, shortness of breath,
clouding or loss of consciousness; blood pressure, cardiac
output, venous pressure decrease; pulse usually becomes
rapid; extremities become cold, clammy, an pale.
2,000–2,500 40–50 Lactic acidosis, shock; irreversible shock, death
Source : Greer JP, et al. Wintrobe’s Clinical Hematology, 11 ed, Philadelphia, PA: Lippincott Williams & Wilkins, 2004:975.

iron deficiency, and the newly formed cells are morphologically


TABLE 8.2 Blood Loss Anemia hypochromic and microcytic. The WBC count is normal or
slightly decreased. Platelets are commonly increased, and only
Chronic later, in severe iron deficiency, are they likely to be decreased.
Acute (24 hours) (months)
Etiology Trauma GI tract
Menstruation CHAPTER HIGHLIGHTS
Urinary tract Acute blood loss is usually associated with traumatic condi-
Blood volume Yes No tions during or after surgery. It does not produce an imme-
disruption diate anemia. A severe hemorrhage reduces an individual’s
Iron deficiency No Yes total blood volume and can be fatal because of the collapse
Hematocrit (packed Usually normal Decreased
of the circulatory system.
cell volume)
In acute blood loss, the body attempts to adjust to the sit-
uation by expanding the circulatory volume, which produces
WBC count Increased Normal the subsequent anemia. Fluid from the extravascular spaces
Platelets Increased Normal enters the blood circulation and has a diluting effect on the
Reticulocytes Normal Increased remaining cells.
The earliest hematological change in acute blood loss is a
transient fall in the platelet count, which may rise to elevated
levels within 1 hour. The next change is the development of
If blood is lost in small amounts over an extended period, neutrophilic leukocytosis with a shift to the left. The hemo-
both the clinical and hematological features seen in acute globin and hematocrit do not fall immediately. It can be 48
bleeding are absent. Regeneration of RBCs occurs at a slower or 72 hours after the hemorrhage until the full extent of the
rate. The reticulocyte count may be normal or only slightly red cell loss is apparent.
increased. Chronic blood loss lacks both the clinical and hematological
A noticeable anemia does not usually develop until after features seen in acute bleeding. A noticeable anemia does not
storage iron is depleted. At first, the anemia is normochromic usually develop until after storage iron is depleted. In time, the
and normocytic. Gradually, the chronic bleeding results in an peripheral blood smear becomes hypochromic and microcytic.

CASE STUDIES

CASE 8.1 abdominal injuries sustained in an automobile accident.


A 38-year-old white woman was treated in the emer- She was admitted to the hospital for observation and
gency department for severe lacerations and possible further evaluation. On admission, a complete blood
(continued)

Turgeon_Chap08.indd 151 10/13/2010 6:48:28 PM


152 PART 2 ■ Erythrocytes

CASE STUDIES (continued)

count (CBC), urinalysis, and radiograph series were 3. Acute blood loss immediately begins to stimulate a
ordered. healthy, normal bone marrow. Reticulocytosis becomes
apparent within 24 hours and peaks at 7 to 10 days after
■ Laboratory Data
severe blood loss. When erythrocyte restoration is com-
Her CBC results were as follows:
pleted, reticulocytosis ceases. Reticulocytes are seen as
Hemoglobin 10.5 g/dL polychromasia or polychromatophilic RBCs when the
Hct 34% RBCs are stained with Wright stain. A supervital stain
RBC 3.8 × 1012/L such as new methylene blue needs to be used to visibly
WBC 12.0 × 109/L demonstrate reticulocytes in erythrocytes.
The RBC indices were as follows:
MCV 89.6 fL DIAGNOSIS: Acute Blood Loss
MCH 27.6 pg
MCHC 31 g/dL CASE 8.2
The peripheral blood smear showed essentially normal A 55-year-old white male college professor had been
RBC morphology and platelet distribution. Forty-eight experiencing fatigue and shortness of breath when walk-
hours after admission, a stat repeat CBC was ordered. The ing over the past several months. Getting more sleep at
results were as follows: night did not help. He reported eating a balanced diet of
fruits, vegetables, meat, and dairy products. Upon physi-
Hemoglobin 8.0 g/dL cal examination, he appeared slightly pale but had no other
Hct 26% abnormalities. His primary care physician ordered a CBC,
RBC 2.9 × 1012/L urinalysis, and fecal occult blood (×3) tests.
WBC 15.5 × 109/L
■ Laboratory Data
The RBC indices were all within their normal ranges.
A peripheral blood smear showed normal RBC morphol- Laboratory findings were as follows:
ogy, although some polychromatophilia was noted. The Hemoglobin 12.5 g/dL
distribution of platelets had increased. A follow-up platelet Hematocrit 32%
count was 0.60 × 1012/L. RBC 4.2 × 1012/L
Subsequently, the patient was typed and cross-matched WBC count within normal limits
for 6 units of blood. Two units of whole blood cells were RBC indices:
administered immediately. An emergency laparotomy MCV 42 fL
revealed that the patient had injuries to both the liver and MCH 29.7 pg
spleen. Urinalysis: normal findings
Fecal occult blood (×3) positive
■ Questions
1. Why was the patient’s hemoglobin and hematocrit nor- ■ Questions
mal on admission but decreased after 48 hours? 1. What is the most likely cause of this patient’s anemia?
2. What is the significance of this patient’s increased leuko- 2. What type of red cell morphology would be expected on
cyte (WBC) and thrombocyte (platelet) count? a peripheral blood smear?
3. What is the reason for the polychromatophilia noted on 3. What follow-up tests should be conducted?
the 48-hour peripheral blood film?
■ Discussion
■ Discussion
1. The most likely cause of this patient’s anemia is chronic
1. The body adjusts to severe hemorrhaging by expanding blood loss. The source of the bleeding could be the GI
the circulating volume at the expense of the extravascular tract, in view of the fact that he had a positive test result
fluid. This volume adjustment produces a delayed ane- for fecal blood.
mia. As the extravascular fluid enters the bloodstream, 2. Hypochromic, microcytic RBCs would be expected on
it dilutes the RBCs. The hemoglobin and hematocrit his peripheral blood smear.
become decreased after 48 to 72 hours in such cases. 3. The source of the fecal blood needs to be located. In
2. In acute blood loss, the platelets and circulating granulo- this case, the patient had a follow-up colonoscopy that
cytes increase within a few hours. Immature WBCs may revealed a number of nonmalignant, bleeding polyps.
also be seen. Increased leukocytes and platelets are a nor-
mal body response to stress. The body continually strives DIAGNOSIS: Chronic Blood Loss
to maintain homeostasis.

Turgeon_Chap08.indd 152 10/13/2010 6:48:28 PM


CHAPTER 8 ■ Acute and Chronic Blood Loss Anemias 153

REVIEW QUESTIONS

Questions 1 through 5: Match the following characteristics 6. The erythrocyte morphology associated with anemia in
with either A or B. an otherwise healthy individual caused by acute blood
1. _____ Disorders of the GI system or heavy men- loss is usually
struation A. microcytic
2. _____ Increased thrombocytes (platelets) B. megaloblastic
3. _____ Traumatic conditions C. normochromic
4. _____ Does not disrupt the blood volume D. hypochromic
5. _____ Results in an iron deficiency and a hypochromic/ 7. Anemia caused by chronic blood loss is characterized by
microcytic erythrocyte morphology on a peripheral A. hypochromic, microcytic erythrocytes
blood smear B. decreased packed cell volume
A. Acute blood loss C. increased platelets
B. Chronic blood loss D. both A and B

BIBLIOGRAPHY
Greer JP, et al. Wintrobe’s Clinical Hematology, 11 ed, Philadelphia, Handin RI, Lux SE, Stossel TP. Blood, 2 ed, Philadelpha, PA: Lippin-
PA: Lippincott Williams & Wilkins, 2004. cott Williams & Wilkins, 2003.

Turgeon_Chap08.indd 153 10/13/2010 6:48:28 PM


CHAPTER 9 ■ Aplastic and Related Anemias 165

REVIEW QUESTIONS

1. Acquired aplastic anemia may be caused by 5. _____ Fanconi anemia


A. benzene or benzene derivatives 6. _____ Familial aplastic anemia
B. ionizing radiation and vitamin B12 7. _____ Pure red cell anemia
C. purine or pyrimidine analogues 8. _____ Diamond-Blackfan syndrome
D. all of the above A. A subset of Fanconi anemia
2. The sudden appearance of aplastic anemia or pure red B. A rare congenital form of red cell aplasia
cell aplasia is often caused by C. Is characterized by selective failure of red blood cell
A. a hemolytic process production
B. an immune process D. The best-described congenital form of aplastic
C. acute leukemias anemia
D. chronic leukemias 9. Hematopoietic cell targets in aplastic anemia are
3. Aplastic anemia can occur years before a diagnosis of affected by
_____ is made. A. activated cytotoxic T lymphocytes
A. paroxysmal nocturnal hemoglobinuria B. activation of the Fas receptor
B. myelodysplasia C. direct cell–cell interactions between lymphocytes
C. acute myelogenous leukemia and target cells
D. all of the above D. all of the above
4. If a patient with aplastic anemia is referred to as exhibit- 10. Fanconi anemia is associated with abnormal genes
ing pancytopenia, which cell lines are affected? located on chromosomes __q___, __q___.
A. Erythrocytes A. 9, 20
B. Leukocytes B. 5, 22
C. Thrombocytes C. 9, 12
D. All of the above D. 8, 23

Questions 5 to 8: Match the following. (Use an answer


only once)

BIBLIOGRAPHY Montanaro M, et al. Pure red cell aplasia, thrombotic thrombocy-


topenic purpura and rapidly fulminating myocarditis in a patient
Ayari AS, et al. Prevalence, severity and outcome of 20 children presenting acute B19 human parvovirus infection: a case report, J
and adolescents with Fanconi anemia, J Hematol, 4(Suppl 2):48, Hematol, 4(Suppl 2):47, 2003.
2003. Narla A, Ebert BL. Ribosomopathies: human disorders of ribosome
Bennett CL, et al. Pure red cell aplasia and epoetin therapy. NEJM, function, Blood, 115(16):3196–3205, 2010.
351(14):1403–1408, 2004. Petrov L, et al. Immunosuppressive therapy in severe and very severe
Calado RT. Telomeres and marrow failure. Am Soc Hematol, aplastic anemia, J Hematol, 4(Suppl 2):45–46, 2003.
Educational program, Vol. 1, Annual issue, 338–342, 2009. Ray MA, et al. Enhanced alternative splicing of the FLVCR1 gene
Choesmel V, et al. Impaired ribosome biogenesis in Diamond-Black- in Diamond Blackfan anemia disrupts FLVCR1 expression and
fan anemia, Blood, 109(3):1275–1292, 2007. function that are critical for enythropoiesis. Haematologica,
D’Andrea AD. Susceptibility pathways in Fanconi’s anemia and breast 93(11):1617–1626, 2008.
cancer, NEJM, 362(20):1909–1919, 2010. Seiwerth RS, et al. Transplantation of CD 34-enriched peripheral blood
Dianzani I, Loreni F. Daimond-Blackfan anemia: a ribosomal puzzle, stem cells from a haploidentical donor in a patient with severe ane-
Haematologica, 93(11):1601–1604, 2008. mia, J Hematol, 4(Suppl 2):47, 2003.
Gan SS, et al. Mycophenolate mofetil and aplastic anemia, J Hematol, Shimamura A. Clinical approach to marrow failure. Am Soc Hematol,
4(Suppl 2):47, 2003. Educational program, Vol. 1, Annual issue, 329–336, 2009.
Keohane EM. Acquired aplastic anemia. Cl Lab Sci, 17(3):165–170, Stern MA, Eckman J, Offermann MK. Aplastic anemia after exposure
2004. to burning oil, N Engl J Med, 331(1):58, 1994.
Lipton JM. Diamond-Blackfan anemia: novel mechansims-ribosomes Tulpule A, et al. Knockdown of Fanconi anemia genes in human
and the erythrron. Blood, 109(3):850, 2007. embryonic stem cells reveals early developmental defects in the
Mikhailova EA, et al. Program of combined immunosuppressive ther- hematopoietic lineage, Blood, 115(17):3453–3462, 2010.
apy, J Hematol, 4(Suppl 2):46, 2003. Yamaguchi H, et al. Mutations in TERT, the Gene for Telomerase Reverse
Miniero R, et al. Treatment of severe aplastic anemia (SAA) with Transcriptase in Aplastic Anemia, NEJM, 352(14):1413–1424, 2005.
mycophenolate mofetil: a case report, J Hematol, 4(Suppl 2):47, Young NS, Maciejewski J. The pathophysiology of acquired aplastic
2003. anemia, N Engl J Med, 336(19):1365–1372, 1997.

Turgeon_Chap09.indd 165 10/15/2010 12:49:59 PM


CHAPTER 10 ■ Iron Deficiency Anemia and Anemia of Chronic Inflammation 179

REVIEW QUESTIONS

1. The etiology of IDA is 14. In IDA, the erythrocytic indices are typically
A. nutritional deficiency A. MCV increased, MCH decreased, and MCHC
B. faulty iron absorption decreased
C. excessive loss of iron B. MCV decreased, MCH decreased, and MCHC
D. all of the above decreased
2. Iron deficiency is still common in C. MCV decreased, MCH increased, and MCHC
A. toddlers decreased
B. adolescent girls D. MCV decreased, MCH decreased, and MCHC
C. women of childbearing age normal
D. all of the above 15. The peripheral blood smear demonstrates _____ red
blood cells in IDA.
Questions 3 through 7: Match the following categories with A. microcytic, hypochromic
an appropriate example. (Use an answer only once) B. macrocytic, hypochromic
3. _____ Decreased iron intake C. macrocytic and spherocytic
4. _____ Faulty iron absorption D. either A or B
5. _____ Pathological iron loss
16. In IDA, the
6. _____ Physiological iron loss
A. serum iron is severely decreased and the TIBC is
7. _____ Increased iron utilization
increased
A. Sprue
B. serum iron is decreased and the TIBC is normal
B. Colon cancer
C. serum iron is normal and the TIBC is normal
C. Adolescent growth spurt
D. serum iron is increased and the TIBC is normal
D. Menstruation
E. Meat-poor diet 17. Anemias of inflammation/chronic diseases can be caused by
8. The average adult has _____ g of total iron. A. inflammation
A. 0.2 to 1.4 B. infection
B. 1.5 to 3.4 C. malignancy
C. 3.5 to 5.0 D. all of the above
D. 5.1 to 10.0 18. AOI can result from
9. Most functional iron in humans is found in A. inappropriately decreased erythropoietin
A. the bone marrow B. suppression of erythropoiesis by cytokines from
B. the liver activated macrophages and lymphocytes
C. hemoglobin molecules of erythrocytes (RBCs) C. impaired iron metabolism
D. the free hemoglobin in the circulation D. all of the above
19. The typical peripheral blood film of a patient with AOI
Questions 10 and 11: Approximately (10) _____% of iron typically reveals _____ erythrocytes.
from food is in the form of (11) _____ iron. A. microcytic, hypochromic
10.
B. macrocytic, hypochromic
A. 25 C. normocytic, normochromic
B. 50 D. many spherocytes
C. 70
D. 90 20. Leukoerythroblastosis can appear as _____ on a periph-
11.
eral blood smear.
A. immature leukocytes
A. Nonheme
B. immature erythrocytes
B. Heme
C. immature thrombocytes
12. Most ingested iron is readily absorbed into the body in the
D. both A and B
A. stomach and duodenum
B. duodenum and upper jejunum 21. What is the most appropriate treatment for AOI?
C. ileum and duodenum A. Red blood cell transfusion
D. upper jejunum and ileum B. Iron therapy
13. Transferrin represents a C. Erythropoietin injections
A. storage form of iron D. Treatment of the inflammatory condition
B. beta globulin that moves iron 22. Sideroblastic anemia can be caused by
C. glycoprotein that moves iron A. congenital (chromosomal) defect
D. both B and C B. drugs (e.g., chloramphenicol)

(continued)

Turgeon_Chap10.indd 179 10/15/2010 2:30:31 PM


180 PART 2 ■ Erythrocytes

REVIEW QUESTIONS (continued)

C. association with malignant disorders (e.g., acute my- C. Cytochromes


elogenous leukemia) D. Myoglobin
D. all of the above 25. Storage iron in the human body is
23. A common feature of sideroblastic anemia is A. found in hepatocytes
A. ringed sideroblasts B. found in macrophages
B. decreased serum iron C. sequestered as ferritin
C. decreased serum ferritin D. all of the above
D. macrocytic red blood cells 26. The most sensitive assay for the diagnosis of hereditary
24. The greatest portion of operational body iron is nor- hemochromatosis (HH) is
mally contained in what compound? A. serum iron
A. Hemoglobin B. serum iron–binding capacity
B. Ferritin C. transferrin
D. transferrin saturation

BIBLIOGRAPHY Hedenus M, et al. Darbepoetin alfa significantly increases hemoglo-


bin and reduces transfusions in patients with lymphoproliferative
Andrews NC. Forging a field: the golden age of iron biology, Blood, malignancies: Results of a randomized double-blind, placebo-
112(2):219–230, 2008. controlled study, Hematol J, 4(Suppl 2):253, 2003.
Adamson JW. The anemia of inflammation/malignancy: mecha- Hershko C, et al. Mechanism of iron regulation and of iron deficiency,
nisms and management, Am Soc of Hematol: ASH Education Book, Eur Hemtol Assoc, 1(1):1–8, 2007.
159–164, 2008. Jolobe O. Reticulocyte hemoglobin content vs. soluble transferrin
Bamberg R. Occurrence and detection of iron-deficiency anemia in receptor and feritn index in iron deficiency anemia accompanied
infants and toddlers, Clin Lab Sci, 21(4):225–231, 2008. with inflammation, Int J Lab Hem, 30:175–176, 2008.
Bergamaschi G, et al. Anemia of chronic disease and defective erythro- Laudicina RJ. Anemia in an aging population, Clin Lab Sci, 21(4):
poietin production in patients with celiac disease, Haematologica, 232–239, 2008.
93(12):1785–1790, 2008. Looker AC, et al. Prevalence of iron deficiency in the United States,
Beutler E, Waalen J. The definition of anemia: what is the lower limit JAMA, 277(12):973–976, 1997.
of normal of the blood hemoglobin concentration? Blood, 107 Lux SE. Hematologic aspects of systemic disease. In: Handin RI, Lux
(5):1747–1750, 2006. SE, Stossel TP (eds.). Blood, 2nd ed, Philadelphia, PA: Lippincott
Beutler E, West C. Hematologic differences between African- Williams & Wilkins, 2003:1977–2009.
Americans and whites: the roles of iron deficiency and (alpha) Malope BI, et al. The ratio of serum transferrin receptor and serum
thalassemia on hemoglobin levels and MCV, Blood, 106(2):740– ferritin in the diagnosis of iron status, Br J Haematol, 115:84–89,
745, 2005. 2001.
Beutler E, Studenski S. Toward a better understanding of anemia in Patel KV, Guralnik JM. Prognostic implications of anemia in older
older adults, The Hematologists, ASH News and Reports, 2(4):7, adults, Haematologica, 94(1):1–2, 2009.
2005. Patel KV. Variability and heritability of hemoglobin concentration: an
Bovy C. Mature erythrocytes indices: new markers of iron availability, opportunity to improve understanding of anemia in older adults,
Hematologia, 90(4):549–551, 2005. Hematologica, 93(9):1281–1283, 2008.
Brandão M, et al. The soluble transferrin receptor as a marker of iron Price EA, Schrier SL. Anemia in the older adult. www.UpToDate.com.
homeostasis in normal subjects and in HFE-related hemochroma- Retrieved April 6, 2010.
tosis, Haematologica, 90(1): 31–37, 2005. Skikne BS. Circulating transferrin receptor assay: Coming of age, Clin
Brugnara C, et al. Reticulocyte hemoglobin content to diagnose iron Chem, 44(1):7–9, 1998.
deficiency in children, JAMA, 281(23):2225–2230, 1999. Tsang CW, et al. Hematologic indices in an older population sample: Der-
Culleton BF, et al. Impact of anemia on hospitalization and mortality ivation of healthy reference values, Clin Chem, 44(1):96–101, 1998.
in older adults, Blood, 107(10):3841–3846, 2006. Turgeon ML. Anemia: is it iron deficiency? Presented at MEDLAB
Ekiz C, et al. the effect of iron deficiency anemia on the function of the 2009 Arab Health Conference, Dubai, UAE, Jan 29, 2009.
immune system. Hematol J, 5:579–583, 2005. Weiss G, Goodnough LT. Anemia of chromic disease, NEJM,
Greendyke RM, et al. Serum levels of erythropoietin and selected other 352(10):1011–1023, 2005.
cytokines in patients with anemia of chronic disease, Am J Clin Weiss G. Pathophysiology, diagnosis and treatment of the anemia of
Pathol, 101:338, 1994. chronic disease, Eur Hemtol Assoc, 1(1):9–16, 2007.

Turgeon_Chap10.indd 180 10/15/2010 2:30:31 PM


190 PART 2 ■ Erythrocytes

CASE STUDY

CASE 11.1
A 50-year-old white woman had seen her physician and
reported having no energy and feeling tired all the time.
She also reported experiencing mild pain in the abdominal
region. The physician ordered a routine CBC.
■ Laboratory Data
The results of the blood count were as follows:
Hemoglobin 6.2 g/dL
Hct 22%
RBC 1.7 × 1012/L
WBC 4.0 × 109/L
Her RBC indices were as follows: (Reprinted with permission from Anderson SC, Anderson’s
MCV 129.4 fL Atlas of Hematology, Philadephia, PA: Wolters Kluwer Health/
MCH 36.5 pg Lippincott Williams & Wilkins, Copyright 2003.)
MCHC 28 g/dL
The peripheral blood smear demonstrated abnormalities of
2. What specific kind of anemia can be diagnosed based on
erythrocytes and leukocytes. On receipt of the laboratory
the laboratory findings?
data, the physician ordered the following additional tests:
3. What is the etiology and physiological process in this
vitamin B12 and folate assays, reticulocyte count, serum
anemia?
iron and TIBC, serum bilirubin, and serum LDH. A fecal
examination for occult blood was additionally ordered. ■ Discussion
The results of the tests were as follows: 1. The increased RBC size as seen on the peripheral
Vitamin B12: 121 pmol/L (decreased) blood (macrocytes) and the increased MCV indicate a
Serum folate level: normal macrocytic-megaloblastic–type anemia.
Reticulocyte count: 0.4% 2. The two most common megaloblastic anemias are perni-
Serum iron and TIBC: normal cious anemia and folic acid deficiency. Supporting labo-
Serum bilirubin: 1.8 mg/dL (slightly increased) ratory assays can differentiate between these two types
Serum LDH: >3,000 units (significantly increased) of anemias. In this case, a decreased vitamin B12 level,
normal folic acid level, significantly increased LDH level,
The test result for occult blood was negative. and absence of hydrochloric acid in the stomach support
■ Questions
the diagnosis of pernicious anemia.
1. What category of anemia is suggested by the hematologi- 3. Many drugs can cause megaloblastic anemia by inter-
cal findings in this case? fering with DNA synthesis, functioning as folic acid
antagonists, or inhibiting purine and pyrimidine syn-
thesis. Classic pernicious anemia is a chronic disease
with a familial incidence, although no clear pattern of
genetic transmission exists. Pernicious anemia usually
becomes apparent in midlife or later. The macrocytosis
in pernicious anemia is the result of a defect in nuclear
maturation or DNA impairment. Because RNA synthesis
is normal, the normal nuclear-cytoplasmic ratio is matu-
rationally asynchronous. The disease complex includes
atrophy of the gastric mucosa and changes caused by
the deficiency of vitamin B12. Atrophic gastric mucosa
secretes neither IF nor hydrochloric acid. A few patients
have antibodies to IF in their gastric juice and serum.
This antibody condition causes a failure in the absorp-
tion of vitamin B12 even if it is available.
(Reprinted with permission from Rubin E, Farber JL. Pathol-
ogy, 3rd ed, Philadelphia, PA: Lippincott-Raven, 1999:1076.) DIAGNOSIS: Megaloblastic Anemia (Pernicious Anemia)

Turgeon_Chap11.indd 190 10/15/2010 9:33:39 AM


CHAPTER 11 ■ Megaloblastic Anemias 191

REVIEW QUESTIONS

1. Megaloblastic anemias can be caused by 6. The peripheral erythrocyte morphology in folate defi-
A. tapeworm infestation ciency is similar to pernicious anemia, and the RBCs
B. gastric resection are
C. nutritional deficiency A. small
D. all of the above B. normal size
2. Megaloblastic anemia related to folic acid deficiency is C. large
associated with 7. In a case of classic pernicious anemia, the patient has
A. abnormal absorption A. leukopenia
B. increased utilization B. hypersegmented neutrophils
C. nutritional deficiency C. anemia
D. all of the above D. all of the above
3. The underlying type A gastritis that causes pernicious 8. The reticulocyte count in a patient with untreated perni-
anemia is immunologically related to cious anemia is characteristically
A. autoantibody to IF A. 0%
B. low serum gastrin B. 0.3%
C. autoantibody to parietal cells C. <1.0%
D. both A and C D. approximately 1.8%
4. Cobalamin transport is mediated by
A. IF Questions 9 through 15: Match the following clinical
B. TC II chemistry assays with their expected value in pernicious
C. R proteins anemia: (An answer can be used more than once.)
D. all of the above 9. _____ Serum haptoglobin–binding capacity
5. In megaloblastic anemia, the typical erythrocytic indices 10. _____ Serum B12
are 11. _____ Folate
A. MCV increased, MCH increased, and MCHC 12. _____ Serum iron
normal 13. _____ Percent transferrin
B. MCV increased, MCH variable, and MCHC 14. _____ Serum LDH
normal 15. _____ Unconjugated bilirubin
C. MCV increased, MCH decreased, and MCHC A. Decreased
normal B. Normal
D. MCV normal, MCH increased, and MCHC C. Increased
normal D. Significantly increased

BIBLIOGRAPHY Harriman GR, et al. Vitamin B12 malabsorption in patients with acquired
immunodeficiency syndrome, Arch Intern Med, 149:2039–2041, 1989.
Andres E, Boichot B, Schlienger J. Food cobalamin malabsorption: Hillman RS, Finch CA. Red Cell Manual, 7th ed, Philadelphia, PA: FA
A usual cause of vitamin B12 deficiency, Arch Intern Med, 160:2061, Davis, 1996.
2000. Kapadia C, Donaldson RM. Disorders of cobalamin (vitamin B12)
Carson-DeWitt RS. Pernicious anemia. In: Olendorf D, Jeryan C, absorption and transport. Ann Rev Med, 36:93–110, 1985.
Boyden K (eds), Gale Encyclopedia of Medicine, 1st ed, Farmington Moridani M, Ben-Poorat S. Laboratory investigation of vitamin B12
Hills, MI: The Thomson Corp., 1999:2224. deficiency, Labmedicine, 37(3):166–174, 2006.
El-Newihi HM, et al. Gastric cancer and pernicious anemia appear- Snow CF. Laboratory diagnosis of vitamin B12 and folate deficiency,
ing as pseudoachalasia, SMJ, September 1996 (retrieved from www. Arch Intern Med, 159:1289–1298, 1999.
sma.org/ smj/96sept13.htm on February 21, 2003). Toh B-H, et al. Pernicious anemia. N Engl J Med, 337(20):1441–1448,
Emery E, Homans AC, Colletti RB. Vitamin B12 deficiency: a 1997.
cause of abnormal movements in infants, Pediatrics, 99(2):255, Turgeon ML. Immunology and Serology in Laboratory Medicine, 4 ed,
1997. St. Louis, MO: Elsevier, 2009.

Turgeon_Chap11.indd 191 10/15/2010 9:33:43 AM


CHAPTER 12 ■ Hemolytic Anemias 207

CASE STUDIES (continued)

■ Questions episodes were described as intermittent by the patient,


1. What is this patient’s corrected reticulocyte count? a diagnosis of PNH would be suspected. That condition
2. What is the RPI? is a rare, acquired chronic hemolytic anemia. The epi-
3. From what type of defect is this patient suffering? sodes of intravascular hemolysis typically occur while the
patient is asleep. During sleep, the blood pH decreases,
■ Discussion
making it easier for RBCs to lyse because of a membrane
1. The corrected reticulocyte count is as follows:
defect. This disorder represents an acquired erythro-
patient's hematocrit
Reticulocyte count (%) × =% cytic membrane defect that renders one population
normal hematocrit based
on age and gender of RBCs sensitive to lysis by normal plasma comple-
ment components. Hemosiderinuria is an important
0.25L / L
0.13 × = 7.2% diagnostic feature of this disorder. Demonstration of
0.45L / L PNH in vitro depends on the lysis of PNH erythrocytes
2. The RPI is as follows: by complement. A leukocyte alkaline phosphatase
cytochemical stain may also be valuable because PNH
Corrected reticulocyte count in % 7.2 is one of two disorders that show a decreased score.
= = 3.5%
Normal maturation time of 2 days 2 The other disorder is chronic myelogenous leukemia
(discussed in Chapter 21).
3. The occasional presence of brown urine on early morn-
ing voiding is suggestive of hemolysis. Because these DIAGNOSIS: Paroxysmal Nocturnal Hemoglobinuria

REVIEW QUESTIONS

1. Hemolytic disruption of the erythrocyte involves Questions 9 through 13: Match the following (use an
A. an alteration in the erythrocyte membrane answer only once):
B. a defect of the hemoglobin molecule 9. _____ Hereditary spherocytosis
C. an antibody coating the erythrocyte 10. _____ Hereditary elliptocytosis
D. physical trauma 11. _____ Hereditary pyropoikilocytosis (HPP)
12. _____ Hereditary stomatocytosis
Questions 2 and 3: Match the following. 13. _____ Hereditary xerocytosis
2. _____ Intravascular hemolysis A. An overabundance of oval-shaped red cells
3. _____ Extravascular hemolysis B. A permeability disorder
A. Destruction of RBCs outside the circulatory blood C. The most common prevalent hereditary hemolytic
B. Destruction of RBCs within the circulatory blood anemia among people of Northern European descent
4. Which of the following tests is not useful in determining D. Can be seen in the genetic hemoglobin defect, thala-
increased erythrocyte destruction? ssemia
A. Reticulocyte count E. A subgroup of common hereditary elliptocytosis
B. Total leukocyte count 14. Heinz bodies are associated with the congenital hemo-
C. Serum haptoglobin lytic anemia
D. Unconjugated bilirubin A. G6PD deficiency
B. abetalipoproteinemia
Questions 5 through 8: Match the following disorders C. hereditary spherocytosis
with the appropriate category of defect (an answer may be D. hemolytic anemias
used more than once): 15. A hemolytic crisis may be precipitated in 10% of Ameri-
5. _____ G6PD deficiency can black males suffering from G6PD deficiency by
6. _____ Hereditary spherocytosis A. fava beans
7. _____ Thalassemia B. primaquine
8. _____ Pyruvate kinase (PK) deficiency C. quinine
A. Structural membrane defect D. quinidine
B. Erythrocytic enzyme defect
C. Defect of the hemoglobin molecule

(continued)

Turgeon_Chap12.indd 207 10/15/2010 9:44:38 AM


208 PART 2 ■ Erythrocytes

REVIEW QUESTIONS (continued)

16. What is the most common glycolytic enzyme deficiency 23. _____ Cold-type AIHA
associated with the aerobic pathway of erythrocyte 24. _____ Isoimmune hemolytic anemia
metabolism? A. IgM, usually anti-I
A. Glucose-6-phosphate dehydrogenase (G6PD) B. Rh antibodies are the most frequent cause
B. Pyruvate kinase (PK) C. Usually occurs in newborn infants
C. Methemoglobin reductase deficiency 25. The erythrocyte alteration characteristically associated
D. Hexokinase deficiency with hemolytic anemias is
17. What is the most common glycolytic enzyme deficiency A. hypochromia
associated with the anaerobic pathway of erythrocyte B. macrocytosis
metabolism? C. spherocytosis
A. Glucose-6-phosphate dehydrogenase (G6PD) D. burr cells
B. Pyruvate kinase (PK) 26. What laboratory procedures would reflect a typical
C. Methemoglobin reductase deficiency hemolytic anemia?
D. Hexokinase deficiency A. Increased osmotic fragility
18. What laboratory assay would specifically indicate a defi- B. Increased total serum bilirubin
ciency of G6PD enzyme? C. Increased reticulocyte count, unless hematopoiesis is
A. Heinz bodies on peripheral blood smears suppressed
B. Reticulocyte count D. All of the above
C. Hemoglobin and hematocrit 27. Which of the following is not associated with hemolytic
D. Osmotic fragility test anemia?
19. What enzyme deficiency causes methemoglobinemia? A. Decreased hemoglobin and packed cell volume
A. Glucose-6-phosphate dehydrogenase (G6PD) B. Increased reticulocyte count
B. Pyruvate kinase (PK) C. Increased serum haptoglobins
C. NADH-methemoglobin reductase D. Decreased erythrocyte survival
D. Hexokinase deficiency 28. Paroxysmal nocturnal hemoglobinuria exhibits sensitiv-
20. Acquired hemolytic anemia can be caused by ity of one population of red blood cells to
A. chemicals or drugs A. warm antibodies
B. infectious organisms B. cold antibodies
C. antibody reactions C. complement
D. all of the above D. either A or B
21. The infectious microorganism directly associated with 29. Paroxysmal nocturnal hemoglobinuriaepisodes are usu-
hemolytic uremic syndrome is ally associated with
A. Pasteurella tularensis A. cold temperatures
B. E. coli O157-H7 B. hot temperatures
C. Staphylococcus aureus C. sleep
D. Clostridia botulinum D. certain foods or drugs
30. The defect in PNH probably is a (an) _____ associated
Questions 22 through 24: Match the following immune- defect of the red cell membrane.
mediated acquired hemolytic anemias with their respec- A. structural protein
tive answer (use an answer only once). B. hemoglobin
22. _____ Warm-type autoimmune hemolytic anemia C. antibody
(AIHA) D. enzyme

BIBLIOGRAPHY Bergstein JM, Riley M, Bang NU. Role of plasminogen-activator


inhibitor type 1 in the pathogenesis and outcome of the
Aster RH. Quinine sensitivity: A new cause of the hemolytic uremic hemolytic uremic syndrome, N Engl J Med, 327(11):755–759,
syndrome, Ann Intern Med, 119(3):243–244, 1993. 1992.
Au WU, et al. Late onset glucose 6-phosphate dehydrogenase defi- Bianchi P, et al. A lethal variant of pyruvate kinase deficiency associ-
ciency in Chinese women, J Hematol, 4(Suppl 2):50, 2003. ated with a missense mutation (G409) and a large deletion in the
Bell BP, et al. A multistate outbreak of Escherichia coli O157:H7-asso- PR-PK gene, Hematol J, 4(Suppl 2):57, 2003.
ciated bloody diarrhea and hemolytic uremic syndrome from ham- Bick RL. Paroxysmal nocturnal hemoglobinuria, Lab Med, 25(3):
burgers, JAMA, 272(17):1349–1353, 1994. 148–151, 1994.

Turgeon_Chap12.indd 208 10/15/2010 9:44:39 AM


232 PART 2 ■ Erythrocytes

REVIEW QUESTIONS

1. The common denominator in the hemoglobinopathies 10. What factors contribute to the sickling of erythrocytes
is that all are in sickle cell anemia crisis?
A. structural defects in the erythrocyte membrane A. Increase in blood pH and increase in oxygen
B. metabolic defects in the erythrocytic physiology B. Extremely hot weather
C. inherited or genetic defects related to hemoglobin C. Extremely reduced oxygen and increased acidity in
D. acquired defects related to hemoglobin the blood
2. Hemoglobinopathies can be classified as D. Sickling is spontaneous
A. abnormal hemoglobin globulin structure 11. The most common complaint associated with sickle cell
B. a defect of hemoglobin globulin synthesis anemia?
C. a combination of defects of both structure and synthesis A. Acute pain
D. all of the above B. Organ or tissue failure
3. Normal adult hemoglobin contains the following com- C. Stroke
ponents: Hb A (95% to 98%), Hb A2 (2% to 3%), Hb A1 D. All of the above
(3% to 6%), and Hb F (<1%). 12. Thalassemias are characterized by
A. True A. abnormal amino acid sequence of the hemoglobin
B. False molecules
4. In the hemoglobinopathies, a trait is described as B. defective iron synthesis
A. heterozygous and asymptomatic C. absence or decrease in synthesis of one or more
B. heterozygous and symptomatic globlin subunits
C. homozygous and asymptomatic D. skeletal membrane defects
D. homozygous and symptomatic 13. Homozygous b-thalassemia patients have
5. In sickle cell anemia the cause is A. no manifestations of anemia
A. a change of a single nucleotide (GAT to GTT) B. only mild anemia
B. the substitution of valine for glutamic acid at the sixth C. moderate anemia
position on the beta chain of the hemoglobin molecule D. severe transfusion-dependent anemia
C. not genetic 14. In a-type thalassemia, with three inactive a genes, which
D. both A and B of the following is characteristic?
6. In sickle cell disease the abnormality is related to A. Hb A2
A. the rate of synthesis of hemoglobin B. Hb A
B. an abnormal molecular structure of hemoglobin C. Hb H
C. an acquired defect D. Hb F and A2
D. a membrane dysfunction 15. What is the primary risk to thalassemia major patients
7. One of the two most common monogenetic diseases of who receive frequent and multiple blood transfusions?
man is A. Iron overload
A. sickle cell trait B. Citrate toxicity
B. sickle cell anemia C. Polycythemia
C. a-thalassemia D. Hyperviscosity
D. Hb SC disease 16. The peripheral blood smear in silent state patients with
8. If a patient with sickle cell anemia is in an acute crisis a-thalassemia typically appears as
state, peripheral blood smears may exhibit A. normochromic, normocytic
A. leptocytes B. microcytic, hypochromic
B. drepanocytes C. macrocytic, normocytic
C. ovalocytes D. macrocytic, hypochromic
D. stomatocytes 17. The characteristic hemoglobin concentration in a
9. What estimated percentage of black Americans are patient’s silent state with heterozygous b-thalassemia is
heterozygous for Hb S? A. Hb A level normal
A. 4% B. Hb A2 increased
B. 8% C. Hb A2 level decreased
C. 12% D. Hb F level increased
D. More than 25%

(continued)

Turgeon_Chap13.indd 232 10/13/2010 6:59:35 PM


CHAPTER 13 ■ Hemoglobinopathies and Thalassemias 233

REVIEW QUESTIONS (continued)

18. Deoxyhemoglobin C has C. Vietnam


A. decreased solubility D. Native Americans
B. increased solubility 20. Most unstable hemoglobins
C. the ability to form intracellular crystals A. are inherited autosomal dominant disorders
D. both A and C B. result from amino acid substitutions or deletions
19. The incidence of Hb E hemoglobinopathy is highest in C. are hemoglobin variants
A. Southeast Asia D. all of the above
B. China

BIBLIOGRAPHY Mayo Clinic Staff. www.MayoClinic/sickle cell anemia.com. Retrieved


16 December 2009.
Abou-Diwan C, Young AN, Molinaro RJ. Hemoglobinopathies and Persons DA. Hematopoietic stem cell gene transfer for the treatment
clinical laboratory testing, Med Lab Observ, 41(8):10–16, 2009. of hemoglobin disorders, Am Soc Hematol, 690–697, 2009 (50th
Abdulrahman, J. Hemoglobinopathies in the United Arab Emirates: Anniversary Issue).
current screening and diagnostic techniques. Unpublished Mas- Platt OS, Guinan EC. Bone marrow transplantation in sickle cell
ter’s Thesis: Northeastern University, Boston, 2006. anemia—the dilemma of choice, N Engl J Med, 335(6):426–427, 1996.
Ahmed S, et al. Screening extended families for genetic hemoglobin Rund D, Rachmilewitz E. b-Thalassemia, NEJM, 353:11, 2005.
disorders in Pakistan, N Engl J Med, 347(15):1162–1168, 2002. Steensma DP, Hoyer JD, Fairbanks VF. Hereditary red blood cell
Alymara V, et al. Evaluation of pulmonary function in patients with disorders in middle eastern patients, Mayo Clinic Proc, 76:285–293,
b-thalassemia Major, J Hematol, 20(4 Suppl 2):61, 2003. 2001.
Angelucci E, Baronciani D. Allogenic stem cell transplantation for Steinberg MH. Management of sickle cell disease, N Engl J Med,
thalassemia major, Hematologia, 93(12):1780, 2008. 340(13):1021–1030, 1999.
Ataga KI. Novel therapies in sickle cell disease, Am Soc Hematol, , Thien SL. Genetic modifiers of b-thalassemia. Haematologica,
54–60, 2009 (50th Anniversary Issue). 90(5):649–660, 2005.
Camberlein E, et al. Anemia in b-thalassemia patients targets hepatic Turgeon ML. Clinical Hematology, 4th ed, Philadelphia, PA: Lippin-
hepcidin transcript leels independently of iron metabolism genes cott Williams & Wilkins, 2003.
controlling hepcidin expression, Haematologica, 93(1):111–115, U.S. Department of Health and Human Services. Guideline: laboratory
2008. screening for sickle cell disease, Lab Med, 24(8):515–522, 1993.
Cao A, Galanello R. Effect of consanguinity on screening for thalas- Verduzco LA, Nathan DG. Sickle cell disease and stroke, Blood,
semia, N Engl J Med, 347(15):1200–1202, 2002. 114(25):5117–5125, 2009.
Chen FE, et al. Genetic and clinical features of hemoglobin H disease Vichinsky VP. Changing Patterns of Thalassemia Worldwide, Annals of
in Chinese patients, N Engl J Med, 343(8): 544–550, 2000 the New York Academy of Sciences-Cooley’s Anemia: Eighth Sympo-
Chui D. Thal for Thal, Blood, 110(8):2788–2789, 2007. sium, 1054:18–24, 2006.
Cohen AR. Iron chelation therapy: you gotta have heart, Blood, Voon HPJ, Vadolas J. Controlling a-globin: a review of a-globin
115(12), 2333–2334, 2010. expression and its impact on b-thalassemia, Haematologica,
Colombatti R, et al. Hospitalization of children with sickle cell dis- 93:1868–1876, 2008.
ease in a region with increasing immigration rates, Haematologica, Voskaridou E, et al. The effect of prolonged administration of
93(3):463–464, 2008. hydroxyurea on morbidity and mortality in adult patients with
Farmaki KF. Combined chelation therapy in patients with thalas- sickle cell syndromes: results of a 17-year, single-center trial
saemia major: Research protocol or routine therapy, J Hematol, (LaSHS), Blood, 115(12):2354–2363, 2010.
4(Suppl 2):64, 2003. Wadia MR, et al. Usefulness of automated chromatography for rapid
Hoppe CC. Newborn screening for non-sickling hemoglobinopathies, fetal blood analysis for second trimester prenatal diagnosis of
Am Soc Hematol, 19–40, 2009 (50th Anniversary Issue). b-thalassemia, Prenat Diagn, 22:153–157, 2002.
Khoury MJ, et al. Population screening in the age of genomic medi- Ware RE, Aygun B. Advances in the use of hydroxyurea, Am Soc
cine, N Engl J Med, 348(1):50–58, 2003. Hematol, 62–68, 2009 (50th Anniversary Issue).
Matsui NM. P-selectin mediates the adhesion of sickle erythrocytes to Zago MA, et al. Hydroxyurea treatment reduces the exposure of phos-
the endothelium, Blood, 98(6):1955–1962, 2001. phatidylserine on erythrocytes and platelets in sickle cell anemia,
J Hematol, 4(Suppl 2):60, 2003.

Turgeon_Chap13.indd 233 10/13/2010 6:59:35 PM


CHAPTER 14 ■ Leukocytes: The Granulocytic and Monocytic Series 251

CASE STUDY (continued)

A few nucleated red blood cells were observed per 100 stress from a variety of nonmalignant disorders (e.g.,
WBCs on the peripheral blood smear. severe infection or inflammation, ovarian cysts, or
inflammatory bowel disease).
■ Questions
3. To distinguish between a leukemic condition and a
1. What abnormalities appeared in the patient’s laboratory
leukemoid reaction, the LAP test could be used. An
values?
increased score is associated with a leukemoid reaction
2. What could cause this type of abnormality?
or a variety of malignancies (e.g., lymphoma, multiple
3. How could leukemia be differentiated from a leukemia-
myeloma, polycythemia vera). Decreased scores are
like, leukemoid, reaction or lymphoma?
associated with chronic myelogenous leukemia and
■ Discussion some other conditions.
1. A leukoerythroblastic reaction is evident because of the
presence of both immature granulocytes and nucleated DIAGNOSIS: Leukemoid Reaction Subsequent to
erythrocytes. Ovarian Cysts
2. This type of reaction could be associated with malignant
diseases (e.g., leukemias or lymphomas) or exogenous

REVIEW QUESTIONS

1. The granulocyte cells that are believed to descend from a 5. The stages of neutrophilic granulocyte development are
common multipotential stem cell in the bone marrow are A. promyelocyte, myeloblast, myelocyte, metamyelocyte,
A. neutrophils and eosinophils and band and segmented neutrophils
B. basophils and lymphocytes B. myeloblast, promyelocyte, myelocyte, metamyelocyte,
C. lymphocytes and monocytes and band and segmented neutrophils
D. Both A and B C. myelocyte, myeloblast, promyelocyte, metamyelocyte,
2. The types of granulocytic leukocytes found in the prolif- and band and segmented neutrophils
erative compartment of the bone marrow are D. myeloblast, promyelocyte, metamyelocyte,
A. myeloblasts, myelocytes, and metamyelocytes myelocyte, and band and segmented neutrophils
B. myeloblasts, promyelocytes, and myelocytes 6. Marginating granulocytes in the peripheral blood can be
C. myeloblasts, promyelocytes, myelocytes, and found
metamyelocytes A. in the circulating pool
D. myeloblasts, promyelocytes, myelocytes, metamy- B. in the tissues
elocytes, and band neutrophils C. adhering to the vascular endothelium
3. The types of granulocytic leukocytes found in the D. all of the above
maturation-storage compartment of the bone marrow 7. The major function of neutrophilic granulocytes is
are A. antibody production
A. metamyelocytes, band form neutrophils, segmented B. destruction of parasites
neutrophils, mature eosinophils, and mature baso- C. phagocytosis
phils D. suppression of inflammation
B. only band form neutrophils, segmented neutrophils, 8. The half-life of circulating granulocytes in normal blood
mature eosinophils, and mature basophils is estimated to be
C. metamyelocytes, band form neutrophils, segmented A. 2.5 to 5 hours
neutrophils, mature eosinophils, and mature baso- B. 7 to 10 hours
phils C. 24 hours
D. segmented neutrophils, immature and mature D. 2 days
monocytes, and mature lymphocytes 9. Identify the cell with these characteristics: prominent
4. Release of neutrophils from the bone marrow is believed primary granules that are rich in myeloperoxidase and
to be influenced by chloroacetate esterase and has a diameter of 14 to 20 mm.
A. CSF A. myeloblast
B. interleukins B. promyelocyte
C. interferon C. myelocyte
D. all of the above D. promonocyte

(continued)

Turgeon_Chap14.indd 251 10/13/2010 7:05:32 PM


252 PART 3 ■ Leukocytes

REVIEW QUESTIONS (continued)

10. The earliest granulocytic maturational stage in which 19. The immediate precursor of the macrophage is the
secondary or specific granules appear is A. myeloblast
A. myeloblast B. monoblast
B. monoblast C. promonocyte
C. promyelocyte D. monocyte
D. myelocyte 20. The correct sequence(s) of events in successful
11. The mature granulocytes seen in the peripheral blood of phagocytosis is (are)
healthy persons include A. chemotaxis, opsonization, phagosome formation,
A. band form and segmented neutrophils and the action of antibacterial substances
B. eosinophils and basophils B. opsonization, chemotaxis, phagosome formation,
C. lymphocytes and monocytes and the action of antibacterial substances
D. Both A and B C. engulfment, opsonization, digestion, and destruction
12. The granules of segmented neutrophils contain of bacteria or particulate matter
A. lysosomal hydrolases D. Both A and C
B. lysozymes 21. The major function of eosinophils is
C. myeloperoxidase A. suppression of inflammatory reactions
D. all of the above B. destruction of protozoa
13. Which of the following are contents of basophilic C. participation in anaphylaxis
granules? D. phagocytosis
A. Heparin 22. Monocytes are capable of
B. Histamine A. phagocytosis
C. Myeloperoxidase B. synthesis of biologically important compounds
D. Both A and B C. assuming a killer role
14. The tissue basophil can be referred to as a/an D. All of the above
A. mast cell 23. The hematology tests that are useful in the early diagnosis
B. macrophage of acute inflammation are the
C. mononuclear cell A. total leukocyte count and total erythrocyte count
D. antibody-producing cell B. total leukocyte count and white blood cell differential
15. A leukocyte with the morphological characteristics of being count
the largest normal mature leukocyte in the peripheral C. ESR and absolute neutrophil cell count
blood and having a convoluted or twisted nucleus is the D. Both B and C
A. myelocyte 24. The total leukocyte count can be increased in certain
B. metamyelocyte states. Select the conditions when this is not true.
C. promonocyte A. Strenuous exercise
D. monocyte B. Overwhelming bacterial infection
16. The reference range of PMN neutrophil count in adults C. Sepsis
is D. Use of immunosuppressive agents
A. 20 – 40% 25. Acute inflammation is based on
B. 40 – 60% A. total leukocyte count >10.5 × 109/L
C. 60 – 80% B. neutrophilic band count <2%
D. 80 – 100% C. symptoms of long duration
17. The principal leukocyte type involved in phagocytosis is D. an increase in lymphocytes
the 26. On the basis of the following data, calculate the absolute
A. monocyte value of the segmented neutrophils. Total leukocyte
B. neutrophil count = 12 × 109/L; percentage of segmented neutro-
C. eosinophil phils on the differential count = 80%. The absolute
D. basophil segmented neutrophil value is
18. The mononuclear phagocyte system consists of reticular A. 2.5 × 109/L
cells. These cells can be found in the B. 4.5 × 109/L
A. connective tissue C. 6.5 × 109/L
B. spleen D. 9.6 × 109/L
C. lymph nodes
D. all of the above

(continued)

Turgeon_Chap14.indd 252 10/13/2010 7:05:32 PM


CHAPTER 14 ■ Leukocytes: The Granulocytic and Monocytic Series 253

REVIEW QUESTIONS (continued)

27. An increase in metamyelocytes, myelocytes, and 29. The absolute value of segmented neutrophils can be an
promyelocytes can be referred to as unreliable indicator of overwhelming infection because
A. leukocytopenia A. it drops in many patients because the circulating gran-
B. a shift to the right ulocytes are mobilized into the tissue site of infection
C. a shift to the left B. the bone marrow reserve becomes exhausted
D. Pelger-Huet anomaly C. the infection suppresses granulocytic production
28. What is the normal range of the segmented neutrophil D. All of the above
absolute value? 30. The CLSI-recommended method for the ESR is the
A. 1.4 to 6 × 109/L A. Wintrobe method
B. 2.5 to 6.5 × 109/L B. Westergren method
C. 3.5 to 8 × 109/L C. Duke
D. 5.5 to 10 × 109/L D. Ivy

BIBLIOGRAPHY Mueller H, et al. Tyrosine kinase Btk regulates E-selectin-mediated


integrin activation and neutrophil recruitment by controlling
Akin, C. What does a basophil do? Blood, 110(3):790, 2007. phospholipase C (PLC)g2 and PI3Kg pathways, Blood, 15(15):
Baer DM (ed.). Absolute neutrophil critical values, Med Lab Observ, 3118–3127, 2010.
34(9):34, 2002. Nimrichter L, et al. E-selectin receptors on human leukocytes, Blood,
Bunting M, et al. Leukocyte adhesion deficiency syndromes: Adhesion 112(9):3744–3752, 2008.
and tethering defects involving b2 integrins and selectin ligands, Pelletier M, et al. Evidence for a cross-talk between human neutrophils
Curr Opin Hematol, 9(1):30–35, 2002. and Th17 cells, Blood, 115(2):335–343, 2010.
Dale DC, Boxer L, Liles WC. The phagocytes: neutrophils and monocytes, Peters-Golden M, Henderson WR Jr. Leukotrienes, NEJM, 357:
Am Soc Hematol, ASH 50th Anniversary Review, 121–128, 2008. 1841–1854, 2007.
Delves PJ, Roitt IM. The immune system, N Engl J Med, 343(1):37–50, Phipps S, et al. Eosinophils contribute to innate antiviral immu-
2000. nity and promote clearance of respiratory syncytial virus, Blood,
Dinarello CA. The role of the interleukin-1-receptor antagonists in 110(5):1578–1586, 2007.
blocking inflammation mediated by interleukin-1, N Engl J Med, Ridker PM, et al. C-Reactive protein and other markers of inflamma-
343(10): 732–734, 2000. tion in the prediction of cardiovascular disease in women, N Engl J
Etzioni A. Integrins: The molecular glue of life, Hosp Pract, (www.hosppract. Med, 342 (12):836–837, 2000.
com/issues/2000/03/etzioni.htm). Retrieved November 14, 2002. Saffar AS, Gounni AS. Neutrophils in health and disease. MedLab Mag,
Gerard C. Complement C5a in the sepsis syndrome—Too much of a 1(1):18–19, 2009.
good thing, N Engl J Med, 348(2):167–169, 2003. Statland BE (ed.). Normal values for bands, Med Lab Observ, 17(4):12,
Hotchkiss RS, Karl IE. The pathophysiology and treatment of sepsis, 1985.
N Engl J Med, 348(2):138–150, 2003. Soehnlein O, Lindbom L, Weber C. Mechanisms underlying neutro-
Klein C. Congential neutropenia, Am Soc Hematol, 344–348, 2009; phil-mediated monocyte recruitment, Blood, 114(21):4613–4621,
50th Anniversary edition 2009. 2009.
Koepke JA, Bull BS, Simson E, van Assendelft, OW. Reference and Van Ziffle JA, Lowell CA. Neutrophil-specific deletion of Syk kinase
Selected Procedure for the Human Erythrocyte Sedimentation Rate results in reduced host defense to bacterial infection, Blood,
(ESR) test. H2-T2. Approved Standard, 4th ed, Villanova, PA: 114(23):4871–4882, 2009.
NCCLS, 1977. Wheeler AP, Bernard GR. Treating patients with severe sepsis, N Engl J
Markell EK, et al. Medical Parasitology, Philadelphia, PA: Saunders, Med, 340(3):207–214, 1999.
1986:314. Wilson JM, Ziemba SE. Neutrophils fight infectious disease but may
McCabe BH. A brief history of the erythrocyte sedimentation rate, Lab promote other human pathologies, Lab Med, 30(2):123–128,
Med, 16(3):177–178, 1985. 1999.

Turgeon_Chap14.indd 253 10/13/2010 7:05:32 PM


264 PART 3 ■ Leukocytes

CASE STUDIES (continued)

E. chaffeensis, and the HGE agent as well as polymerase chain ■ Questions


reaction (PCR) for the causative agents of human ehrlichio- 1. Which serological assays were significant in establishing
sis were ordered. The patient was admitted to the hospital. a diagnosis?
2. Is this confirmation of HGE?
■ Laboratory Data
3. Is HGE more common in children than adults?
RBC 2.90 × 1012/L
Hemoglobin 9.5 g/dL ■ Discussion
Hematocrit 27% 1. The diagnosis of HGE was established serologically by
WBC 10.0 × 109/L the positive PCR for HGE agent, the presence of an ele-
Platelet count 175.0 × 109/L vated IFA to E. equi, and an increase in the IFA E. equi
titer between the acute and convalescent specimens.
■ Leukocyte Differential
2. Yes. The confirmed case definition of HGE includes a
Band neutrophils 10%
fourfold rise in antibody titer or seroconversion between
Segmented neutrophils 59%
acute and convalescent specimens.
Lymphocytes 21%
3. Unlike Lyme disease, another tickborne infectious disease,
Monocytes 10%
HGE is less common in children than in adults. Among
Platelet distribution was slightly decreased. RBC morphol- confirmed cases, rates of Ehrlichiosis in Connecticut,
ogy was within normal limits. children who were 9 years old or younger had the lowest
Subsequent serology specimens reported that the acute rate of confirmed cases compared to Lyme disease where
polyclonal (indirect fluorescent antibody [IFA]) for E. equi disease rates are highest in this age group. Very few cases
was nonspecific. The convalescent specimen obtained 23 of pediatric HGE are diagnosed in individuals younger
days later was positive (titer of 1:320). Five months later, than 20 years of age.
the IFA for E. equi was positive for IgG (1:264), but the
IgM component was less than 1:20. IgM and IgG titers for DIAGNOSIS: Human Granulocytic Ehrlichiosis (HGE)
E. chaffeensis were negative.

REVIEW QUESTIONS

1. Leukocytosis can be caused by 4. Monocytosis can be observed in


A. increased movement of immature cells out of the A. tuberculosis
bone marrow’s proliferative compartment B. fever of unknown origin
B. increased mobilization of granulocytes from the C. rheumatoid arthritis
maturation-storage compartment D. all of the above
C. increased movement of granulocytes from the mar- 5. Neutropenia can be observed in
ginating pool to the circulating pool A. bone marrow injury
D. all of the above B. nutritional deficiency
2. Neutrophilia can be related to a variety of conditions or C. increased destruction and utilization
disorders. Select the appropriate conditions. D. all of the above
A. Surgery
Questions 6 through 9: Match the following abnormalities
B. Burns
with the appropriate characteristic.
C. Stress
A. Gigantic peroxidase-positive deposits.
D. All of the above
B. Precipitated mucopolysaccharides.
3. Charcot-Leyden crystals can be found in _____ of
C. Döhle body–like inclusions.
patients with active eosinophilic inflammation. D. Single or multiple pale-blue staining inclusions.
A. sputum
6. _____ Alder-Reilly inclusions
B. tissues
7. _____ Chédiak-Higashi syndrome
C. stool
8. _____ Döhle body inclusions
D. all of the above
9. _____ May-Hegglin anomaly

(continued)

Turgeon_Chap15.indd 264 10/15/2010 9:46:05 AM


CHAPTER 15 ■ Nonmalignant Disorders of Granulocytes and Monocytes 265

REVIEW QUESTIONS (continued)

Questions 10 through 12: Match the following abnormali- Questions 17 through 21: Select the appropriate cell type
ties with the appropriate characteristic. involved in the following disorders.
A. Dark blue-black precipitates of RNA. A. Neutrophilic series
B. Five or more nuclear segments. B. Monocytic-macrophage series
C. Failure of the nucleus to segment. 17. _____ Gaucher disease
D. Precipitated mucopolysaccharides. 18. _____ Niemann-Pick disease
10. _____ Pelger-Huët anomaly 19. _____ Chédiak-Higashi syndrome
11. _____ Toxic granulation 20. _____ Chronic granulomatous disease
12. _____ Hypersegmentation 21. _____ Lazy leukocyte syndrome
22. In the United States, human diseases caused by Ehrlichia
Questions 13 through 16: Match the following abnormalities species can be caused by
with the appropriate condition. A. E. chaffeensis
A. Associated with a deficiency of vitamin B12 or folic B. E. ewingii
acid C. E. phagocytophilia (similar or identical to)
B. Associated with frequent infections in children or D. all of the above
young adults 23. Ehrlichiosis is transmitted by _____.
C. May be related to a maturational arrest in some acute A. mosquitoes
infections B. ticks
D. Associated with viral infections and burns C. rats
13. _____ Chédiak-Higashi syndrome D. cats
14. _____ Döhle bodies 24. Gaucher cells have
15. _____ Pelger-Huët anomaly A. wrinkled cytoplasm
16. _____ Hypersegmentation B. one to three nuclei
C. a deficiency of b-glucocerebrosidase
D. all of the above

BIBLIOGRAPHY Howard J. Myeloid series abnormalities: Neutrophilia, Lab Med,


14(3):147–151, 1983.
Bakken JS, et al. Human granulocytic ehrlichiosis in the Upper Mid- Koepke JA (Ed). Leukocyte Differential Counting, Tentative Standard
west United States, JAMA, 272(3):212–218, 1994. H20-T. Villanova, PA: National Committee for Clinical Laboratory
Barenfanger J, et al. Identifying human ehrlichiosis, Lab Med, Standards (NCCLS), 1982.
27(6):372–374, 1996. Lopker A, et al. Stereoselective muscarinic acetylcholine and opiate
Beutler E. Gaucher’s disease, N Engl J Med, 325(19):1354–1359, 1991. receptors in human phagocytic leukocytes, Biochem Pharmacol,
Centers for Disease Control and Prevention (CDC). Human ehrlichio- 29:1361–1365, 1980.
sis in the United States (www.cdc.gov). Retrieved October 16, 2003. Markell EK, et al. Medical Parasitology, Philadelphia, PA: Saunders,
Dannenberg AM Jr. Macrophages and monocytes. In: Spivak JL (ed.). 1986:314.
Fundamentals of Clinical Hematology, Hagerstown, MD: Harper & O’Connor BH. A Color Atlas and Instruction Manual of Peripheral
Row, 1980:137–153. Blood Morphology, Baltimore, MD: Williams & Wilkins, 1984.
Figueroa ML, et al. A less costly regimen of alglucerase to treat Gau- Raphael SS. Lynch’s Medical Laboratory Technology, 4th ed., Philadelphia,
cher’s disease, N Engl J Med, 327(23):1632–1636, 1992. PA: Saunders, 1983:653–692.
Francis GE, et al. DNA strand breakage and ADP-ribosyl transferase Robinson SH, Reich PR. Hematology: Pathophysiological Basis for Clin-
mediated DNA ligation during stimulation of human bone mar- ical Practice, 3rd ed., Boston: Little, Brown & Co., 1993.
row cells by granulocyte-macrophage colony stimulating activity, Sidransky E, Ginns EI. Clinical heterogeneity among patients with
Leuk Res, 8(3):407–415, 1984. Gaucher’s disease, JAMA, 269(9):1154–1157, 1993.
Grabowski GA, et al. Enzyme therapy in type I Gaucher disease: Compar- Stein RB. Granulocytosis and granulocytic leukemoid reactions. In:
ative efficacy of mannose-terminated glucocerebrosidase from natural Koepke J (ed.). Laboratory Hematology, Edinburgh, UK: Churchill-
and recombinant sources, Ann Intern Med, 122(1):33–39, 1995. Livingstone, 1984:153–187.
Hartmann LC, et al. Granulocyte colony-stimulating factor in Tsan M-F. Neutrophils. In: Spivak JL (ed.). Fundamentals of Clinical
severe chemotherapy-induced afebrile neutropenia, N Engl J Med, Hematology, Hagerstown, MD: Harper & Row, 1980:109–117.
336(25):1776–1785, 1997. Wynn TE. Letters to the editor, Lab Med, 15(4):276–277, 1984.
Hellmann A. Production of colony stimulating activity in mixed Zigmond SH, Lauffenburger DA. Assays of leukocyte chemotaxis,
mononuclear cell culture, Br J Haematol, 45:245–249, 1980. Annu Rev Med, 137:149–155, 1986.

Turgeon_Chap15.indd 265 10/15/2010 9:46:05 AM


CHAPTER 16 ■ Leukocytes: Lymphocytes and Plasma Cells 279

average of 35% lymphocytes in the circulating blood, with a are responsible for cellular immune responses and in either
distribution of 60% to 80% T cells and 20% B cells. helping or suppressing the activation of B lymphocytes. B
lymphocytes serve as the primary source of cells responsible
Morphological Characteristics of Normal for humoral immune (antibody) responses. NK lympho-
Lymphocytes cytes lack the mature surface markers of T and B cells. These
cells destroy target cells through the nonphagocytic process
The stages of lymphocyte development are the lymphoblast, referred to as a cytotoxic reaction.
the prolymphocyte, and the mature lymphocyte. When lym- All cells of the immune system have specialized recep-
phocytes are stained with Wright stain, their maturational tors on their membrane surfaces. Functional testing
characteristics are generally consistent with those seen in may be used to evaluate the response of lymphocytes to
other leukocytes. mitogens.
Lymphocytes can assume variant morphological features.
As many as 5% to 6% of variant lymphocytes can be found in Plasma Cell Development and Maturation
healthy persons; however, an increase in these forms is asso-
ciated with a variety of disorders. Some of the disorders that Plasma cells are not normally found in the circulating blood
can produce an increase in variant lymphocytes are infec- but can be seen in nonmalignant or malignant disorders.
tious mononucleosis, viral hepatitis, and viral pneumonia. Plasma cells arise from stimulated B lymphocytes. Abnor-
mal forms of the plasma cell include Russell body inclusions,
Functions and Membr ane Characteristics of grape cells, and flame cells.
Lymphocytes A few plasma cells can be seen in the circulating blood in
severe chronic infections or viral disorders. Increased num-
The categories of lymphocytes that are recognized as func- bers of plasma cells are associated with malignant conditions,
tionally active are T cells, B cells, and NK cells. T lymphocytes such as multiple myeloma.

REVIEW QUESTIONS

Questions 1 through 6: Match the following anatomical 10. T lymphocytes constitute _____% of the blood lympho-
structures with the appropriate anatomical category. An cyte pool in adults.
answer can be used more than once. A. 0–20
1. _____ Lymph nodes B. 20–40
2. _____ Liver C. 40–60
3. _____ Spleen D. 60–80
4. _____ Red bone marrow 11. Lymphocytes represent approximately _____% of the
5. _____ Thymus total circulating leukocytes in adults.
6. _____ Peyer patches of the intestine A. 15
A. Primary lymphoid tissue B. 35
B. Secondary lymphoid tissue C. 55
C. Not a lymphoid tissue D. 75
7. T cells are found in the: 12. The percentage of lymphocytes as compared with the
A. perifollicular areas of the lymph nodes other types of leukocytes in the peripheral blood _____
B. paracortex regions of the lymph nodes as humans age.
C. periarteriolar regions of the spleen A. increases
D. all of the above B. decreases
8. A major site of B-lymphocyte localization and prolifera- C. remains the same
tion is 13. If a patient has a total leukocyte count of 20 × 109/L and
A. lymphoid follicles a 50% lymphocyte count on the differential count, the
B. deep cortical zone absolute lymphocyte value is _____ × 109/L.
C. paracortex A. 1
D. all of the above B. 5
9. The process of lymphocyte recirculation is important in C. 10
A. antibody production D. 15
B. lymphocyte proliferation
C. dissemination of antigen-sensitized memory cells Questions 14 through 18: Complete the following state-
D. commitment of lymphocytes to T and B cells ments with answers A, B, or C.

(continued)

Turgeon_Chap16.indd 279 10/15/2010 9:49:05 AM


280 PART 3 ■ Leukocytes

REVIEW QUESTIONS (continued)

14. As a lymphocyte matures, the nuclear-cytoplasmic ratio 24. T cells are


_____. A. lymphocytes
15. As a lymphocyte matures, the overall size _____. B. monocytes
16. As a lymphocyte matures, the number of nucleoli C. helper or suppressor types
_____. D. both A and C
17. As a lymphocyte matures, the chromatin clumping 25. B cells are
_____. A. lymphocytes
18. As a lymphocyte matures, the quantity of cytoplasm B. associated with antigen recognition
_____. C. found in the thymus and bone marrow
A. increases D. all of the above
B. decreases 26. NK cells are classified as
C. remains about the same A. macrophages
19. The most characteristic morphological features of vari- B. monocytes
ant lymphocytes include C. effector lymphocytes
A. increased overall size, possibly one to three nucleoli, D. K-type lymphocytes
and abundant cytoplasm 27. Which of the following statements is (are) true of T
B. increased overall size, round nucleus, and increased cells?
granulation in the cytoplasm A. Responsible for humoral responses
C. segmented nucleus, light-blue cytoplasm, and no B. Responsible for cellular immune responses
nucleoli C. Responsible for chronic rejection in organ trans-
D. enlarged nucleus, six to eight nucleoli, and dark-blue plantation
cytoplasm D. Both B and C
28. Which of the following statements is (are) true of B
Questions 20 through 23: Match the following using an cells?
answer only once. A. Responsible for antibody responses
20. _____ Rieder cells B. Protect against intracellular pathogens
21. _____ Vacuolated lymphocytes C. Responsible for chronic rejection in transplanta-
22. _____ Crystalline inclusions tion
23. _____ Smudge cells D. Both A and B
A. Niemann-Pick disease and Burkitt lymphoma 29. An abnormal plasma cell with red-staining cytoplasm
B. CLL is a
C. Leukosarcoma A. Russell body
D. Natural artifact B. Mott cell
C. grape cell
D. flame cell

BIBLIOGRAPHY LeBien TW, Tedder TF. B lymphocytes: how they develop and func-
tion, Blood, 2008; 112(5):1570–1580.
Beck JW, Davies JE. Medical Parasitology, 2nd ed, St. Louis, MO: Sigma Diagnostics, Monoclonal Antibodies, St. Louis, MO: 1985.
Mosby, 1976:58–61. Reisfeld RA. Memory T cells: death by acquisition, Blood,
Caligiuri MA. Human natural killer cells, Blood 2008;112(5):,461–469. 2007;109(6):2269–2270.
Caligiuri MA, Blaser BW. Natural killer cells: development, receptor Seif AE, et al. Identifying autoimmune lymphoproliferative syndrome
biology, and their role in cancer immunotherapy, The Hematolo- in children with Evans syndrome: a multi-institutional study,
gists 2008;5(1):7. Blood, 115(11):2142–2145, 2010.
Giovannetti A, Stifano AG, Pierdominici M. Evaluating T cell Zhang X, Jianjua Y. Target recognition-induced NK-cell responses,
homeostasis in the biomedical laboratory, MedLab Mag, Blood, 115(11):2119–2120, 2010.
2008;(3):26–28. Zhu J, Paul, WE. CD4 T cells: fates, functions, and faults, Blood,
Henry JB. Clinical Diagnosis and Management, 17th ed, Philadelphia, 112(5):1557–1569, 2008.
PA: Saunders, 1984:824–827, 1049, 1286, 1290–1291.
Koepke JA, et al. Reference Leukocyte Differential Count (Proportional)
and Evaluation of Instrumental Methods H20-A, Villanova, PA:
NCCLS, 1992.

Turgeon_Chap16.indd 280 10/15/2010 9:49:05 AM


CHAPTER 17 ■ Leukocytes: Nonmalignant Lymphocytic Disorders 297

REVIEW QUESTIONS

1. Lymphocytopenia means a C. no agglutination of the patient’s serum with horse


A. total increase in leukocytes erythrocytes
B. total increase in lymphocytes D. all of the above
C. total increase in the absolute value or percentage of 11. Which of the following characterizes infectious lympho-
lymphocytes cytosis?
D. total decrease in lymphocytes A. An adult disorder
2. The helper subset of T lymphocytes is _____ in B. Leukocytopenia in the early stages
AIDS. C. Lymphocyte differential counts over 95%
A. increased D. Lymphoblasts on the peripheral blood smear
B. decreased 12. Which of the following are characteristics of CMV infec-
C. not altered tion?
A. Etiology: a herpes family virus
Questions 3 through 8: Match the following disorders B. Lymphocytopenia
with either A or B. C. A positive heterophil test result
A. Lymphocytosis D. Both A and B
B. Lymphocytopenia 13. AIDS is caused by
3. _____ Radiation exposure A. a herpes family virus
4. _____ Infectious mononucleosis B. CMV
5. _____ Cytotoxic drugs C. HIV-1
6. _____ Whooping cough D. EBV
7. _____ Immune deficiency disorders 14. Which of the following generally characterize(s) toxo-
8. _____ Toxoplasmosis plasmosis?
9. Which of the following characterizes infectious mono- A. Symptoms may resemble infectious mononucleosis
nucleosis? B. Occurrence in pregnant women who own cats
A. Etiology: EBV C. Etiology: parasitic
B. A T-cell disorder D. All of the above
C. A greater incidence in Africa 15. Which antibody test has replaced the LE cell preparation
D. Nonheterophil antibodies in the diagnosis of SLE?
10. The laboratory findings in infectious mononucleosis are A. Rheumatoid arthritis factor
generally characterized by B. ANA test
A. an increase in variant lymphocytes C. Complement fixation test
B. a heterophil titer less than 1:56 D. Antibody Smith test

BIBLIOGRAPHY Fleischer GR. Epstein-Barr virus. In: Belshe M (ed.). Textbook of


Human Virology, Littleton, MA: PSG Publishing, 1984:490–558.
Ascherio A, et al. Epstein-Barr virus antibodies and risk of multiple Hall SM. The diagnosis of toxoplasmosis, Br Med J, 289:570–571,
sclerosis: A prospective study, JAMA, 286(24):3083, 2001. 1984.
Bowden R, et al. Cytomegalovirus immune globulin and seronega- Kinney JS, et al. Cytomegaloviral infection and disease, J Infect Dis,
tive blood products to prevent primary cytomegalovirus infec- 151:772–774, 1985.
tion after marrow transplantation, N Engl J Med, 314:1006–1010, Lennette ET, Henle W. Epstein-Barr virus infections: Clinical and
1986. serologic features, Lab Manage, 25:23–28, 1987.
Brown KA. Nonmalignant disorders of lymphocytes, Clin Lab Sci, Mandell GE (ed.). Principles and Practices of Infectious Disease, 2nd ed,
10(6):329–335, 1997. New York, NY: Wiley, 1985.
Bruce-Chwatt LJ. Transfusion associated parasitic infections. In: Dodd Markell EK, et al. Medical Parasitology, 6th ed, Philadelphia, PA: Saun-
R (ed.). Infection, Immunity, and Blood Transfusion, New York, NY: ders, 1986:112–117, 131–138.
Wiley–Liss, 1985:101–125. Steinbrook R. The AIDS epidemic in 2004, N Engl J Med, 351(2):115–
Centers for Disease Control and Prevention. Mother-to-Child (Perin- 117, 2004.
atal) HIV Transmission and Prevention, Oct 2007 Retrieved www. World Health Organization United Nations Children’s Fund. HIV/
cdc.gov January 4, 2010. AIDS-2009. Retrieved January 10, 2010.

Turgeon_Chap17.indd 297 10/15/2010 9:54:23 AM


306 PART 3 ■ Leukocytes

blood. Lymphomas are similar to leukemias, but lymphomas forms is from several weeks to several months, compared
are solid tumors of lymph nodes and associated tissues or with the untreated chronic forms, which can have a prog-
bone marrow. nosis of survival ranging from months to many years after
The clinical symptoms, the maturity of the affected cells, diagnosis.
and the total leukocyte count determine whether a leuke- The commonly encountered types of leukemias are sepa-
mia will be classified as acute or chronic. Acute leukemias are rated into three broad groupings by cell type: myelogenous,
characterized as having symptoms of short duration, many monocytic, and lymphocytic.
immature cell forms in the bone marrow and/or peripheral Untreated leukemias and lymphomas are ultimately fatal.
blood, and an elevated total leukocyte count. Chronic leu- Specific treatment requires accurate diagnosis and classifica-
kemias have symptoms of long duration, mostly mature cell tion of the disorder by the clinical laboratory.
forms in the bone marrow and/or peripheral blood, and total The occurrence of leukemias and lymphomas has been
leukocyte counts that range from extremely elevated to less related to a variety of factors. These factors include genetic
than normal. The prognosis of survival in untreated acute conditions, environmental exposure, and viral agents.

REVIEW QUESTIONS

1. A definition of a leukemia could include C. chemical exposure to benzene


A. an overproduction of leukocytes D. all of the above
B. solid, malignant tumors of the lymph nodes 5. HIV is associated with
C. malignant cells trespass the blood-brain barrier A. hairy cell leukemia
D. both A and C B. Sézary cell syndrome
2. Descriptive terms for most lymphomas can include C. AIDS
A. a nonneoplastic proliferative disease D. leukemia
B. a solid malignant tumor of the lymph nodes 6. Cancer-predisposing genes may act by
C. a lymphocytopenia A. affecting the rate at which exogenous precarcinogens
D. freely trespassing the blood-brain barrier are metabolized to actively carcinogenic forms
3. An acute leukemia can be described as being B. affecting the host’s ability to repair resulting damage
A. of short duration with many mature leukocyte forms to DNA
in the peripheral blood C. altering the immune system’s ability to recognize
B. of short duration with many immature leukocyte and wipe out incipient tumors
forms in the peripheral blood D. all of the above
C. of short duration with little alteration of the leuko- 7. The incidence of leukemia is higher in
cytes of the peripheral blood A. Scandinavian versus Japanese populations
D. of long duration with many mature leukocyte forms B. American blacks versus American whites
in the peripheral blood C. chronic forms in children versus chronic forms in
4. The etiological agents of leukemias can include adults
A. ionizing radiation D. acute forms in older adults versus acute forms in
B. certain infectious agents children

BIBLIOGRAPHY USA and USSR. Bethesda, MD: National Cancer Institute,


1980:125–127.
Alford KA, et al. Perturbed hematopoiesis in the Tc1 mouse model of Neri G. Some questions on the significance of chromosome alterations
Down syndrome, Blood, 115(14), 2928-2937, 2010. in leukemias and lymphomas: A review, Am J Med Genet, 18:471–
Chan MP, Weissman IL, Park CY. Cancer stem cells: on the verge of 481, 1984.
clinical translation, Labmedicine, 39(11):679–686, 2008. Stevens W, et al. Leukemia in Utah and radioactive fallout from the
DeThe G, et al. Viruses as risk factors or causes of human leukemias Nevada test site: A case–control study, JAMA, 264(5):585–591,
and lymphomas? Leuk Res, 9(6):691–696, 1985. 1990.
Malinge S, Izraeli S, Crispino JD. Insights into the manifestations, out- Swerdlow SH, et al (ed.). WHO Classification of Tumours of
comes, and mechanisms of leukemogenesis in Down syndrome, Haematopoietic and Lymphoid Tissues, Lyon, France: IARC,
Blood, 113(12):2619–2628, 2009. 2008.
Miller RW. The features in common among persons at high risk Wing S, et al. Mortality among workers at Oak Ridge National Labora-
of leukemia. In: Levin DL (ed.). Cancer Epidemiology in the tory, JAMA, 265(11):1397–1408, 1991.

Turgeon_Chap18.indd 306 10/13/2010 7:31:52 PM


CHAPTER 19 ■ Acute Leukemias 333

CASE STUDIES (continued)

An additional PAS cytochemical stain may be valuable noted on the smear. The urinalysis was normal. The urine
because PAS-positive particles are frequently found in culture was negative.
lymphoblasts, sometimes in large quantities. This feature A follow-up bone marrow examination and serum uric
is important in differentiating between lymphoblasts and acid were ordered. The bone marrow revealed numerous
myeloblasts or monoblasts. immature, mononuclear cells. Subsequent partial immu-
3. Although meningeal leukemia is frequently encountered nophenotyping results were
in childhood ALL, remission rates for children are high.
Nuclear Tdt Positive
Approximately 90% of children achieve at least one
HLA-DR Positive
remission. According to current statistics, approximately
CD 10 (CALLA) Positive
50% of children with ALL will live at least 5 years.
CD 19 Positive
DIAGNOSIS: Acute Lymphoblastic (FAB L1) Leukemia CD 34 Positive
The follow-up serum uric acid was elevated.
CASE 6
■ Questions
A 12-year-old white girl was brought by her mother to see
the pediatrician. She was complaining of joint pain, jaw pain, 1. Based on the laboratory findings, what is the most likely
and tiredness. No bleeding was noted. She had no history of diagnosis in this case?
infectious disease exposure or recent immunizations. 2. What is the significance of the follow-up bone marrow
Physical examination revealed that the child had an examination and phenotyping?
enlarged liver and spleen. A CBC, urinalysis, and urine 3. What is the prognosis for a patient with this immuno-
culture were ordered. phenotyping?
■ Discussion
■ Laboratory Data
1. Based on the history, physical findings, and preliminary
RBC 2.61 × 1012/L laboratory findings, ALL is a strong consideration.
Hematocrit 25% 2. This patient was diagnosed with common (L1) ALL.
Hemoglobin 8.0 g/dL The presence of specific cluster designations (CDs) con-
WBC 28.0 × 109/L firmed the diagnosis.
Platelets 57 × 109/L 3. The prognosis is excellent. In fact, after initial treat-
■ Leukocyte Differential
ment that achieved remission in 4 weeks and follow-up
Segmented neutrophils 5% consolidation and maintenance therapy over a 2-year
Lymphocytes 5% period, the patient returned to good health. Three years
Mononuclear cells 90% after diagnosis, she was well and needed no further
Decreased platelets were noted on the peripheral blood therapy.
smear; 2+ anisocytosis and slight hypochromia also were DIAGNOSIS: Acute Lymphoblastic Leukemia

REVIEW QUESTIONS

1. Which of the following are typical characteristics of an Questions 2 through 5: Match the following types of acute
acute leukemia? leukemia with their FAB classifications.
A. Replacement of normal marrow elements by leuko- 2. _____ Myeloid and monocytic
cytic blasts and bleeding episodes 3. _____ Monocytic
B. Blasts and immature leukocyte forms in the periph- 4. _____ Myeloid without maturation
eral blood and anemia 5. _____ Lymphoblastic (one cell population)
C. Leukocytosis
D. All of the above

(continued)

Turgeon_Chap19.indd 333 10/15/2010 10:01:11 AM


334 PART 3 ■ Leukocytes

REVIEW QUESTIONS (continued )

A. M1 C. leukocytopenia
B. M4 D. a leukemia of older persons
C. M5
D. L1 Questions 18 through 20: Match one of the following
6. Characteristics of FAB M1 include
chromosomal alterations with the appropriate FAB type.
18. _____ t(15q+;17q−)
A. leukocytosis with maturation of the myeloid cell line
19. _____ t(8q−;21q+)
in the peripheral blood
20. _____ t(9;22)
B. leukocytosis with maturation of the lymphocytic cell
A. FAB M2
line in the peripheral blood
B. FAB M3
C. leukocytosis without maturation of the myeloid cell
C. FAB ALL
line in the peripheral blood
D. leukocytosis with many mature leukocytes in the pe-
Questions 21 through 23: Match one of the following
ripheral blood
cytochemical stains with the appropriate constituent.
7. The incidence of FAB M1 is 21. _____ Sudan black B
A. high in children younger than 18 months of age 22. _____ Myeloperoxidase
B. high in children between 1.5 and 12 years of age 23. _____ PAS
C. high in middle-aged adults A. Glycogen
D. both A and C B. Enzymes
C. Lipids
Questions 8 through 12: Match the following predomi- 24. The Sudan black B cytochemical stain differentiates
nant peripheral blood cell morphological appearances A. acute myeloid from ALL
with the FAB classifications. B. acute monocytic from AML
8. _____ A mixture of myeloid and monocytic blasts C. myeloid leukemia from a leukemoid reaction
9. _____ Blasts of the monocytic type D. acute myeloid from acute myelomonocytic
10. _____ Many coarsely granular promyelocytes with leukemia
dumbbell-shaped or bilobed nuclei 25. Myeloperoxidase differentiates
11. _____ Myeloblasts, promyelocytes, and myelocytes A. acute myeloid from chronic myelocytic leukemia
12. _____ Immature leukocytic and erythrocytic cell B. acute myeloid and acute monocytic from ALL
types C. acute myelomonocytic from acute monocytic
A. FAB M2 leukemia
B. FAB M3 D. acute lymphoblastic from acute monocytic leukemia
C. FAB M4 26. The PAS reaction is
D. FAB M6 A. positive in the neutrophilic granulocytes, except
E. FAB M5 blasts
B. positive in a block-like pattern in some lympho-
Questions 13 through 15: Match the FAB classifications blasts
with the correct descriptive term. C. negative in megakaryoblasts
13. _____ Leukemia secondary to Burkitt lymphoma D. negative in myelocytes
14. _____ Childhood lymphoblastic leukemia 27. Esterase (naphthol AS-D chloracetate) differentiates
15. _____ Older children and adults A. granulocytic (promyelocytic to segmented neutro-
A. FAB M1 phils) from the monocytic cell line
B. FAB L1 B. promyelocytes from myelocytes
C. FAB L2 C. monoblasts from myeloblasts
D. FAB L3 D. metamyelocytes from myelocytes
16. Chloromas are associated with 28. In the nonspecific esterase (alpha-naphthyl acetate)
A. FAB M1 staining reaction, the cells of monocytic origin are
B. FAB M3 A. strongly positive
C. FAB M4 B. positive initially and positive after sodium fluoride
D. FAB M5 incubation
17. A common characteristic of ALL is C. positive initially and negative after sodium fluoride
A. bone and joint pain incubation
B. many blast cells with Auer rods D. negative

(continued)

Turgeon_Chap19.indd 334 10/15/2010 10:01:11 AM


CHAPTER 19 ■ Acute Leukemias 335

REVIEW QUESTIONS (continued )

29. Specific nature B-cell surface marker(s) membrane is 30. Patients with AML have a good prognosis if
A. CD 79a A. Less than 45 years of age
B. CD19 B. Over rods are present in blast cells
C. CD 20 C. Ph chromosome
D. all of the above D. all of the above

BIBLIOGRAPHY Gupta P. Granulocyte colony-stimulating factor in children with acute


lymphoblastic leukemia, N Engl J Med, 337(18):1320, 1997.
Bacher U, et al. A comparative study of molecular mutations in 381 Hassane DC, et al. Discovery of agents that eradicate leukemia stem
patients with myelodysplastic syndrome and in 4130 patients with cells using an in silico screen of public gene expression data, Blood,
acute myeloid leukemia, Haematologica, 92(6):744–752, 2007. 111:5654–5662, 2008.
Bell A, et al. Use of cytochemistry and FAB classification in leukemia and Hershey DW. Detection of minimal residual diseases in childhood
other pathological states, Am J Med Technol, 47(6):437–470, 1981. leukemia with the polymerase chain reaction, N Engl J Med,
Bonci D, et al. Blocking the APRIL circuit enhances acute myeloid leu- 324(11):772–773, 1991.
kemia cell chemosensitivity, Haematologica, 93(12):1899–1902. Hess, CE. Acute Lymphoblastic Leukemia (ALL). www.healthsystem.
Cascavilla N, et al. Philadelphia chromosome and BCR/ABL positive virginia.edu. Retrieved Nov 30, 2008.
early T acute lymphoblastic leukemia (ALL) preceding B-lineage Iacobucci I, et al. Identification of different Ikaros cDNA transcripts
(CD10+) ALL, Hematol J, 4(Suppl 2):119, 2003. in Philadelphia-positive adult lymphoblastic leukemia by a high-
Cazzaniga G, Biondi A. Molecular monitoring of childhood acute lym- throughput capillary electrophoresis sizing method, Haematolog-
phoblastic leukemia using antigen receptor gene rearrangements ica, 93(12):1814–1821, 2008.
and quantitative polymerase chain reaction technology, Haemato- Jaffe ES, et al. Tumours of Haematopoietic Tissues, Lyon, France: IARC,
logica, 90(3):382–390, 2005. 2001.
Chalandon U, Schwaller J. Targeting mutated protein typrosine Jeha S, et al. Prognostic significance of CD20 expression in child-
kinases and their signaling pathways in hematologic malignancies, hood B-cell precursor acute lymphoblastic leukemia, Blood,
Haematologica, 90(7):949–968, 2005. 108(10):3302–3304, 2006.
Cochran DL. Unique features of acute promyelocytic leukemia, Clin Korycka AK, Robak T. The interaction of new purine nucleoside ana-
Lab Sci, 10(6):315–319, 1997. logues with signal transduction inhibitors (ST1571 and R115777) on
Cox CV, et al. Characterization of a progenitor cell population in myeloid progenitor cells in vitro, Hematol J, 4(Suppl 2):20, 2003.
childhood T-cell acute lymphoblastic leukemia. Blood, 109(2):674– Kroschinsky FP, et al. Cup-like acute myeloid leukemia: new disease or
682, 2007. artifical phenomenon? Haematologica, 93(2), 283–286, 2008.
de Vries ACH, et al. Role of mutation independent constitutive acti- Li Z, et al. Gene expression-based classification and regulatory networks
vation of FLT 3 in juvenile myelomonocytic leukemia, Haemato- of pediatric acute lymphoblastic leukemia, Blood, 114(20):4486–
logica, 92(11):1557–1560, 2007. 4493, 2009.
DeThe G, et al. Viruses as risk factors or causes of human leukemias Lo-Coco F, et al. Prognostic impact of genetic characterization in the
and lymphomas? Leuk Res, 9(6):691–696, 1985. GIMEMA LAM99P multicenter study for newly diagnosed acute
Döhner K, Döhner H. Molecular characterization of acute myeloid myeloid leukemia, Hematologica, 93(7):1017–1024, 2008.
leukemia, Haematologica, 93(7):976, 2008. Lowenberg B. Post-remission treatment of acute myeloid leukemia,
Flotho C, et al. A set of genes that regulate cell proliferation predicts N Engl J Med, 332(4):260–262, 1995.
treatment outcome in childhood acute lymphoblastic leukemia, Lowenberg B, et al. Acute myeloid leukemia, N Engl J Med,
Blood, 110(4):1271–1277, 2007. 341(14):1051–1066, 1999.
Gallo R. Human T-cell leukemia (lymphotropic) retroviruses and Malfuson J, et al. Risk factors and decision criteria for intensive che-
their causative role in T-cell malignancies and acquired immune motherapy in older patients with acute myeloid leukemia, Haema-
deficiency syndrome, Cancer, 55(10):2317–2323, 1985. tologica, 93(12):1806–1813, 2008.
Gallo RC. The human T cell leukemia/lymphotropic retroviruses Mead AJ, et al. FLT3 tyrosine kinase domain mutations are biologi-
(HTLV) family: Past, present and future, Can Res Suppl, 45:4524– cally distinct from and have a significantly more favorable prog-
4533, 1985. nosis than FLT3 internal tanem duplications in patients with acute
Ghaith F, et al. Molecular genetic detection of P53, T(1:19), MLL gene myeloid leukemia, Blood, 110:1262–1270, 2007.
rearrangement and cyclin D1 gene expression in acute lymphoblas- Mittal P, Meehan KR. The acute leukemias, Hosp Phys, 37(5):37–44,
tic leukemia, Hematol J, 4(Suppl 2):115, 2003. 2001.
Griffin JD. Surface marker analysis of acute myeloblastic leukemia. Mulligan CG, et al. BCR-ABL 1 lymphoblastic leukemia is character-
In: Bloomfield CD (ed.). Chronic and Acute Leukemias in Adults, ized by the deletion of Ikaros, Nature, 453:110–114, 2008.
Boston, MA: Martinus Nijhoff, 1985:113–137.

Turgeon_Chap19.indd 335 10/15/2010 10:01:11 AM


358 PART 3 ■ Leukocytes

REVIEW QUESTIONS

1. The most common form of chronic leukemia in West- C. is a lymphoma, characterized by Reed-Sternberg
ern countries is cells, and occurs more frequently in females than in
A. myelogenous males
B. lymphocytic D. is a lymphoma, characterized by Reed-Sternberg
C. monocytic cells, and occurs more frequently in males than in
D. eosinophilic females
2. The median survival time of patients with CLL, compared 7. Rare forms of lymphoma include
with patients with chronic monocytic leukemia, is A. Hodgkin and non-Hodgkin lymphoma
A. not significantly different B. Burkitt lymphoma and mycosis fungoides
B. shorter C. Hodgkin and non-Hodgkin lymphoma and Burkitt
C. longer lymphoma
D. shorter, if the patient is female D. Non-Hodgkin lymphoma and mycosis fungoides
3. CLL is classically a 8. Multiple myeloma is a disorder of
A. T-cell disorder A. T lymphocytes
B. B-cell disorder B. megakaryocytes
C. null cell disorder C. plasma cells
D. disorder of the young D. the lymph nodes
4. CLL symptoms frequently include 9. The abnormal protein frequently found in the urine of
A. weight loss, anemia, and extreme leukocytosis persons with multiple myeloma is
B. absolute lymphocytosis, edema, and splenic infarction A. albumin
C. absolute lymphocytosis, malaise, and low-grade B. globulin
fever C. IgG
D. neutrophilia, splenomegaly, and anemia D. Bence Jones
5. Characteristics of malignant lymphoma typically 10. WM is characterized by increased levels of
include A. IgG
A. overproliferation of neutrophils B. IgM
B. overproliferation of lymphocytes C. IgD
C. lymph node involvement D. IgA
D. both B and C 11. Which cluster designations are positive in typical HCL?
6. Hodgkin disease A. CD25 CD11c, CD19, CD20
A. is characterized by neutrophilia in the early stages of B. CD25 CD11c, CD19, CD10
the disease C. CD25 CD11c, CD10, CD5
B. occurs more frequently in females than males D. CD25 CD22, CD19, CD20

BIBLIOGRAPHY Burger JA. Fledgling prognostic markers in CLL, Blood, 110(12):3820–


3821, 2007.
Attal M, et al. Maintenance therapy with thalidomide improves sur- Caligaris-Cappio F. IG genes and hairy cell leukemia, Blood,
vival in patients with multiple myeloma, Blood, 108(10):3289–3294, 114(21):4610, 2009.
2006. Calin GA, Croce CM. Chronic lymphocytic leukemia: interplay
Awan FT, et al. Mcl-1 expression predicts progression-free survival in between noncoding RNAs and protein-coding genes, Blood,
chronic lymphocytic leukemia patients treated with pentostatin, 114(23):4761–4770, 2009.
cyclophosphamide, and rituximab, Blood, 113(3):535–537, 2009. Calin GA, et al. A microRNA signature associated with prognosis
Ayliffe MJ, et al. Demonstration of changes in plasma cell subsets in and progression in chronic lymphocytic leukenia, N Eng J Med,
multiple myeloma, Haematologica, 92(08):1135–1138, 2007. 353(17):1793–1801, 2005.
Baskar S, et al. A human monoclonal antibody drug and target dis- Cazin B. First line treatment of CLL: New approaches. In: New Treat-
covery platform for B-cell chronic lymphocytic leukemia based on ment Approaches for Chronic Lymphoproliferative Disorders and Lym-
allogeneic hematopoietic stem cell transplantation and phage dis- phoma. Satellite symposium presentation at the 8th Congress of the
play, Blood, 114(20):4494–4502, 2009. European Hematology Association, Lyon, France June 12, 2003.
Bench AJ, et al. Molecular genetic analysis of haematological malignan- Child JA, et al. High-dose chemotherapy with hematopoietic stem-cell
cies II: mature lymphoid neoplasms, Int J Lab Hematol, 29(4):229– rescue for multiple myeloma, N Engl J Med, 348(19):1875–1883,
260, 2007. 2003.
Binet J-L, et al. Perspectives on the use of new diagnostic tools in the Coifier B, et al. CHOP chemotherapy plus rituximab compared with
treatment of chronic lymphocytic leukemia, Blood, 107(3):859– CHOP alone in elderly patients with diffuse large-B-cell lymphoma,
861, 2006. N Engl J Med, 346(4):235–241, 2002.

Turgeon_Chap20.indd 358 10/15/2010 10:06:50 AM


382 PART 4 ■ Myeloproliferative Neoplasms

CASE STUDIES (continued)

■ Discussion 3. No. Reactive thrombocytosis is associated with acute or


1. The significantly increased platelet count and slightly chronic inflammatory conditions, acute hemorrhage, and
decreased red blood cell parameters are an indication a variety of other conditions. The patient had no medical
of a probable hematological problem. Follow-up bone history suggestive of these causes of his thrombocytosis.
marrow and platelet aggregation studies were ordered. The diagnosis of essential (primary) thrombocythemia is
The follow-up bone marrow exhibited a hypercellular mar- one of exclusion.
row. The Prussian blue iron stain demonstrated normal
marrow iron stores. The platelet aggregation studies DIAGNOSIS: Essential (primary) Thrombocythemia
were grossly abnormal.
2. In contrast to PV, this patient has normal bone marrow
iron stores.

REVIEW QUESTIONS

1. MPNs are characterized by all of the following except C. decreased APTT, decreased factor V level, and
A. clonal disorders increased concentration of D-dimers
B. they may evolve into acute leukemia D. decreased concentration of D-dimers, decreased
C. initial increase of immature cells concentration of antithrombin III, and increased
D. increased production of mature cells concentration of plasmin-alpha 2-plasmin inhibitor
2. In CML, the total leukocyte (WBC) count is complex
A. extremely increased 7. Interferon alfa has been shown to
B. slightly increased A. stimulate trilineage cell proliferation
C. extremely variable B. suppress proliferation of hematopoietic progenitor
D. usually normal cells
3. Primary myelofibrosis differs from other types of MPN C. subdue erythropoiesis only
in which of the following ways? D. suppress megakaryocytopoiesis only
A. Ph1 chromosome is present 8. A leukemia of long duration that affects the neutrophilic
B. Marrow fibrosis is greatly increased granulocytes is referred to as
C. LAP score is increased A. acute lymphoblastic leukemia
D. Platelet count is increased B. acute myelogenous leukemia
4. Which of the following is a remarkable characteristic C. acute monocytic leukemia
of PV compared with other types of MPNs? D. CML
A. Extremely increased erythrocyte mass 9. The alkaline phosphatase cytochemical staining reaction
B. Extremely increased leukocyte count is used to differentiate between
C. Extremely increased platelet count A. chronic lymphoblastic leukemia and acute myelog-
D. Teardrop-shaped erythrocytes enous leukemia
5. Which of the following is a predominant feature of B. acute lymphoblastic leukemia and acute myelog-
essential thrombocythemia compared with other types enous leukemia
of MPNs? C. CML and severe bacterial infections
A. Variable number of platelets D. leukemoid reactions and severe bacterial
B. Moderately increased number of platelets infections
C. Extremely increased number of platelets 10. Patients with the initial phase of CML are prone to
D. Increased marrow fibrosis A. weight gain, edema, and fatigue
6. In MPN, the test results of disorders of hemostasis and B. edema, anemia, and splenic infarction
coagulation that are most likely to be abnormal are C. low-grade fevers, night sweats, and splenic
A. decreased platelet count, increased APTT, and infarction
increased factor V level D. prominent lymphadenopathy and night sweats
B. increased APTT, decreased factor V level, and
increased concentration of antithrombin III in
many

(continued)

Turgeon_Chap21.indd 382 10/13/2010 7:42:49 PM


CHAPTER 21 ■ Myeloproliferative Neoplasms 383

REVIEW QUESTIONS (continued)

11. The total leukocyte count in CML usually is _____ × 20. Treated patients with PV have a _____ life expectancy
109/L. after diagnosis.
A. normal A. 1- to 6-month
B. <25 B. 6- to 12-month
C. <50 C. 1- to 5-year
D. >50 D. more than 10-year
12. The Philadelphia chromosome is typically associated 21. The primary treatment for PV is
with A. therapeutic phlebotomy
A. acute myelogenous leukemia B. myelosuppressive agents
B. leukemoid reactions C. radioactive phosphorus
C. acute lymphoblastic leukemia D. low-dose busulfan
D. CML 22. Primary myelofibrosis is also called
13. Patients with PV suffer from A. essential thrombocythemia
A. leukemic infiltration B. CML
B. bone marrow fibrosis C. PV
C. hypervolemia D. agnogenic myeloid metaplasia
D. anemia 23. The incidence of primary myelofibrosis is known to
14. In PV, cytogenetic results do not predict/provide increase after exposure to
A. duration of the disease A. sunshine
B. consequences of myelosuppressive therapy B. benzene
C. clues to hematological phenotype C. antibiotics
D. evolution of the disease D. interferon
15. Hyperviscosity can produce 24. The predominant clinical manifestation of primary
A. anemia myelofibrosis is
B. dizziness A. anemia
C. hemorrhages B. splenomegaly
D. psychotic depression C. medullary fibrosis
16. The major criteria for diagnosis of PV include all of the D. all of the above
following except 25. The most constant feature of primary myelofibrosis is
A. increased red blood cell mass A. dyserythropoiesis
B. presence of JAK2V617F B. dysleukopoiesis
C. hypercellular bone marrow C. dysmegakaryocytopoiesis
D. splenomegaly D. trilineage maturational disruption
17. Increased blood viscosity in patients with PV can cause 26. A leukoerythroblastic picture includes all of the following
a dangerous condition of except
A. hot flushes A. teardrop-shaped erythrocytes
B. shortness of breath B. nucleated erythrocytes
C. high RDW C. immature lymphocytes
D. vascular occlusion D. immature myeloid cells
18. The level of erythropoietin in the urine is _____ 27. The median survival time for patients with primary
in patients with PV compared with other kinds of myelofibrosis is approximately _____ year(s).
polycythemia. A. 1
A. increased B. 3
B. the same C. 5
C. variable D. 10
D. decreased 28. The least common form of MPN is
19. Patients with PV demonstrate a(n) _____ of hemosid- A. PV
erin in the bone marrow. B. CML
A. absence C. primary myelofibrosis
B. normal amount D. essential thrombocythemia
C. slightly increased amount
D. extremely increased amount

(continued)

Turgeon_Chap21.indd 383 10/13/2010 7:42:49 PM


384 PART 4 ■ Myeloproliferative Neoplasms

REVIEW QUESTIONS (continued)

29. A major criterion for the diagnosis of essential 31. The bone marrow architecture in essential thrombo-
thrombocythemia is cythemia is similar to the architecture seen in
A. absence of Ph1 chromosome A. erythroid hyperplasia
B. increased red blood cell mass B. leukocyte hyperplasia
C. mild neutrophilia in peripheral blood C. CML
D. persistent increase of platelets in peripheral blood D. lymphocytic leukemia
30. The most common disorder in patients with essential
thrombocythemia is
A. neurological manifestations
B. thrombotic or bleeding problems
C. abnormal karyotype
D. anemia

BIBLIOGRAPHY Hehlmann R, Saussele S. Treatment of chronic myeloid leukemia in


blast crisis, Haematologica, 93(12):1765–1768, 2008.
Apperly J. Allografting for chronic myeloid leukemia in the Glivec era, Hochhaus A. Molecular response and resistance to Imatinib (Glivec®).
Hemato J, 4 Suppl 3. In: Educational Book of the 8th Congress of the In: Education Book of the 8th EHA Congress, Lyon, France:
European Hematology Association. Lyon: France, 2003:11–14. 2003:15–20.
Bowers GR (ed.). Polycythemia rubra vera, CDH Oncol, 7(7):1–6, Hochhaus A. Perspectives on managing resistance in CML. In:
2003. Imatinib—Impact on CML Management and Beyond, 8th EHA
Campbell PJ, Green AR. The myeloproliferative disorders, N Eng J Congress, Lyon, France: 2003.
Med, 355(23):2452–2466, 2006. Kantarjian HM, et al. Long-term survival benefit and improved com-
Clark M. Chronic myelomonocytic leukemia transforming into acute plete cytogenetic and molecular response rates with imatinib mesy-
myelogenous leukemia, Lab Med, 40(1):19–21, 2009. late in Philadelphia chromosome-positive chronic-phase chronic
Conley CL. Polycythemia vera, JAMA, 263(18):2481–2483, 1990. myeloid leukemia after failure of interferon-a, Blood, 104(7):
Crisan D, Carr ER. BCR/abl gene rearrangement in chronic myelog- 1979–1988, 2006.
enous leukemia and acute leukemias, Lab Med, 23(11):730–735, Kantarjian H, et al. Nilotinib in imatinib-resistant CML and
1992. Philadelphia chromosome-positive ALL, N Eng J Med, 354(24):
Drucker BJ. Translation of the Philadelphia chromosome into therapy 2542–2551, 2006.
for CML, Blood, 112(13):4808–4817, 2008. Kralovics R, et al. A gain-of-function mutation of JAK2 in myelopro-
Drucker BJ, et al. Five year follow-up of patients receiving imatinib for liferative disorders, N Eng J Med, 352(17):1779–1790, 2005.
chronic myeloid leukemia, N Engl J Med, 355(23):2408–2417, 2006. Klippel S, et al. Biochemical characterization of PV-1, a novel
Drucker BJ, et al. Efficacy and safety of a specific inhibitor of the BCR- hematopoietic cell surface receptor, which is overexpressed in
ABL tyrosine kinase in chronic myeloid leukemia, N Engl J Med, polycythemia rubra vera, Blood, 100(7):2441–2448, 2002.
344(14): 1031–1037, 2001. Lee SJ, et al. impact of prior imatinib mesylate on the outcome of
Fabarius A, et al. Dynamics of cytogenetic aberrations in Philadelphia hematopoietic cell transplantation for chronic myeloid leukemia,
chromosome positive and negative hematopoiesis during dasatinib Blood, 112(8):3500–3507, 2008.
therapy of chronic myeloid leukemia patients after imatinib failure, Levine RL, Gilliland G. Myeloproliferative disorders, Blood, 112(6):
Haematologica, 92(06):834–837, 2007. 2190–2198, 2008.
Faderl S, et al. The biology of chronic myeloid leukemia, N Engl J Med, Martinelli G, et al. New tyrosine kinase inhibitors in chronic myeloid
341(3):164–172, 1999. leukemia, Haematologica, (90)4:534–541, 2005.
Fischer T. Beyond CML—New horizons for Imatinib. In: Imatinib— Marsh GM, et al. Mortality patterns among petroleum refinery and
Impact on CML Management and Beyond, 8th EHA Congress, Lyon, chemical plant workers, Am J Ind Med, 19(1):29–42, 1991.
France: 2003. Mertens F, et al. Karyotypic patterns in chronic myeloproliferative dis-
Gilliland DG, et al. Clonality in myeloproliferative disorders: Analysis orders: Report on 74 cases and review of the literature, Leukemia,
by means of the polymerase chain reaction, Proc Natl Acad Sci USA, 5(3):214–220, 1991.
88(15):6848–6852, 1991. Mumprecht S, et al. Imatinib mesylate selectively impairs expansion of
Goldman J. Novel treatment approaches. In: The Educational Book of memory cytotoxic T cells without affecting the control of primary
the 8th EHA Congress, Lyon, France: 2003:21–24. viral infections, Blood, 108(10):3406–3413, 2006.
Goldman JM. Practical considerations to optimize outcome for O’Brien SG, et al. Imatinib compared with interferon and low-dose
patients with CML in chronic phase. In: Imatinib—Impact on CML cytarabine for newly diagnosed chronic-phase chronic myeloid
Management and Beyond, 8th EHA Congress, Lyon, France: 2003. leukemia, N Engl J Med, 348(11):994–1004, 2003.
Guilhot F, et al. Interferon alfa-2b combined with cytarabine versus Palandri F, et al. Chronic myeloid leukemia in blast crisis treated
interferon alone in chronic myelogenous leukemia, N Engl J Med, with imatinib 600 mg: outcome of the patients alive after a 6-year
337(4):223–228, 1997. follow-up, Haematologica, 93(12):1792, 2008.

Turgeon_Chap21.indd 384 10/13/2010 7:42:49 PM


CHAPTER 22 ■ Myelodysplastic Syndromes and Myelodysplastic/Myeloproliferative Neoplasms 397

CASE STUDIES (continued)

■ Discussion
1. With the presence of an increased MCV, it is unlikely that
this patient has iron deficiency anemia. An elevated MCV
could be caused by liver damage (e.g., alcohol abuse) or a
vitamin deficiency. His history of colitis could be the cause.
2. A hypercellular bone marrow would rule out aplastic
anemia as the cause of the patient’s anemia. In addition,
the presence of iron in the bone marrow further con-
firms the absence of iron deficiency anemia.
3. The medications that are being taken by the patient are
known to have the potential to cause bone marrow sup-
pression. No other evidence supports a virally induced
(Reprinted with permission from Anderson SC, Anderson’s, aplastic anemia or the presence of leukemia.
Atlas of Hematology, Philadelphia, PA: Wolters Kluwer Health,
Lippincott Williams & Wilkins, 2003, Copyright 2003.) DIAGNOSIS: MDS

REVIEW QUESTIONS

1. Patients with some variety of MDS are at increased risk 6. The incidence of chromosomal abnormality in adults
of developing with MDS is
A. acute lymphoblastic leukemia A. 5% to 15%
B. AML B. 15% to 25%
C. chronic lymphocytic leukemia C. 25% to 60%
D. chronic myelogenous leukemia D. 40% to 90%
2. Which of the following agents has not been supported 7. The karyotype associated with a high probability of
by scientific research as being associated with the devel- transforming to AML is
opment of secondary MDS? A. monosomy 5
A. Alkylating agents B. monosomy 7
B. Organic solvents C. trisomy 11
C. Insecticides D. both A and B
D. Both B and C 8. Patients with MDSs commonly suffer from _____ initially.
3. An increased incidence of MDSs is seen in A. a rash
A. males younger than 55 years of age B. anemia
B. females younger than 55 years of age C. visual disturbances
C. males older than 55 years of age D. vertigo
D. females older than 55 years of age
Questions 9 through 11: Match the type of myelodysplastic syndrome
4. The most frequently involved chromosomes in adults
with the appropriate description.
with MDS are
9. Refractory anemia (RA)
A. 1, 5, and 7
10. Refractory anemia with ring sideroblasts (RARS)
B. 3, 5, and 8
11. Refractory anemia with excess blasts, type 1 (RAEB-1)
C. 5, 7, and 8
A. anemia, no blasts
D. 8, 12, and 13
B. anemia, less than 1% blasts
5. The most frequent chromosomal abnormalities
C. cytopenia(s), less than 5% blasts, no Auer rods
in children with MDS include all of the following
12. In young patients, the therapy of choice for MDSs
except
involves
A. trisomy 8
A. vitamins
B. monosomy 7
B. allogeneic bone marrow transplantation
C. deletion of long arm of chromosome 20
C. cytotoxic drugs
D. all of the above
D. colony-stimulating growth factors

Turgeon_Chap22.indd 397 10/26/2010 12:19:02 PM


426 PART 5 ■ Principles and Disorders of Hemostasis and Thrombosis

process is the conversion of circulating insoluble coagulation reduces the chance of local fibrin formation. Another normal
factors into a gelatinous fibrin clot with entrapped blood mechanism against inappropriate thrombosis is the removal
cells, a blood clot. As repair of damaged tissue takes place, from the blood of activated clotting factors by hepatocytes.
the clot is lysed and the particulate matter is removed by the This process, along with the naturally occurring inhibitors,
mononuclear phagocytic system. limits intravascular clotting and fibrinolysis by inactivation
The intrinsic and extrinsic pathways are now thought to of such factors as XIa, IXa, Xa, and IIa. Removal of particulate
function in an interrelated manner in vivo, and previously material by the cells of the mononuclear phagocytic system is
established in vitro methods are valid to screen for abnor- also important in preventing the initiation of coagulation.
malities. A variety of laboratory procedures are valuable in The in vivo existence of natural anticoagulant systems is
assessing coagulation factors. General procedures include the essential to prevent thrombosis. These natural anticoagulant
aPTT, the PT, the thrombin time, and quantitative fibrinogen systems include AT-III, HC-II, and protein C and its cofactor,
concentration assay. More specialized or classic procedures protein S. AT-III and HC-II are serine protease inhibitors.
can be performed in special circumstances. When activated, protein C is capable of degrading activated
factors V (Va) and VIII (VIIIa) in the presence of the cofactor
Normal Protective Mechanisms Against protein S.
Thrombosis
Modern View of Hemostasis
In the blood circulation, the predisposition to thrombosis
depends on the balance between procoagulant and anti- In addition to other discoveries, thrombin has been found to
coagulant factors. The normal flow of blood prevents the have a far more important role as an activator and inhibitor
accumulation of procoagulant material. This mechanism than previously thought.

REVIEW QUESTIONS

1. Normal hemostasis depends on all of the following C. veins


except D. arteries
A. an intact vascular system 10. The initiating stimulus to blood coagulation following
B. inadequate numbers of platelets injury to a blood vessel is
C. appropriate coagulation factors A. contact activation with collagen
D. fibrinolysis B. vasoconstriction
C. stenosis
Questions 2 through 6: The sequence of events following D. release of serotonin
injury to a small blood vessel is (2) _____ to (3) _____ to (4) 11. Endothelium is involved in the metabolism and clear-
_____ to (5) _____ to (6) _____. ance of molecules such as
A. Contact between damaged blood vessel, blood plate- A. serotonin
lets, and coagulation proteins B. angiotensin
B. Formation of a platelet plug C. bradykinin
C. Fibrinolysis and reestablishment of vascular integ- D. all of the above
rity 12. Which of the following is not correct?
D. Development of a blood clot around the injury A. Vasoconstriction reduces blood flow and promotes
E. Blood vessel spasm (vasoconstriction) contact activation of platelets and coagulation factors
7. Which blood vessels have the thickest walls? B. Platelets adhere to exposed endothelial connective
A. Veins tissues
B. Arteries C. Aggregation of platelets releases thromboxane A2
C. Capillaries and vasoactive amines (serotonin and epinephrine)
D. Arterioles D. None of the above
8. All blood and lymphatic vessels are lined with 13. Which of the following is (are) true of endoreduplica-
A. endothelium tion?
B. nerve endings A. Duplicates DNA without cell division
C. stratified epithelial cells B. Results in cells with ploidy values of 4n, 8n, 16n,
D. simple squamous epithelium and 32n
9. Blood passes from the arterial to the venous system via C. Is unique to the megakaryocytic type of blood cell
A. arterioles D. All of the above
B. capillaries 14. Which of the following is (are) true of thrombopoietin?

(continued)

Turgeon_Chap23.indd 426 10/15/2010 10:13:46 AM


CHAPTER 23 ■ Principles of Hemostasis and Thrombosis 427

REVIEW QUESTIONS (continued)

A. Thought to stimulate the production and matura- 23. Agents that are capable of aggregating platelets include
tion of megakaryocytes A. collagen
B. Is influenced by various cytokines, which increase B. thrombin
megakaryocyte size C. serotonin
C. Is influenced by various cytokines, which impact D. all of the above
maturational stage and ploidy 24. Examination of a Wright-stained peripheral blood
D. All of the above smear provides an estimate of platelet numbers. Using
15. Which of the following is not a characteristic of 100× (oil) immersion in the areas of erythrocytes just
platelets? touching each other, the upper limit of the number of
A. The presence of a nucleus platelets seen per field should not exceed
B. Size of 2 to 4 mm A. 10 to 15
C. Cytoplasm is light blue with fine red-purple gran- B. 15 to 20
ules C. 20 to 25
D. A discoid shape as an inactive cell D. 25 to 30
16. The cellular ultrastructural component(s) unique to the 25. If 10 platelets are seen per oil immersion field, what is
platelet is (are) the approximate platelet count?
A. Cytoplasmic membrane A. 50 × 109/L
B. Glycocalyx B. 100 × 109/L
C. Mitochondria C. 150 × 109/L
D. Microtubules D. 200 × 109/L
17. Choose the incorrect statement regarding storage gran- 26. Aspirin ingestion has the following hemostatic effect in
ules related to hemostasis in the mature platelet. a normal person.
A. Alpha-granules contain platelet factor 4, beta-throm- A. Prolongs the bleeding time
boglobulin, and platelet-derived growth factor B. Prolongs the clotting time
B. Alpha-granules contain platelet fibrinogen and von C. Inhibits factor VIII
Willebrand factor D. Has no effect
C. Dense bodies contain serotonin and ADP 27. The bleeding time test measures
D. Lysosomes contain actomyosin, myosin, and fil- A. the ability of platelets to stick together
amin B. platelet adhesion and aggregation on locally injured
18. At all times, approximately _____ of the total number of vascular subendothelium
platelets are in the systemic circulation. C. the quantity and quality of platelets
A. one fourth D. antibodies against platelets
B. one third 28. The clot retraction test is
C. one half A. a visible reaction to the activation of platelet acto-
D. two thirds myosin (thrombosthenin)
19. The reference range of platelets in the systemic circulation is B. a reflection of the quantity and quality of platelets
A. 50 to 150 × 109/L and other factors
B. 100 to 200 × 109/L C. a measurement of the ability of platelets to stick to
C. 150 to 350 × 109/L glass
D. 150 to 450 × 109/L D. a measurement of the cloudiness of blood
20. The functions of platelets in response to vascular dam-
age include Questions 29 through 31: Match the following.
A. maintenance of vascular integrity by sealing minor 29. _____ Fibrinogen group
defects of the endothelium 30. _____ Prothrombin group
B. formation of a platelet plug 31. _____ Contact group
C. promotion of fibrinolysis A. Factors II, VII, IX, and X
D. all of the above B. Factors I, V, VIII, and XIII
C. Factors XI, XII, prekallikrein, and high–molecular-
Questions 21 and 22: If vascular injury exposes the weight kininogen
endothelial surface and underlying collagen, platelets 32. The fibrinogen group of coagulation factors is
(21) _____ to the collagen fibers and (22) _____. A. known to increase during pregnancy
A. adhere B. known to increase in conditions of inflammation
B. aggregate

(continued)

Turgeon_Chap23.indd 427 10/15/2010 10:13:47 AM


428 PART 5 ■ Principles and Disorders of Hemostasis and Thrombosis

REVIEW QUESTIONS (continued)

C. known to increase subsequent to the use of oral 46. Prothrombin to thrombin conversion is accelerated by
contraceptives A. a complex of activated factors IX and VII
D. all of the above B. factor V and ionized calcium
33. The prothrombin group of coagulation factors is C. a complex of phospholipids and factor VII
A. dependent on vitamin K for production D. a complex of activated factors X and V
B. considered to be stable 47. Fibrinogen is converted to fibrin monomers by
C. well preserved in stored plasma A. prothrombin
D. all of the above B. thrombin
34. Warfarin acts by C. calcium ions
A. neutralizing the effects of thrombin D. factor XIIIa
B. interfering with fibrin monomer formation 48. The inactive plasminogen is activated to _____ by
C. acting as a vitamin K antagonist proteolytic enzymes.
D. inducing hypercoagulation A. prothrombin
B. plasmin
Questions 35 through 38: Match the name of the coagula- C. plasma kallikrein
tion factor with the appropriate symbolic designation. D. plasma thromboplastin antecedent
35. _____ Thrombin 49. Which of the following statements are true of the fibrin-
36. _____ Tissue thromboplastin olytic system?
37. _____ Antihemophilic factor A. Plasmin digests fibrin and fibrinogen
38. _____ Hageman factor B. The active enzyme of the system is plasmin
A. III C. Inactive plasminogen circulates in the plasma until
B. XII an injury occurs
C. VIII D. All of the above
D. IIa 50. If a pediatric preoperative patient has a family history
of bleeding but has never had a bleeding episode herself,
Questions 39 through 42: Arrange the four stages of coag- what test should be included in a coagulation profile in
ulation in their proper sequence. addition to the PT, aPTT, and platelet count?
39. _____ A. Lee-White clotting time
40. _____ B. Clot retraction
41. _____ C. Bleeding time
42. _____ D. Fibrin split products
A. Fibrinolysis 51. A patient with a severe decrease in factor X activity
B. Formation of thrombin from prothrombin would demonstrate normal
C. Generation of plasma thromboplastin A. aPTT
D. Formation of fibrin from fibrinogen B. PT
43. The extrinsic pathway of coagulation is triggered by the C. thrombin time
entry of _____ into the circulation. D. bleeding time
A. membrane lipoproteins (phospholipoproteins) 52. Neither the aPTT nor the PT detects a deficiency of
B. tissue thromboplastin A. platelet factor 3
C. Ca2+ B. factor VII
D. factor VII C. factor VIII
44. The intrinsic pathway of coagulation begins with the D. factor IX
activation of _____ in the early stage. 53. The function of thromboplastin in the prothrombin test
A. factor II is to provide _____ to the assay.
B. factor I A. kaolin
C. factor XII B. fibrinogen
D. factor V C. phospholipoprotein
45. The final common pathway of the intrinsic-extrinsic D. thrombin
pathway is 54. An abnormally prolonged aPTT may indicate
A. factor X activation A. a severe depletion of fibrinogen
B. factor II activation B. the presence of a circulating anticoagulant
C. factor I activation. C. factor VIII deficiency
D. factor XIII activation. D. all of the above

(continued)

Turgeon_Chap23.indd 428 10/15/2010 10:13:47 AM


CHAPTER 23 ■ Principles of Hemostasis and Thrombosis 429

REVIEW QUESTIONS (continued)

55. If a child ingested rat poison, which of the following 60. If heparin therapy is initiated in a patient, a decreased
tests should be performed to test the effect of the poison anticoagulant response can be caused by decreased levels
on the child’s coagulation mechanism? of
A. aPTT A. platelet factor 3
B. PT B. platelet factor 4
C. Fibrinogen assay C. antithrombin III
D. Thrombin time D. factor XIII
56. Which of the following conditions can cause an increased 61. Which of the following is (are) characteristic of protein
thrombin time? C?
A. Fibrin split products A. It is not vitamin K dependent
B. High concentrations of immunoglobulins B. It is formed in response to thrombin generation
C. Heparin therapy C. It inactivates factors Va and VIIIa
D. All of the above D. Both B and C
57. Heparin inhibits the clotting of blood by neutralizing 62. Which of the following characteristics is (are) true of
the effect of protein S?
A. thrombin A. It is a cofactor of protein C
B. calcium ions B. It increases the rate of inactivation of factor Va
C. platelets C. It enhances the binding of APC to phospholipids
D. factor VIII D. All of the above
58. A patient has a prolonged aPTT and a normal PT. The 63. Antithrombin III is the principal physiological inhibitor
aPTT is not corrected by factor VIII–deficient plasma of
but is corrected by factor IX–deficient plasma. In which A. thrombin
factor does the patient appear to be deficient? B. factor Xa
A. Factor II C. factor XIa
B. Factor V D. both A and B
C. Factor VIII 64. Which of the following is not correct regarding cellular
D. Factor IX proteases?
59. The normal protective mechanisms against thrombosis A. They block the activation or action of plasmin
include B. They include alpha-2 inhibitor
A. the flow of blood C. They rapidly neutralize the fibrinolytic properties of
B. the action of antithrombin III. plasmin
C. protein C and protein S D. They participate in clot formation
D. all of the above

BIBLIOGRAPHY Castellone D. Fundamentals of coagulation testing, Adv Admin Lab,


10(2):47–51, 2001.
Babich V, et al. Selective release of molecules from Weibel-Palade Castellone D. Reflections in coagulation, Adv Admin Lab, 11(12):
bodies during a lingering kiss, Blood, 111(11):5282–5290, 2008. 17–20, 2002.
Baudhuin LM. Warfarin pharmacogenetics: ready for clinical utility? Castellone D. INR’s impact on coagulation, Adv Admin Lab, 14(5):
Clin Lab Sci, 22(3):151–155, 2009. 36–44, 2005.
Beyer LK, Santrach PJ. The basics of rapid coagulation, Adv Lab, Castellone D. Clinical decision support systems in coagulation, Adv
14(10):52–58, 2005. Lab, 62–66, 2005.
Bick RL. Oral anticoagulants in thromboembolic disease, Lab Med, Castellone D. Progress continues in coagulation, Adv Med Lab Prof,
26(3):188–193, 1995. 18–29, 2006
Bode AP, et al. Evaluation of Platelet Function, Beaumont, TX: Helena Castellone D. Will this patient bleed? Adv Med Lab Prof, 18–25,
Laboratories, 1993. 2007.
Byrne KM, et al. Platelets: key player in hemostasis, Adv Med Lab Prof, Chandler WL. For warfarin monitoring in patients with lupus inhibi-
18–21, 2006. tors, review PT method, CAP TODAY, 17(1):18, 2003.
Carrol JJ. Role of endothelial cells in coagulation, Adv Med Lab Prof, Cohen A, Rosen MH. Handbook of Microscopic Anatomy for the Health
8(1):10–18, 1996. Sciences, St. Louis, MO: Mosby, 1975:45–46.
Castellone D. Anticoagulation: Heparin, Coumadin and beyond, Adv Colman RW. Platelet receptors, Hematol Oncol Clin North Am,
Admin Lab, 11(10):82–90, 2002. 4(1):27–42, 1990.

Turgeon_Chap23.indd 429 10/15/2010 10:13:47 AM


462 PART 5 ■ Principles and Disorders of Hemostasis and Thrombosis

CASE STUDIES (continued)

referred to the outpatient laboratory for a hemoglobin, significant abnormalities. At the time of admission, she was
hematocrit, and coagulation profile. having irregular contractions.
In the delivery room, bleeding became extensive. A
Laboratory Data STAT hemoglobin, hematocrit, type and crossmatch for
Hemoglobin 10.0 g/L four units of blood, and coagulation profile were ordered.
Hematocrit 27% Laboratory Data
Her coagulation profile results were as follows: Hemoglobin 10.0 g/L
Hematocrit 27%
Bleeding time 7 minutes (normal, 1 to 3 minutes) Platelet count 75 × 109/L
PT 11 seconds (control, 12.2 seconds) Bleeding time 10 minutes
aPTT 29 seconds (control, 34 seconds) aPTT 65 seconds (control, 29 seconds)
Clot retraction decreased PT 19 seconds (control, 11 seconds)
Questions Thrombin time 24 seconds (normal, 18 to 22 seconds)
1. What additional tests would be suggested based on the Fibrinogen 90 mg/dL (normal, 200 to 400 mg/dL)
initial laboratory results? FSP screen positive
2. What would the Wright-stained blood film look like? Protamine sulfate test positive
3. What is the most likely diagnosis and prognosis? Questions
1. What is the most probable cause of the extensive bleed-
Discussion
ing in this case?
1. A platelet count and qualitative platelet studies would
2. What is the etiology of this disorder?
be appropriate follow-up procedures in view of the pro-
3. Will the transfusion of whole or fresh blood repress the
longed bleeding time and poor clot retraction.
bleeding?
2. If a platelet disorder is suspected, the peripheral blood
smear may be valuable. Obviously, decreases in platelets Discussion
would support the quantitative assay and an abnormality 1. The laboratory test results support the condition of
in platelet size could be detected. In this case, both the fibrinolysis, specifically DIC. In secondary fibrinolysis as
distribution and morphology of the platelets appeared compared with primary fibrinolysis, the platelet count is
normal. decreased and the protamine sulfate test result is positive.
3. Further testing for platelet function revealed a deficiency 2. The release of placental tissue into the maternal circu-
in both platelet aggregation and adhesion. The diagnosis lation can trigger the coagulation mechanism. When
of Glanzmann thrombasthenia was made. This auto- this stimulation supersedes the body’s natural protec-
somal recessive disorder usually becomes less severe as tive mechanism, secondary bleeding occurs. In this case,
a patient ages. In this woman’s case, severe bleeding or the consumption of coagulation factors is evident in the
future surgical interventions would need to be supported laboratory findings.
by the use of platelet concentrates. 3. Although transfusions may temporarily replace the
blood volume lost through bleeding, they will not allevi-
ate the problem. Overwhelming fibrinolysis can be fatal.
DIAGNOSIS: Glanzmann Thrombasthenia
In many cases, heparin is administered to stop the cycli-
cal process that is in progress.
CASE 24.5
A woman was admitted in labor to the obstetrical unit at DIAGNOSIS: Disseminated Intravascular Coagulation
11 PM. Her history and physical examination revealed no (DIC)

REVIEW QUESTIONS

1. Which of the following is a condition associated with Questions 2 through 4: Match the following platelet dis-
purpura? orders with the appropriate morphology (use an answer
A. Direct endothelial damage only once).
B. Inherited disease of the connective tissue 2. _____ Wiskott-Aldrich syndrome
C. Mechanical disruption of small venules 3. _____ May-Hegglin anomaly
D. All of the above

(continued)

Turgeon_Chap24.indd 462 10/15/2010 3:34:27 PM


CHAPTER 24 ■ Disorders of Hemostasis and Thrombosis 463

REVIEW QUESTIONS (continued)

4. _____ Bernard-Soulier syndrome 12. The hallmark of secondary fibrinolysis is the


A. Giant platelets presence of
B. Smallest platelets seen A. fibrin split products
C. Large platelets B. fibrin degradation products
C. fibrin monomers
Questions 5 through 7: Match the etiologies of these plate- D. all of the above
let dysfunctions with the appropriate associated disorder 13. DIC is characterized by
(use an answer only once). A. microvascular thrombosis
5. _____ Acquired B. fibrin deposition
6. _____ Drug induced C. active fibrinolysis
7. _____ Hereditary D. all of the above
A. Aspirin
14. Which of the following factors can contribute to hyper-
B. von Willebrand disease
coagulation?
C. Uremia
A. Vascular endothelial damage
8. Which of the following parameters can be abnormal in B. Increased blood flow
classic von Willebrand disease type I? C. Decreased platelets
A. Bleeding time D. Decreased titers of clotting factors
B. PT
C. Platelet count Questions 15 through 19: Match the following.
D. All of the above 15. _____ Antithrombin III deficiency
9. The most common form of von Willebrand disease is 16. _____ Oral contraceptives
A. type I 17. _____ Protein C deficiency
B. type II 18. _____ Cancer
C. type III 19. _____ Pregnancy
D. all have about the same incidence A. Primary hypercoagulable state
10. Laboratory results in acute DIC reflect abnormalities B. Secondary hypercoagulable state
in which of the following coagulation components?
A. Platelet function Questions 20 through 22: Match the following terms with
B. Excessive clotting and fibrinolysis the appropriate description.
C. Accelerated thrombin formation 20. _____ Circulating anticoagulants
D. Fibrin formation 21. _____ LA
22. _____ Factor VIII inhibitor
11. Primary fibrinolysis is characterized by
A. The most common specific factor inhibitor
A. gross activation of the fibrinolytic mechanism
B. Acquired inhibitors of clotting proteins
B. consumption of fibrinogen
C. Also known as antiphospholipid or
C. consumption of coagulation factors
D. all of the above
anticardiolipin

BIBLIOGRAPHY Bromberg ME. Immune thrombocytopenic purpura-the changing


therapeutic landscape, N Engl Med, 355(16):1643–1645, 2006.
Adcock DM. Is there a genetic relationship between arterial and venous Canfield P, Perotta PL. Recognizing HIT, Adv Lab, 16(2):42–43, 2007.
thrombosis? Clin Lab Sci, 20(4):221–223, 2007. Castellone D. A case study in coagulation, Adv Lab, 15(1):51–53,
Agapitov AV, Haynes WG. Role of endothelin in cardiovascular dis- 2006.
ease, J Renin Angiotens Aldost Syst, 3(1):1–15, 2002. Cazzola M. Molecular basis of thrombocytosis, Haematologica,
Arepally GM, Ortel TL. Heprin-induced thrombocytopenia, N Engl 93(5):646–648, 2008.
Med, 355(8):809–817, 2006. Dahlbäck B. Advances in understanding pathogenic mechanisms of
Aster RH, Bougie DW. Drug induced immune thrombocytopenia, N thrombophilic disorders, Blood, 112(1):19–27, 2008.
Engl Med, 357(6):580–587, 2007. Danese S, et al. The protein C pathway in tissue inflammation and
Bates SM, Ginsberg JS. Treatment of deep-vein thrombosis, N Engl injury: pathogenic role and therapeutic implications, Blood, 115(6),
Med, 351(3):268–277, 2004. 1121–1130, 2010.
Bernard GR, et al. Efficacy and safety of recombinant human activated Drygalski AV, et al. Vancomycin-induced immune thrombocytopenia,
protein C for severe sepsis, N Engl J Med, 344(10):699–709, 2001. N Engl Med, 356(9):904–910, 2007.

Turgeon_Chap24.indd 463 10/15/2010 3:34:28 PM


CHAPTER 25 ■ Body Fluid Analysis 493

and transport nutrients to the articular cartilage. Impaired allow for consideration or exclusion of rheumatoid arthritis
function of synovial fluid with age or disease may play a role and SLE. Synovial fluid analysis can also support a diagno-
in the development of degenerative joint disease (osteoarthri- sis of diseases as disparate as amyloidosis, hypothyroidism,
tis). A variety of disorders produce changes in the number and ochronosis, hemochromatosis, and even simple edema. In
types of cells and the chemical composition of the fluid. addition, arthrocentesis may alleviate elevated intra-articu-
Analysis of synovial fluid plays a major role in the diag- lar pressure.
nosis of joint diseases. Arthrocentesis constitutes a liquid
biopsy of the joint. Analysis of aspirated synovial fluid is Body Fluid Slide Preparation
essential in the evaluation of any patient with joint disease
because it provides a better reflection of the events in the A differential cell count on a body fluid should be performed on
articular cavity than do blood tests. For example, abnormal stained smears prepared from a concentrated preparation—not
test results such as ANA, increased ESR, elevated uric acid in a hemocytometer. Ordinary centrifugation can be used to
level, and rheumatoid factor can be seen in healthy individu- concentrate cellular elements in the sediment, and slides can
als or in those with unrelated joint diseases. Disorders such be prepared with the traditional push method. More effective
as gout, CPPD deposition disease, and septic arthritis can methods of concentrating cells include sedimentation, cyto-
be diagnosed definitively by synovial fluid analysis and may centrifugation, and filtration.

REVIEW QUESTIONS

Questions 1 to 5: Match each of the terms with their 13. CSF production is associated with the
appropriate synonyms. A. arachnoid mater and pia mater
1. _____ CSF B. choroid plexus and ependymal lining
2. _____ Synovial fluid C. arachnoid mater and subarachnoid space
3. _____ Peritoneal fluid D. subarachnoid space and pia mater
4. _____ Pericardial fluid 14. CSF is collected from an intervertebral space between
5. _____ Pleural fluid the _____ and _____ vertebrae.
A. Lumbar puncture fluid A. T4, T5
B. Joint fluid B. L2, L3
C. Chest fluid C. L3, L4
D. Ascitic fluid D. L4, L5
E. Fluid from around the heart
Questions 15 to 17: Match the following test tube aliquots
Questions 6 to 8: Match the fluids with the appropriate of CSF with the typical type of testing that should be
normal characteristics. performed.
6. _____ CSF 15. _____ Tube 1
7. _____ Synovial fluid 16. _____ Tube 2
8. _____ Seminal fluid 17. _____ Tube 3
A. Clear and yellow A. Gross examination, cell count, and morphology
B. Turbid and viscous B. Microbial examination
C. Clear and colorless C. Chemical and serological examination
Questions 9 to 11: Match the fluids and normal total leu- Questions 18 to 21: Match the following gross examina-
kocyte or total sperm count. tion findings of CSF with the appropriate diagnosis.
9. _____ CSF
18. _____ Cloudy and turbid
10. _____ Synovial fluid
19. _____ Grossly bloody specimen
11. _____ Seminal fluid
20. _____ Xanthochromia (yellow color)
A. 0 to 10 × 106/L
21. _____ Gel formation
B. 60 to 150 × 109/L
A. Increased fibrinogen
C. Less than 200/mL
B. Subarachnoid hemorrhage
12. The anatomical structures associated with the circula-
C. Subarachnoid hemorrhage (more than 12 hours
tion of CSF are after the bleed)
A. ventricles and subarachnoid spaces
D. Pleocytosis
B. subarachnoid space and pia mater
C. ependyma and pia mater
D. arachnoid mater and pia mater

(continued)

Turgeon_Chap25.indd 493 10/15/2010 12:54:23 PM


494 PART 6 ■ Fundamentals of Hematological Analysis

REVIEW QUESTIONS (continued)

Questions 22 to 26: Match the following microscopic find- C. pH 7.35 to 7.45 and protein concentration >3.0 g/dL
ings of CSF with the associated condition. D. lactic dehydrogenase <200 IU/L and protein concen-
22. _____ Intraventricular rupture of brain abscess tration >3.0 g/dL
23. _____ Viral infection
24. _____ 0 to 5 × 106/L Questions 34 to 36: Match the term with the appropriate
25. _____ Bacterial infection physical description.
26. _____ CNS leukemia or lymphoma 34. _____ Pleura
A. Lymphocytosis 35. _____ Peritoneum
B. Increased polymorphonuclear segmented 36. _____ Pericardium
neutrophils (PMNs) A. Covers abdominal walls and viscera of the abdomen
C. Macrophages B. Covers the lungs
D. Extremely elevated leukocyte count in CSF C. A fibrous sac around the heart
E. Normal leukocyte reference range for CSF 37. Conditions not associated with pleural effusion include
27. Normal CSF contains A. tuberculosis
A. lymphocytes and ependymal cells B. infectious diseases
B. ependymal and choroidal cells C. mesothelioma
C. mesothelial and ependymal cells D. viral pneumonia
D. erythrocytes and leukocytes
Questions 38 to 42: Match the representative exudate
28. The cell count on a CSF specimen should be performed
appearance with a typical associated disorder.
within _____ of collection. 38. _____ Yellow and turbid
A. 30 minutes
39. _____ Milky
B. 1 hour
40. _____ Bloody
C. 2 hours
41. _____ Clearly visible pus
D. 12 hours
42. _____ Foul odor
E. 24 hours
A. Empyema
29. Clotting in CSF may be caused by
B. Infectious process
A. increased protein concentration
C. Anaerobic bacterial infection
B. increased electrolyte concentration
D. Chylothorax
C. increased glucose concentration
E. Malignancy in the absence of trauma
D. the presence of bacteria
43. Pleural fluid can have a white supernatant fluid after
30. An increased total leukocyte count in a CSF specimen
centrifugation owing to
can be caused by A. increased concentration of leukocytes
A. bacterial meningitis
B. presence of lipids
B. viral meningoencephalitis
C. presence of chylomicrons
C. intravascular rupture of a brain abscess
D. both A and B
D. both A and C
44. An extremely elevated leukocyte concentration in pleu-
31. An increase in the number of lymphocytes in a CSF
ral fluid is typically associated with
specimen can be caused by A. hemothorax
A. multiple sclerosis
B. malignancy
B. viral meningoencephalitis
C. empyema
C. fungal meningitis
D. classic rheumatoid effusion
D. all of the above
45. Which of the following cells can be seen in pleural
32. Which of the following is (are) characteristic of an
fluid?
effusion? A. LE cells
A. Abnormal accumulation of fluid
B. Mononuclear phagocytes
B. Can be a transudate
C. Mesothelial cells
C. Can be an exudate
D. All of the above
D. All of the above
46. All of the following describe the characteristics of malig-
33. A transudate can be described as
nant cells except
A. specific gravity >1.016, low to moderate number of
A. multiple round aggregates of cells
leukocytes, and lactic dehydrogenase <200 IU/L B. high N:C ratio
B. specific gravity <1.016, pH 7.4 to 7.5, and lactic de-
C. large, irregular nucleoli
hydrogenase <200 IU/L D. smooth chromatin

(continued)

Turgeon_Chap25.indd 494 10/15/2010 12:54:24 PM


CHAPTER 25 ■ Body Fluid Analysis 495

REVIEW QUESTIONS (continued)

Questions 47 to 50: Match the cellular abnormality Questions 59 to 61: Match an increase in the following cells
encountered in pleural and peritoneal fluids with a repre- in peritoneal fluid with the representative abnormality.
sentative disorder (use an answer only once). 59. _____ Eosinophils
47. _____ Many neutrophils, histiocytes, and mesothelial cells 60. _____ Lymphocytes
48. _____ Abundant, multinuclear cells and clusters of cells 61. _____ Mesothelial cells
49. _____ Many malignant cells (in clusters) A. Chronic peritoneal dialysis
50. _____ Many lymphocytes, mesothelial cells, histiocytes, B. Congestive heart failure, cirrhosis, and nephrotic
and plasma cells syndrome
A. Viral infection C. Tuberculous peritonitis
B. Acute bacterial inflammation
C. Metastatic adenocarcinoma Questions 62 to 64: Match the various types and respective
D. Malignant mesothelioma causes of pericardial effusion.
E. Chronic granulomatous inflammation 62. _____ Infectious agents
63. _____ Collagen vascular disease
Questions 51 and 52: In a pleural effusion, the percentage 64. _____ Neoplastic disease
of (51) _____ is extremely high in pneumonia and the per- A. Rheumatic disease
centage of (52) _____ is extremely high in viral peritonitis. B. Mesothelioma
51. A. polymorphonuclear segmented neutrophils C. Dressler postinfarction syndrome
B. eosinophils D. Coxsackie group viruses
C. basophils 65. A cause of an increased concentration of cells in
D. monocytes pericardial fluid is
52. A. polymorphonuclear segmented neutrophils A. microbial infection
B. eosinophils B. malignancy
C. basophils C. congestive heart failure
D. lymphocytes D. both A and B
53. The causes of peritoneal effusion include all of the fol-
lowing except Questions 66 to 69: Match the following male reproduc-
A. bacterial peritonitis tive structures with their constituents.
B. hepatic cirrhosis 66. _____ Testicle
C. congestive heart failure 67. _____ Seminal vesicles
D. tuberculosis 68. _____ Prostate
54. An abnormal-appearing peritoneal effusion can be 69. _____ Cowper glands
caused by all of the following except A. Fructose and prostaglandins
A. bacterial peritonitis B. Unknown
B. pancreatitis C. Sperm
C. neoplasm D. p30 glycoprotein
D. tuberculous peritonitis 70. Sperm motility can become decreased if the specimen is
A. stored at room temperature
Questions 55 to 57: Match the following peritoneal B. stored in a plastic container for more than 1 hour
effusion colors with the respective condition (use each C. examined after 2 hours of storage
answer once). D. all of the above
55. _____ Pale yellow 71. The normal value of sperm cells is _____ × 109/L.
56. _____ Straw colored A. 15 to 30
57. _____ Bloody B. 30 to 45
A. Normal C. 30 to 60
B. Pulmonary infarct D. 60 to 150
C. Congestive heart failure
58. An extremely increased leukocyte concentration in peri- Questions 72 to 76: Match the normal values or appropri-
toneal fluid can be caused by ate term.
A. bacterial peritonitis 72. _____ Motility (fresh specimen)
B. pancreatitis 73. _____ Sperm morphology
C. cirrhosis 74. _____ Viability (fresh specimen)
D. none of the above 75. _____ Agglutination

(continued)

Turgeon_Chap25.indd 495 10/15/2010 12:54:24 PM


496 PART 6 ■ Fundamentals of Hematological Analysis

REVIEW QUESTIONS (continued)

76. _____ Artificial insemination A. total cell count and differential count
A. At least 50% B. crystal examination
B. 40% to 90% (mature and oval headed) C. Gram stain and culture
C. Test for infectious disease D. all of the above
D. Prostatitis or sperm-agglutinating antibodies 81. Crystals that are in multiple three-dimensional forms
E. Greater than 60% are
77. Arthrocentesis is A. CPPD crystals
A. a bone biopsy B. BCP crystals
B. a liquid biopsy C. MSU crystals
C. not as accurate as blood testing D. cholesterol
D. a good test to monitor the effects of chemotherapy 82. An increased percentage of polymorphonuclear seg-
78. Disorders that can be diagnosed definitively by synovial mented neutrophils (PMNs) is characteristic of
fluid analysis are A. chronic urticaria
A. gout, CPPD deposition disease, and rheumatoid ar- B. septic arthritis
thritis C. rheumatoid arthritis
B. CPPD deposit disease, rheumatoid arthritis, and D. rheumatic fever
SLE
C. rheumatoid arthritis, SLE, and septic arthritis Questions 83 to 86: Match the following crystals with an
D. gout, CPPD deposition disease, and septic arthritis associated disorder (use each answer once).
79. Which of the following would not be an aspiration site 83. _____ MSU
for synovial fluid? 84. _____ Calcium oxalate
A. Knee 85. _____ Cholesterol
B. Elbow 86. _____ Lipid liquid “maltese cross”
C. Posterior iliac crest A. Chronic renal disease
D. Ankle B. Chronic rheumatoid effusions
80. If a synovial fluid aspirate is very turbid and septic arthri- C. Acute and chronic arthritis
tis is suspected, a _____ should definitely be performed. D. Acute gouty arthritis

BIBLIOGRAPHY Judkins SW, Cornbleet PJ. Synovial fluid crystal analysis, Lab Med,
28(12):774–779, 1997.
Baker DJ. Performing a quality semen analysis in the clinical labora- Krieg AF, Kjeldsberg CR. Cerebrospinal fluid and other body fluids.
tory, Med Lab Observ, 32(12):20–31, 2000. In: Henry JB (ed.). Clinical Management and Diagnosis by Labora-
Baker DJ. Questions about semen analysis, Med Lab Observ, 33(4): tory Methods, Philadelphia, PA: Saunders, 1991:445–457, 463–469.
5,2001. Lipsky PE. Rheumatoid arthritis. In: Wilson JD (ed.). Harrison’s Prin-
Baker DJ. Semen analysis, Clin Lab Sci, 20(3):172–187, 2007. ciples of Internal Medicine, 12th ed, New York, NY: McGraw-Hill,
Barry E. Risk to offspring is found in male biological clock, The Bos- 1990:1437–1471.
ton Globe, (www.boston.com/dailyglobe). Retrieved December 4, Lyons MK, Meyer FB. Cerebrospinal fluid physiology and the man-
2002. agement of increased intracranial pressure, Mayo Clin Proc, 65:
Braunwald E, et al. Ascites. In: Wilson JD (ed.). Harrison’s Princi- 684–707, 1990.
ples of Internal Medicine, 12th ed, New York, NY: McGraw-Hill, McCarty DJ. Arthritis associated with calcium-containing crystals. In:
1988:70–74. Stein JH (ed.). Internal Medicine, Boston, MA: Little, Brown & Co.,
Cannon DC, Henry JB. Seminal fluid. In: Henry JB (ed.). Clinical 1990:1809–1813.
Diagnosis and Management by Laboratory Methods, 18th ed, Phila- Nosanchuck JS, Kim CW. Lupus erythematosus cells in CSF, JAMA,
delphia, PA: Saunders, 1991:497–503. 25:2883–2884, 1976.
Eastern JD. Spinal fluid examination. In: Stein JH (ed.). Internal Medi- Oehmichen M, et al. Origin, proliferation and fate of cerebrospinal
cine, Boston, MA: Little, Brown & Co., 1990:1870–1871. fluid cells, J Neurol, 227:145–150, 1982.
Guzick DS, et al. Sperm morphology, motility, and concentration in Plaut D. Analysis of body fluids, Adv Lab, 14(11):46–52, 2005.
fertile and infertile men, N Engl J Med, 345(19):1388–1393, 2001. Pleural effusions. In: Medical Knowledge Self-Assessment Program VIII,
Hoffman GS. Arthritis due to deposition of calcium crystals. In: Wil- Pulmonary Medicine, Part A, Book 6. Philadelphia, PA: American
son JD (ed.). Harrison’s Principles of Internal Medicine, 12th ed, College of Physicians, 1988:231.
New York, NY: McGraw-Hill, 1990:1479–1482. Rheumatology. In: Medical Knowledge Self-Assessment Program VIII,
Jones CD, Cornbleet PJ. Wright-Giemsa cytology of body fluids, Lab Part B, Book 6. Philadelphia, pA: American College of Physicians,
Med, 28(11):713–716, 1997. 1988.

Turgeon_Chap25.indd 496 10/15/2010 12:54:24 PM


522 PART 6 ■ Fundamentals of Hematological Analysis

COAGULATION PROCEDURES (continued)

TABLE 26.4 Probable Coagulation Deficiencies Based on APTT and PT Test Results

Deficient Factor

Test V VII VIII IX X XI or XII


PT Abnormal Abnormal Normal Normal Abnormal Normal
APTT Abnormal Normal Abnormal Abnormal Abnormal Abnormal
Adsorbed plasma Corrects No change Corrects No change No change Corrects
Aged serum No change Corrects No change Corrects Corrects Corrects
PT, Prothrombin time; APTT, activated partial thromboplastin time.

3. Add 0.1 mL of plasma to 0.2 mL of thromboplastin. If BIBLIOGRAPHY


performing this procedure manually, pipette quickly.
Start a stopwatch simultaneously. Provided on this book’s companion Web site at thepoint.
4. Using the nichrome loop technique, the loop is swept lww.com/Turgeon5e.
through the mixture at 2 sweeps per second until the first
strand of fibrin appears. The tube may also be tilted using THROMBIN TIME
a magnifier to observe clot formation. Principle
5. Repeat this procedure in duplicate for all specimens, The thrombin time test determines the rate of
including controls. The duplicate results should be within thrombin-induced cleavage of fibrinogen to fibrin mono-
1 second of one another. mers and the subsequent polymerization of hydrogen-
Reporting Results bonded fibrin polymers. Clinically, extremely low fibrinogen
Reference values range from 10 to 15 seconds. Report both levels, abnormal fibrinogen thrombin inhibitors, and high
the patient and control specimens in seconds. An older alter- concentrations of immunoglobulin (e.g., myeloma proteins)
native method of reporting is to express the percentage of will produce abnormal results. The presence of heparin and
patient activity. This is calculated as high concentrations of fibrin-fibrinogen degradation prod-
ucts will also prolong the time. This procedure is particu-
Control time(seconds) larly useful if other parameters, such as the APTT and PT,
´ 100 = % activity of patient are prolonged.
Patient's time(seconds)
This procedure, in CLSI format, is provided on this book’s
companion Web site at thepoint.lww.com/Turgeon5e.
Clinical Applications
Refer to the reference values on the inside back cover as well
This test depends on the activity of factors VII, V, X, II, and as additional procedures at thepoint.lww.com/Turgeon5e.
I. A deficiency of any of these may produce a 3- to 4-second
prolongation in the test (Table 26.4).

REVIEW QUESTIONS

1. What is the appropriate reagent for the reticulocyte A. The diluting solution lyses erythrocytes with
count? propylene glycol and contains sodium carbonate
A. New methylene blue and water.
B. Phyloxine B B. The procedure measures the rate of erythrocyte set-
C. Solution lyses erythrocytes and darkens the cells to tling.
be counted C. Ferrous ions are oxidized to the ferric state.
2. What is the appropriate procedure and characteristic for D. The diluting solution is either 1% hydrochloric acid
the Westergren method or 2% acetic acid.

(continued)

Turgeon_Chap26.indd 522 10/15/2010 1:00:04 PM


CHAPTER 26 ■ Manual Procedures in Hematology 523

REVIEW QUESTIONS (continued)

3. What source of error will have greatest effect on PCV Questions 18 through 21: Match the following leukocyte
(hematocrit) types with a clinical condition that will produce an increased
A. Incorrect dilution of blood and diluent value.
B. Hemolysis of whole blood specimen 18. _____ Neutrophils
C. Excessive anticoagulant will produce shrinkage of 19. _____ Lymphocytes
cells 20. _____ Monocytes
21. _____ Eosinophils
Questions 4 and 5: Match the procedure and the source of A. Invasive parasites
error that will have the greatest effect on the test result.
B. Bacterial infections
4. _____ Platelet count
C. Viral infections
5. _____ Reticulocyte count
D. Tuberculosis
A. Refractile bodies can produce a false-positive
Questions 22 and 23: Match the following procedure with a
observation.
clinical condition that will produce a decreased value.
B. Specimens stored at room temperature for more
22. _____ Reticulocyte count
than 5 hours will produce inaccurate results.
23. _____ Westergren ESR
Questions 6 through 8: Match the procedure and correct
A. Polycythemia vera
reference value.
B. Acute leukemias
6. _____ Erythrocyte count (adult male)
C. Megaloblastic anemia
7. _____ Hemoglobin assay (adult female)
24. A normal blood smear should have no more than
8. _____ Lymphocytes (adult)
approximately _____ (maximum) number of platelets
A. 0.15 to 0.3 × 109/L
per oil immersion field in an area where the erythrocytes
B. 12.0 to 16.0 g/dL
are just touching each other.
C. 4.5 to 5.9 × 1012/L
A. 10
D. 22 to 40%
B. 15
Questions 9 through 13: Match the procedure and reference
C. 20
value.
D. 25
9. _____ Total leukocyte count
25. The PCV procedure can be affected by the
10. _____ PCV (adult, female)
A. speed of the centrifuge
11. _____ Direct platelet count
B. length of time of centrifugation
12. _____ Reticulocyte count (newborn infant)
C. ratio of anticoagulant to whole blood
13. _____ Westergren ESR method (adult male age 65 years)
D. all of the above
A. Up to 13 mm/hour
26. Which of the following erythrocytic inclusions con-
B. 2.5 to 6.0%
tain RNA and can be observed by staining with new
C. 150 to 450 × 109/L
methylene blue?
D. 36 to 45%
A. Howell-Jolly bodies
E. 4.4 to 11.3 × 109/L
B. Heinz bodies
14. What clinical or specimen condition will produce an
C. Pappenheimer bodies
increased total leukocyte count.
D. Reticulocytes
A. Active allergies
27. The sedimentation rate of erythrocytes can be affected
B. Immediate hypersensitivity reactions
by the
C. Inflammation
A. ratio of anticoagulant to whole blood
D. A lipemic blood specimen
B. position of the tube
Questions 15 through 17: Match the following procedures
C. temperature of the specimen or laboratory
with a clinical or specimen condition that will produce an
D. all of the above
increased test result.
Questions 28 through 31: Match the following procedures
15. _____ PCV
with the specific stains (use an answer only once).
16. _____ Reticulocyte count
28. _____ Acidified serum lysis test (Ham test)
17. _____ Westergren ESR method
29. _____ Donath-Landsteiner test
A. Splenectomy
30. _____ Alpha-naphthyl acetate esterase
B. Rouleaux formation
31. _____ Naphthol AS-D chloroacetate esterase
C. Polycythemia
A. Positive in monocytes
D. Crisis associated with hemolytic anemia
B. Measures an extremely potent hemolysin at 4°C

(continued)

Turgeon_Chap26.indd 523 10/15/2010 1:00:05 PM


524 PART 6 ■ Fundamentals of Hematological Analysis

REVIEW QUESTIONS (continued)

C. Positive in cell of granulocytic lineage Questions 48 through 51: Match the procedure with the
D. Measures hemolysis appropriate reference value or positive test result.
Questions 32 through 35: Match the following procedures 48. _____ Acidified serum lysis test (Ham test)
with the appropriate test reactions (use an answer only 49. _____ Leukocyte alkaline phosphatase
once). 50. _____ G6PD assay
32. _____ G6PD activity 51. _____ Kleihauer-Betke test
33. _____ Heinz bodies A. Normal range: 32 to 182 with fast blue RR dye
34. _____ Kleihauer-Betke test B. Positive test: 10% to 50% hemolysis
35. _____ Hemoglobin S screening test C. Normal: less than 1% Hb F in adults
A. Erythrocytes lysed by toluene and saponin with D. Zero time control should not fluoresce or fluoresce
the released product being reduced by sodium only slightly
hydrosulfite Questions 52 through 54: Match the procedure with the
B. Fetal hemoglobin is not eluted appropriate reference value or positive test result.
C. Denatured by crystal violet 52. _____ Alkaline denaturation
D. Screens for one of the most prevalent hereditary 53. _____ Prussian blue stain
enzyme deficiencies 54. _____ Sucrose hemolysis test (sugar water test)
Questions 36 through 39: Match the following procedures A. Negative: less than 10%
with the appropriate test reactions. B. Newborn normal: 70% to 90%
36. _____ Osmotic fragility C. Normal: 0% to 1% of mature erythrocytes
37. _____ Periodic acid–Schiff Questions 55 through 57: Match the procedure with the
38. _____ Peroxidase stain disorder that can be recognized through the use of the test.
39. _____ Prussian blue stain 55. _____ Acid serum lysis test (Ham test)
A. Precipitates free iron into blue or blue-green 56. _____ Leukocyte alkaline phosphatase stain
granules 57. _____ Donath-Landsteiner test
B. Observation of hemolysis in varying sodium chloride A. Positive in PCH
dilutions B. Diagnostic for paroxysmal nocturnal hemoglobinu-
C. Lymphocytes stain negative ria
D. Intense cytoplasmic granular staining in erythroleu- C. Increased in hereditary spherocytosis
kemia D. Increased in a leukemoid reaction
Questions 40 through 43: Match the procedure with a Questions 58 through 60: Match the procedure with the
possible source of error. disorder that can be recognized through the use of the test.
40. _____ Acidified serum lysis test (Ham test) 58. _____ Heinz bodies
41. _____ Donath-Landsteiner test 59. _____ Kleihauer-Betke test
42. _____ G6PD assay 60. _____ Osmotic fragility test
43. _____ Hemoglobin electrophoresis A. Detects physical alterations in the erythrocyte
A. Identical mobilities membrane
B. Use of ABO-incompatible test serum B. Positive result if unstable hemoglobins are present
C. Quenched by PCVs greater than 0.50 L/L C. Increased in cord blood samples
D. Hemolyzed specimen 61. Which of the following procedures is used to detect the
Questions 44 through 47: Match the procedure with a complement-sensitive cells in paroxysmal nocturnal
possible source of error. hemoglobinuria?
44. _____ Hemoglobin S screening test A. Sucrose lysis test
45. _____ Malaria preparation B. Donath-Landsteiner test
46. _____ Sickle cell screening C. Acidified serum lysis (Ham test)
47. _____ Sudan black B D. Both A and C
A. Superimposed platelets may produce a false-positive 62. Leukocytes that demonstrate a positive reaction in the
result. tartratic acid-resistant acid phosphatase cytochemical
B. Blood specimen from a recently transfused patient stain are the lymphocytes seen in
may produce a false-negative result. A. infectious lymphocytosis
C. Old blood specimen will produce a false-negative B. malignant lymphoma
result. C. acute lymphoblastic leukemia (non-T type)
D. Test reagent more than 1 day old will produce a D. hairy cell leukemia
false-negative result.

(continued)

Turgeon_Chap26.indd 524 10/15/2010 1:00:06 PM


CHAPTER 26 ■ Manual Procedures in Hematology 525

REVIEW QUESTIONS (continued)

63. A decreased leukocyte alkaline phosphatase (LAP) score 73. An increased number of siderocytes can be seen
is seen in A. in chronic lymphocytic leukemia
A. polycythemia vera B. in lead poisoning
B. chronic myelogenous leukemia C. after splenectomy
C. leukemoid reactions D. both B and C
D. acute myelogenous leukemia
Questions 64 through 66: Steps in leukocyte alkaline Coagulation Procedures
phosphatase scoring, in sequence, are (64) _____, (65) Questions 74 through 77: Match the following procedures
_____, and (66) _____. with the appropriate principle or description of the test.
A. adding the scores for the 100 neutrophils counted 74. _____ Activated partial thromboplastin time (APTT)
B. grading the neutrophils using a 0 to 4 pt scale 75. _____ Antithrombin III assay
C. averaging the scores for all neutrophils counted 76. _____ Bleeding time
D. multiplying the number of neutrophils in each cat- 77. _____ Circulating anticoagulant assay
egory by their respective scores A. In the presence of heparin, thrombin is neutralized
67. In the leukocyte alkaline phosphatase procedure, blood B. Measures the time required to generate thrombin
smears should be stained and fibrin polymers via the intrinsic pathway
A. within 8 hours of specimen collection C. Measures inhibitors of specific factors
B. within 48 hours of specimen collection D. An in vivo measurement of platelet adhesion and ag-
C. within 72 hours of specimen collection gregation on locally injured vascular subendothelium
D. within 5 days of specimen collection Questions 78 through 82: Match the following proce-
Questions 68 and 69: The alpha-naphthyl acetate esterase dures with the appropriate reference value or diagnostic
stain is detected primarily in (68) _____ and is almost characteristic.
absent in (69) _____. 78. _____ APTT
A. megakaryocytes 79. _____ Antithrombin III
B. monocytes 80. _____ Ivy bleeding time
C. granulocytes 81. _____ Aspirin tolerance test
D. erythrocytes 82. _____ Circulating anticoagulant
70. If many dense and dark-staining cells are seen in the A. Positive result: increased ratio of normal plasma to
Kleihauer-Betke test, the specimen could be from a patient plasma
patient with B. Normal: 2 to 8 minutes
A. beta-thalassemia C. Normal: 20 to 35 seconds (28 to 42 seconds), the
B. hereditary persistence of fetal hemoglobin range depending on the activator and phospholipid
C. sickle cell anemia reagents
D. all of the above D. Increased twofold in 92% to 95% of patients after
71. The reagent used in the traditional sickle cell screening ingesting two tablets of salicylate
test is E. Normal: 80% to 100% (range, 107 ± 19%)
A. sodium chloride Questions 83 through 86: Match the following procedures
B. sodium citrate with the appropriate reference value.
C. sodium metabisulphite 83. _____ Factor VIII assay
D. sodium-potassium oxalate 84. _____ Fibrin split products
72. Which of the following is a nucleated erythrocyte with 85. _____ Fibrin-stabilizing factor
diffuse iron in the cytoplasm? 86. _____ Fibrinogen assay
A. Ring sideroblast A. Normal: no dissolution of the clot at 24 hours
B. Sideroblast B. Normal: less than 8 to 10 mg/mL
C. Pappenheimer bodies C. Normal: 50% to 150%
D. Siderocyte D. Normal: 200 to 400 mg/dL or a titer of 1:123 to
1:256

Turgeon_Chap26.indd 525 10/15/2010 1:00:06 PM


CHAPTER 27 ■ Instrumentation in Hematology 569

CASE STUDIES (continued)

only measured component that was abnormal was the


decreased platelet count.
2. The histogram is also abnormal. It demonstrates a huge
population of cells in the lymphocyte region extending
through the regions of lymphocytes, variant lympho-
cytes, monocytes, and the upper monocyte region. Cells
associated with this region include blasts, large mono-
cytes, promyelocytes, and myelocytes.
3. Yes. A further workup is needed to establish the throm-
bocytopenia in the presence of red and white blood cells
within the reference range. The presence of blasts and
many mononuclear cells believed to be lymphocytes needs
to be investigated. Either special staining or flow cytom-
Reprinted with permission from McClatchey KD. Clinical etry should be used to further identify the hematologic
Laboratory Medicine, 2nd ed, Philadelphia, PA: Lippincott abnormality in peripheral blood and/or bone marrow.
Williams & Wilkins, 2002.
DIAGNOSIS: Thrombocytopenia Etiology Unknown
2. Is the histogram abnormal? with Increased Number of Mononuclear
3. Should additional laboratory assays be ordered? Peripheral Blood Cells

■ Discussion
1. Yes. The peripheral blood differential revealed imma-
ture white blood cells and decreased platelets. The

REVIEW QUESTIONS

1. Which of the following is not a benefit of laboratory 6. The major application of flow-cell cytometry is
instrumentation to the hematology laboratory? A. determining cell size and granularity
A. Produces faster results from specimens B. sorting of cells and cellular identification using
B. Reduced cost on rarely performed procedures monoclonal antibodies
C. Less variation in technique from technologist to C. treating cancer cells and identifying specific virus
technologist types
D. Increased accuracy because data are collected on D. counting leukocytes and platelets
more cells counted or analyzed 7. The term parameter means
Questions 2 and 3: Match the following principles of cell A. a subset of a population
counting instrumentation. B. the mean value of a sample
2. _____ Electrical impedance principle C. two SDs on either side of the mean value
3. _____ Optical detection principle D. any numerical value that describes an entire
A. The volume of each cell is proportional to the degree population
of light scatter 8. Data output from three-part differential counters
B. Each cell momentarily increases resistance includes
4. The abbreviation laser stands for A. an erythrocyte histogram
A. light-associated simulated emission of radiation B. a leukocyte histogram
B. largely amplified by simulated emission of radiation C. a platelet histogram
C. light amplified by stimulated emission of radiation D. all of the above
D. liquid amplified by stimulated emission of radiation 9. Which parameters are calculated rather than directly
5. A photon is measured?
A. a diffuse form of energy A. Hematocrit and erythrocyte distribution width
B. a piece of equipment in a laser assembly B. Erythrocyte count and leukocyte count
C. the basic unit of all radiation C. Leukocyte count and hematocrit
D. equivalent to an atom D. Platelet count and platelet volume

(continued)

Turgeon_Chap27.indd 569 10/13/2010 8:12:45 PM


570 PART 6 ■ Fundamentals of Hematological Analysis

REVIEW QUESTIONS (continued)

10. The delta check method of quality control 18. The ____ can be determined from a WBC histogram.
A. uses the patient’s own data to monitor population A. percent of lymphocytes
values B. absolute number of lymphocytes
B. uses batches of 20 samples to track MCV, MCH, and C. frequency distribution of granulocytes
MCHC values D. all of the above
C. compares the patient’s leukocyte and platelet counts Questions 19 and 20: The sorting of leukocyte
with his or her previous results subpopulations in the WBC histogram determined by elec-
D. monitors the patient’s values within two SDs of the mean trical impedance reflects the (19) _____, which is primarily
11. Applying the optical principle of laser scatter technology related to their (20) _____.
to cell counting and analysis, discrimination between 19.
erythrocytes and platelets depends on the A. overall size
A. cellular volume B. relative size
B. cellular refractive index C. nuclear size
C. time of flight through the sensing zone D. chromatin pattern
D. all of the above 20.
12. In an erythrocyte histogram, the erythrocytes that are A. cytoplasmic size
larger than normal will be to the _____ of the normal B. nuclear size
distribution curve. C. concentration of granules
A. right D. cytoplasmic color
B. left 21. The mononuclear cells in a WBC histogram can include
C. in the middle A. blast cells
13. A bimodal histogram distribution is suggestive of B. promyelocytes
A. cold agglutinin disease C. monocytes
B. posttransfusion of normal red cells to a person with D. all of the above
abnormally sized red cells 22. A combined scatter histogram measures
C. the presence of RBC fragments A. overall size versus nuclear size
D. all of the above B. cytoplasm-to-nucleus ratio
Questions 14 and 15: Match the appropriate formulas. C. cell size and granularity
14. _____ RDW D. cell shape and cytoplasmic color
15. _____ Red cell measurement index 23. The MPV is
A. analogous to the MCHC
Patient RBC variaton − average normal RBC variation B. a direct measure of the platelet count
A. C. a measurement of the average volume of platelets
SD of average normal RBC variation
D. a comparison of the patient’s value to the normal
value
SD
B. × 100 24. The MPV is often decreased
Mean A. in sickle cell anemia
B. in megaloblastic anemia
16. The RDW and MCV are both quantitative descriptors of C. in idiopathic thrombocytopenic purpura
erythrocyte size. If both are increased, the most probable D. after splenectomy
erythrocytic abnormality would be 25. A normal PDW is
A. iron deficiency anemia A. less than 5%
B. acquired aplastic anemia B. less than 10%
C. megaloblastic anemia C. less than 15%
D. hemoglobinopathy D. less than 20%
17. If the RBC distribution on a histogram demonstrates 26. Which of the following can be an application of flow-cell
a homogeneous pattern and a small SD, the peripheral cytometry?
blood smear would probably exhibit A. Screening erythrocytes for malaria
A. extreme anisocytosis B. Counting of reticulocytes
B. very little anisocytosis C. Quantitation of T and B cells
C. a single population of spherocytes D. All of the above
D. a single population of macrocytes

(continued)

Turgeon_Chap27.indd 570 10/13/2010 8:12:46 PM


CHAPTER 27 ■ Instrumentation in Hematology 571

REVIEW QUESTIONS (continued)

27. Major systems in a flow cytometer include all of the 32. The fibrometer relies on the principle of
following except A. clot elasticity
A. fluidics B. fibrin adhesion
B. optics C. conduction or impedance of an electrical current by
C. computerized electronics fibrin
D. gating D. changes in optical density
28. The restriction of data analysis to one cell population is 33. In the photo-optical method, the change in light trans-
accomplished by mission versus the _____ is used to determine the activ-
A. amplification ity of coagulation factors or stages.
B. gating A. amount of patient’s plasma
C. compensatory monitoring B. amount of test reagent
D. data limitation C. time
29. Which cell surface membrane marker is used for enu- D. temperature
meration of HPC enumeration? 34. In measuring platelet aggregation, platelet-rich plasma
A. CD4 can be treated with _____ to aggregate platelets.
B. CD8 A. saline
C. CD34 B. collagen
D. CD45 C. epinephrine
30. Reticulocytes can be detected by using _____ stain. D. both B and C
A. new methylene blue 35. With a particle-counting instrument, a high background
B. thiazole orange count can be due to
C. propidium iodide A. a partial obstruction of the aperture
D. both A and B B. an electrical line interference
31. The newer clinical instruments for measuring blood C. contaminated diluent
clotting are based on D. bubbles in the diluent
A. clot elasticity 36. A source of error when using the fibrometer in coagula-
B. fibrin adhesion tion studies can be
C. conduction of impedance of an electrical current by A. improper reaction temperature
fibrin B. overincubation of the substrate reagent
D. changes in optical density C. overincubation of the test plasmas
D. all of the above

BIBLIOGRAPHY Dunphy CH. Applications of flow cytometry to diagnostic


hematopathology, Adv Med Lab Prof, 15(3):19–21, 2003.
Aller R (ed.). Coagulation analyzers, CAP TODAY, 1:19–34, 2009. Ford A. High-Volume hematology analyzers, TODAY, 17(12):33–49, 2003.
Bakke AC. ASCP Tech Sample Hematology No. H-6, 287–235, 2002. Gonder J, Mell LD. Everything you wanted to know about automated
Bakke AC. The principles of flow cytometry, Lab Med, 32(4):207–211, instrument selection, but were afraid to ask, Adv Med Lab Prof,
2001. 14(21):17–20, 2002.
Boraiko AA. A splendid light—lasers, Nat Geo, 1984:335–341. HemoSense Product Literature (www.hemosense.com), 2009.
Coulter Electronics, Inc. Calibrating and Caring for Semiautomated Jamieson B. The continued climb of hematology automation, Adv
Models of the Coulter Counter, Hialeah, FL: Coulter Electronics, Admin Lab, 12(4):56–60, 2003.
1982. Jones AR. Evaluation of the Coulter Histogram Differential: A Review of
Coulter Electronics, Inc. Platelet Counting in the ‘80s, Hialeah, FL: the Literature, Hialeah, FL: Coulter Electronics, 1986.
Coulter Electronics, 1982. Kelliher AS, et al. Multiparameter flow cytometry in the clinical
Coulter Electronics, Inc. Expanding Information, Hialeah, FL: Coulter lab: Present capacities & future projections, Adv Med Lab Prof,
Electronics, 1983. 13(17):9–12, 2001.
Coulter Electronics, Inc. Significant Advances in Hematology, Hialeah, Kottke-Marchant K, Aller RD. Towards multi-functional analyzers,
FL: Coulter Electronics, 1983 (slide-tape program). CAP TODAY, 17(1):18, 2003.
Coulter Electronics, Inc. Improved Classification of Anemias, Hialeah, LaPorta AD. Advance in hematology, Adv Med Lab Prof, 14(25):14–16,
FL: Coulter Electronics, 1985 (slide-tape program). 2002.
Dadoun R. Implementing preanalytical automation, Med Lab Observ, McCoy JP. Flow cytometry applications in diagnostic pathology, Clin
32(1):32–36, 2000. Lab News, 29(9):8–10, 2003.
DeRitis S, Smith T. Next-generation hematology, Adv Admin Lab, Orsulak PJ. Automation of pre-analytical processing in the clinical
11(11):18–25, 2002. laboratory, Adv Admin Lab, 12(1):46–49, 2003.

Turgeon_Chap27.indd 571 10/13/2010 8:12:47 PM

You might also like